SlideShare a Scribd company logo
1 of 68
Tải tài liệu tại sividoc.com
Viết đề tài giá sinh viên – ZALO:0973.287.149-TEAMLUANVAN.COM
ĐẠI HỌC THÁI NGUYÊN
TRƯỜNG ĐẠI HỌC KHOA HỌC
--------------

-------------
NGUYỄN THỊ THANH MAI
LƯỚI TỌA ĐỘ
VÀ MỘT SỐ BÀI TOÁN LIÊN QUAN
LUẬN VĂN THẠC SĨ TOÁN HỌC
THÁI NGUYÊN - 2017
Tải tài liệu tại sividoc.com
Viết đề tài giá sinh viên – ZALO:0973.287.149-TEAMLUANVAN.COM
ĐẠI HỌC THÁI NGUYÊN
TRƯỜNG ĐẠI HỌC KHOA HỌC
--------------

-------------
NGUYỄN THỊ THANH MAI
LƯỚI TỌA ĐỘ
VÀ MỘT SỐ BÀI TOÁN LIÊN QUAN
LUẬN VĂN THẠC SĨ TOÁN HỌC
Chuyên ngành: Phương pháp Toán sơ cấp
Mã số: 60 46 01 13
NGƯỜI HƯỚNG DẪN KHOA HỌC
PGS.TS. Nguyễn Việt Hải
THÁI NGUYÊN - 2017
Tải tài liệu tại sividoc.com
Viết đề tài giá sinh viên – ZALO:0973.287.149-TEAMLUANVAN.COM
i
Danh möc h…nh
1.1 L÷îi tåa º nguy¶n . . . . . . . . . . . . . . . . . . . . 3
1.2 L÷îi tåa º tr¶n m°t phflng . . . . . . . . . . . . . . . . 4
1.3 l÷îi tåa º nguy¶n vîi hbh cì sð . . . . . . . . . . . . . 4
1.4 H…nh v‡ a. H…nh v‡ b . . . . . . . . . . . . . . . . . . . 6
1.5 Ngô gi¡c v löc gi¡c . . . . . . . . . . . . . . . . . . . . 10
1.6 H…nh vuæng v h…nh b¡t gi¡c . . . . . . . . . . . . . . . 12
1.7 a gi¡c •u-c⁄nh . . . . . . . . . . . . . . . . . . . . . 14
1.8 a gi¡c •u-gâc . . . . . . . . . . . . . . . . . . . . . . 15
1.9 Tam gi¡c •u hƒu nºi ti‚p l÷îi . . . . . . . . . . . . . . 17
1.10 a gi¡c ìn v a gi¡c khæng ìn . . . . . . . . . . . . 19
1.11 Mºt sŁ b i to¡n li¶n quan . . . . . . . . . . . . . . . . . 23
2.1 Cºng v nh¥n c¡c i”m nguy¶n . . . . . . . . . . . . . . 34
.
2.2
.
36
a.C¡c i”m nguy¶n.(2 + 3i); b. C¡c i”m nguy¶n li¶n k‚t
2.3 F(R) v N(R) . . . . . . . . . . . . . . . . . . . . . . . 39
2.4 r(4)=4;r(25)=12 . . . . . . . . . . . . . . . . . . . . . . 41
2.5 n=8, n=6 . . . . . . . . . . . . . . . . . . . . . . . . . 43
2.6 n=12 . . . . . . . . . . . . . . . . . . . . . . . . . . . . 43
3.1 H…nh chœ nh“t 20 12 . . . . . . . . . . . . . . . . . . 55
Tải tài liệu tại sividoc.com
Viết đề tài giá sinh viên – ZALO:0973.287.149-TEAMLUANVAN.COM
ii
Möc löc
Líi c£m ìn ii
Mð ƒu 1
1 L÷îi tåa º v a gi¡c 3
1.1 L÷îi tåa º nguy¶n tr¶n m°t phflng v c¡c t‰nh ch§t . . 3
1.1.1 L÷îi tåa º tr¶n m°t phflng . . . . . . . . . . . . 3
1.1.2 L÷îi tåa º nguy¶n v a gi¡c •u . . . . . . . . 7
1.1.3 a gi¡c nßa •u nºi ti‚p l÷îi nguy¶n . . . . . . . 12
1.2 a gi¡c hƒu nºi ti‚p l÷îi nguy¶n . . . . . . . . . . . . . 16
1.3 Cæng thøc Picard . . . . . . . . . . . . . . . . . . . . . 19
1.3.1 Tam gi¡c ìn nguy¶n thıy . . . . . . . . . . . . 19
1.3.2 Cæng thøc Picard . . . . . . . . . . . . . . . . . 20
1.3.3 Mºt sŁ b i to¡n ¡p döng cæng thøc Picard . . . . 22
1.4 Mºt sŁ øng döng cıa l÷îi tåa º nguy¶n . . . . . . . . . 25
1.4.1 Gi¡ trà væ t cıa h m l÷æng gi¡c . . . . . . . . . 25
1.4.2 C¡c b i to¡n tr¶n æ vuæng . . . . . . . . . . . . 27
2 L÷îi tåa º nguy¶n v ÷íng trÆn 33
2.1 i”m nguy¶n nguy¶n tŁ . . . . . . . . . . . . . . . . . . 33
2.1.1 Sü chia h‚t cıa c¡c i”m nguy¶n . . . . . . . . . 35
2.1.2 ành lþ cì b£n cıa c¡c i”m nguy¶n . . . . . . . 36
2.2 ÷íng trÆn tr¶n l÷îi tåa º nguy¶n . . . . . . . . . . . 38
2.2.1 SŁ c¡c bi”u di„n cıa mºt sŁ tü nhi¶n th nh tŒng
hai b…nh ph÷ìng . . . . . . . . . . . . . . . . . . 41
Tải tài liệu tại sividoc.com
Viết đề tài giá sinh viên – ZALO:0973.287.149-TEAMLUANVAN.COM
i
2.2.2 ÷íng trÆn Sinhsel . . . . . . . . . . . . . . . . 42
2.3 Mºt sŁ øng döng v o sŁ håc . . . . . . . . . . . . . . . 46
2.3.1 Bº ba Pythagoras . . . . . . . . . . . . . . . . . 46
2.3.2 C¡c d⁄ng cıa i”m nguy¶n nguy¶n tŁ . . . . . . 47
3 C¡c b i to¡n kh¡c 49
3.1 i”m nguy¶n tr¶n ÷íng cong phflng . . . . . . . . . . . 49
3.2 Mºt sŁ to¡n thi håc sinh giäi v thi Olympic . . . . . . 51
T i li»u tham kh£o 59
Tải tài liệu tại sividoc.com
Viết đề tài giá sinh viên – ZALO:0973.287.149-TEAMLUANVAN.COM
ii
Líi c£m ìn
” ho n th nh ÷æc lu“n v«n mºt c¡ch ho n ch¿nh, tæi luæn nh“n
÷æc sü h÷îng d¤n v gióp ï nhi»t t…nh cıa PGS.TS. Nguy„n Vi»t H£i,
Gi£ng vi¶n cao c§p Tr÷íng ⁄i håc H£i PhÆng. Tæi xin ch¥n th nh b y
tä lÆng bi‚t ìn s¥u s›c ‚n thƒy v xin gßi líi tri ¥n nh§t cıa tæi Łi vîi
nhœng i•u thƒy ¢ d nh cho tæi.
Tæi xin ch¥n th nh c£m ìn ban l¢nh ⁄o phÆng o t⁄o sau ⁄i håc, quþ
thƒy cæ gi£ng d⁄y lîp Cao håc K9B2 (2015 - 2017) Tr÷íng ⁄i håc
Khoa håc - ⁄i håc Th¡i Nguy¶n ¢ t“n t…nh truy•n ⁄t nhœng ki‚n thøc
quþ b¡u công nh÷ t⁄o i•u ki»n cho tæi ho n th nh khâa håc.
Tæi xin gßi líi c£m ìn ch¥n th nh nh§t tîi gia …nh, b⁄n b–, nhœng
ng÷íi ¢ luæn ºng vi¶n, hØ træ v t⁄o måi i•u ki»n cho tæi trong suŁt
qu¡ tr…nh håc t“p v thüc hi»n lu“n v«n. Xin tr¥n trång c£m ìn!
H£i PhÆng, th¡ng 9 n«m 2017
Ng÷íi vi‚t Lu“n v«n
Nguy„n Thà Thanh Mai
Tải tài liệu tại sividoc.com
Viết đề tài giá sinh viên – ZALO:0973.287.149-TEAMLUANVAN.COM
1
Mð ƒu
1. Möc ‰ch cıa • t i lu“n v«n
- Nghi¶n cøu c¡c b i to¡n tr¶n l÷îi tåa º nguy¶n: a gi¡c •u tr¶n l÷îi,
÷íng trÆn tr¶n l÷îi, c¡c b i to¡n sŁ håc, h…nh håc tŒ hæp, l÷æng
gi¡c... tr¶n l÷îi v c¡c v§n • li¶n quan.
- Tr…nh b y cì sð khoa håc v v“n döng c¡c ki‚n thøc cıa sŁ håc, ⁄i
sŁ, h…nh håc,...c¡c nguy¶n t›c nh÷ nguy¶n t›c Dirichlet, nguy¶n t›c
cüc h⁄n, nguy¶n t›c b§t bi‚n v o vi»c gi£i c¡c b i to¡n vîi l÷îi nguy¶n.
- C¡c ph÷ìng ph¡p t÷ duy °c tr÷ng v mºt sŁ k‚t qu£ mîi ÷æc tr…nh
b y mºt c¡ch h» thŁng v n¥ng cao qua c¡c b i to¡n hay v khâ trong c¡c
ký thi Olympic QuŁc gia v QuŁc t‚.
- Ng÷íi nghi¶n cøu câ th¶m ki‚n thøc v n«ng lüc bçi d÷ïng håc sinh
giäi v• c¡c v§n • khâ, hay g°p cıa To¡n håc.
2. Nºi dung cıa • t i, nhœng v§n • cƒn gi£i quy‚t
Tr…nh b y h» thŁng mºt sŁ b i to¡n tr¶n l÷îi tåa º nguy¶n düa
tr¶n c¡c b i b¡o [3], [5] v [2]. Ph¡t bi”u v chøng minh c¡c k‚t qu£ li¶n
quan ‚n sŁ håc v h…nh håc. • t i gçm 3 ch÷ìng:
Ch÷ìng 1. L÷îi tåa º v a gi¡c
B i to¡n °t ra l : Düng ÷æc v khæng düng ÷æc a gi¡c •u n o tr¶n
l÷îi tåa º nguy¶n m c¡c ¿nh tròng vîi nót l÷îi? L÷îi tåa º nguy¶n tr¶n
m°t phflng v c¡c t‰nh ch§t, a gi¡c hƒu nºi ti‚p l÷îi nguy¶n, Cæng thøc
Picard v c¡c øng döng cıa l÷îi nguy¶n v o c¡c b i
Tải tài liệu tại sividoc.com
Viết đề tài giá sinh viên – ZALO:0973.287.149-TEAMLUANVAN.COM
2
to¡n h…nh håc, sŁ håc, tŒ hæp, l÷æng gi¡c,... l nhœng v§n • ÷æc nghi¶n
cøu trong ch÷ìng n y. Ch÷ìng 1 câ c¡c möc sau:
1.1. L÷îi tåa º nguy¶n tr¶n m°t phflng v c¡c t‰nh ch§t
1.2. a gi¡c hƒu nºi ti‚p l÷îi nguy¶n
1.3. Cæng thøc Picard
1.4. Mºt sŁ øng döng cıa l÷îi tåa º nguy¶n
Ch÷ìng 2. L÷îi tåa º nguy¶n v ÷íng trÆn
L÷îi tåa º nguy¶n ch‰nh l v nh Gauss n¶n nºi dung ch÷ìng n y
khai th¡c nhœng v§n • v• sŁ håc c¡c sŁ nguy¶n: i”m nguy¶n nguy¶n
tŁ, ÷íng trÆn tr¶n l÷îi tåa º nguy¶n, mºt sŁ øng döng v o sŁ håc v
b i to¡n li¶n quan ‚n bº ba Pythagoras,... Ch÷ìng 2 bao gçm c¡c möc
sau:
2.1. i”m nguy¶n nguy¶n tŁ
2.2. ÷íng trÆn tr¶n l÷îi tåa º nguy¶n
2.3. Mºt sŁ øng döng v o sŁ håc
Ch÷ìng 3. C¡c b i to¡n kh¡c
Li¶n quan ‚n l÷îi tåa º nguy¶n cÆn câ c¡c b i to¡n v• i”m nguy¶n
tr¶n ÷íng cong phflng, C¡c b i to¡n thi håc sinh giäi v thi Olympic.
3.1. i”m nguy¶n tr¶n ÷íng cong phflng
3.1. C¡c b i to¡n thi håc sinh giäi v thi Olympic
T¡c gi£
Tải tài liệu tại sividoc.com
Viết đề tài giá sinh viên – ZALO:0973.287.149-TEAMLUANVAN.COM
3
Ch֓ng 1
L÷îi tåa º v a gi¡c
1.1 L÷îi tåa º nguy¶n tr¶n m°t phflng v c¡c t‰nh ch§t
1.1.1 L÷îi tåa º tr¶n m°t phflng
Tr¶n m°t phflng ta x†t mºt l÷îi t⁄o bði hai hå c¡c ÷íng thflng song
song chia m°t phflng th nh c¡c h…nh b…nh h nh b‹ng nhau.
H…nh 1.1: L÷îi tåa º nguy¶n
T“p hæp t§t c£ c¡c ¿nh c¡c h…nh b…nh h nh gåi l l÷îi tåa º, b£n
th¥n c¡c ¿nh gåi l c¡c nót cıa l÷îi. Måi h…nh b…nh h nh t⁄o bði 2 hå
÷íng thflng song song gåi l h…nh b…nh h nh cì sð cıa ph¥n ho⁄ch
hay h…nh b…nh h nh sinh ra l÷îi.
Chó þ r‹ng mºt l÷îi câ th” nh“n ÷æc tł c¡c hå ÷íng thflng kh¡c nhau:
Tr¶n H…nh 1.1 bi”u di„n mºt l÷îi tåa º nguy¶n, tøc l t“p hæp
Tải tài liệu tại sividoc.com
Viết đề tài giá sinh viên – ZALO:0973.287.149-TEAMLUANVAN.COM
4
c¡c i”m câ tåa º Descartes l c¡c sŁ nguy¶n. L÷îi tåa º nguy¶n ch‰nh
l l÷îi t⁄o bði t§t c£ c¡c ÷íng thflng song song vîi hai tröc tåa º, câ th” coi
l tí gi§y k· æ vuæng væ h⁄n (h…nh b…nh h nh cì sð l h…nh vuæng
c⁄nh 1). L÷îi tåa º nguy¶n câ th” nh“n ÷æc tł c¡c " ÷íng xi¶n" (khi â
h…nh b…nh h nh cì sð l h…nh b…nh h nh ABCD tr¶n H…nh 1.2).
H…nh 1.2: L÷îi tåa º tr¶n m°t phflng
Nh÷ v“y, kh¡i ni»m h…nh b…nh h nh cì sð khæng ch¿ g›n vîi b£n
th¥n l÷îi m cÆn g›n vîi c¡c hå ÷íng thflng sinh ra l÷îi. Ta câ c¡c t‰nh
ch§t ìn gi£n cıa l÷îi nh÷ sau:
H…nh 1.3: l÷îi tåa º nguy¶n vîi hbh cì sð
i. Måi ph†p tành ti‚n song song bi‚n i”m nót n y th nh mºt i”m nót
kh¡c s‡ b£o to n l÷îi (bi‚n l÷îi th nh ch‰nh nâ).
ii. (BŒ • v• ¿nh thø t÷ cıa h…nh b…nh h nh) N‚u 3 ¿nh cıa h…nh
Tải tài liệu tại sividoc.com
Viết đề tài giá sinh viên – ZALO:0973.287.149-TEAMLUANVAN.COM
5
b…nh h nh l c¡c i”m nót cıa l÷îi th… ¿nh thø t÷ công l i”m nót cıa
l֔i.
iii. N‚u qua 2 i”m Q, R cıa l÷îi k· mºt ÷íng thflng th… ÷íng thflng n y
s‡ i qua væ h⁄n c¡c i”m nót cıa l÷îi. Khi â t§t c£ c¡c kho£ng c¡ch
giœa c¡c nót gƒn nhau •u b‹ng nhau.
iv. N‚u mºt h…nh b…nh h nh vîi ¿nh l c¡c i”m nót khæng chøa trong
nâ mºt i”m nót n o cıa l÷îi th… nâ l h…nh b…nh h nh sinh ra l÷îi.
v. (BŒ • v• thæ s«n v thä) Gi£ sß tia ‘ i qua i”m nót A cıa l÷îi n o â.
Khi â t…m ÷æc mºt nót sao cho kho£ng c¡ch tł â ‚n ‘ nhä hìn sŁ
nhä tòy þ cho tr÷îc.
Chøng minh. C¡c t‰nh ch§t i:; ii:; iii:; iv: •u hi”n nhi¶n, ta i chøng
minh bŒ • v: v• thæ s«n v thä: Kþ hi»u giao i”m cıa tia ‘ v ÷íng
thflng nghi¶ng cıa l÷îi l A0 = A; A1; A2; :::; An; :::. Nh§c t§t c£ c¡c
h…nh b…nh h nh m câ c¡c i”m n y n‹m tr¶n ÷íng thflng chøa c⁄nh v•
h…nh b…nh h nh ABCD, khi â mØi i”m An chuy”n th nh i”m An tr¶n
c⁄nh AB.
Vîi måi > 0 cho tr÷îc t…m ÷æc c¡c i”m Am v Am+k nh÷ th‚ sao cho
kho£ng c¡ch giœa chóng nhä hìn . B¥y gií h¢y chøng minh kho£ng
c¡ch tł i”m Ak ‚n mºt trong c¡c nót cıa l÷îi nhä hìn ( °c bi»t n‚u A0
m
tròng vîi A0
m+k th… Ak s‡ l nót cıa l÷îi).
K‚t qu£ sau l hi”n nhi¶n: Gi£ sß a; b l hai sŁ thüc b§t ký. Chøng minh
r‹ng t“p hæp t§t c£ c¡c i”m vîi tåa º (ka; lb) vîi k; l 2 Z l mºt l÷îi.
M»nh • 1.1. Vîi måi sŁ væ t > 0 v måi sŁ d÷ìng •u t…m ÷æc c¡c sŁ tü
nhi¶n m; n sao cho jm nj < :
Chøng minh. L§y h» tåa º Descartes tr¶n m°t phflng v k· ÷íng thflng
y = x. V… l sŁ væ t n¶n tr¶n ÷íng thflng n y câ duy nh§t mºt i”m mang
tåa º nguy¶n l i”m gŁc tåa º. ÷íng thflng y = x c›t ÷íng x = m t⁄i c¡c i”m
(m; m ). Theo t‰nh ch§t v, t…m ÷æc nót (m; n) cıa l÷îi nguy¶n sao
cho kho£ng c¡ch (theo chi•u thflng øng) tł â ‚n i”m (m; m ) nhä hìn . â l
i•u cƒn chøng minh.
Tải tài liệu tại sividoc.com
Viết đề tài giá sinh viên – ZALO:0973.287.149-TEAMLUANVAN.COM
6
D„ th§y r‹ng trong t§t c£ c¡c kh£ n«ng th… kho£ng c¡c æi mºt
giœa c¡c nót cıa l÷îi b§t ký s‡ l sŁ nhä nh§t. T‰nh ch§t n y còng vîi
t‰nh ch§t ii: cıa l÷îi câ th” thay cho ành ngh¾a v• l÷îi.
M»nh • 1.2. Gi£ sß t“p hæp M tr¶n m°t phflng câ c¡c t‰nh ch§t sau:
(1) Kho£ng c¡ch cıa hai i”m b§t ký khæng nhä hìn sŁ d÷ìng d n o â;
(2) N‚u 3 i”m A,B,C cıa M l c¡c ¿nh cıa h…nh b…nh h nh ABCD th…
¿nh thø t÷ D cıa h…nh b…nh h nh n y công thuºc t“p hæp M.
Khi â M l mºt l÷îi.
Chøng minh.
H…nh 1.4: H…nh v‡ a. H…nh v‡ b
L§y i”m B tòy þ thuºc M. Gi£ sß A l i”m trong M, gƒn B nh§t
(H…nh v‡ a), i”m n y tçn t⁄i v… t§t c£ c¡c kho£ng c¡ch æi mºt giœa
c¡c i”m cıa M •u lîn hìn d).
Qua A v B k· mºt ÷íng thflng. Trong c¡c i”m cıa M khæng n‹m tr¶n
÷íng thflng n y chån ÷æc i”m gƒn B nh§t, gåi i”m â l C. Ta düng
h…nh b…nh h nh ABCD. Theo i•u ki»n (2) i”m D công thuºc t“p hæp
M. Ta düng l÷îi sinh bði h…nh b…nh h nh ABCD, ta i chøng minh t“p
hæp M tròng vîi l÷îi vła düng.
Tải tài liệu tại sividoc.com
Viết đề tài giá sinh viên – ZALO:0973.287.149-TEAMLUANVAN.COM
7
Tł i•u ki»n (2) suy ra r‹ng t§t c£ c¡c nót cıa l÷îi n y •u thuºc M. Bði
v“y ch¿ cƒn ki”m tra r‹ng ð bi¶n công nh÷ ð trong h…nh b…nh h nh
ABCD khæng câ i”m n o cıa M kh¡c vîi ¿nh cıa nâ. i•u n y gƒn nh÷
hi”n nhi¶n: N‚u i”m S n‹m ð trong h…nh b…nh h nh (H…nh v‡
[[[[
b.)th… ‰t nh§t mºt trong c¡c gâc ASB; BSC; CSD; ASD l gâc tò
ho°c gâc vuæng v bði v“y kho£ng c¡ch tł S tîi mºt trong c¡c ¿nh cıa
nâ nhä hìn c⁄nh n o â cıa nâ(n‚u S n‹m tr¶n bi¶n th… công v“y).
Chflng h⁄n gi£ sß â l kho£ng c¡ch SC. Ta düng h…nh b…nh h nh
BCSS0
; (S0
2 M). Khi â BS0
nhä hìn BC ho°c BA nh÷ng i•u â m¥u
thu¤n vîi c¡ch chån ho°c cıa i”m C ho°c cıa i”m A.
C¡c tr÷íng hæp cÆn l⁄i t÷ìng tü.
B i to¡n 1.1. Gi£ sß l sŁ væ t tòy þ. Chøng minh r‹ng: vîi måi >
0 v måi sŁ thüc luæn t…m ÷æc c¡c sŁ m; n sao cho: jm +n j < .
B i to¡n 1.2. Chøng minh r‹ng kþ hi»u th“p ph¥n cıa sŁ 2n
câ th” b›t
ƒu b‹ng chœ sŁ b§t ký cıa tŒ hæp c¡c chœ sŁ cho tr÷îc.
1.1.2 L÷îi tåa º nguy¶n v a gi¡c •u
L§y mºt tí gi§y k· æ vuæng. Hi”n nhi¶n h…nh vuæng vîi c¡c ¿nh l
c¡c nót cıa l÷îi luæn düng ÷æc v b‹ng væ sŁ c¡ch. C¡ch l m nh÷ th‚
câ thüc hi»n ÷æc Łi vîi tam gi¡c •u hay löc gi¡c •u hay khæng? Trong
phƒn n y ta s‡ t…m c¥u tr£ líi th‰ch ¡ng.
Gi£ sß ta câ mºt l÷îi nguy¶n hay cÆn gåi l l÷îi Z2
. Tr¶n l÷îi nguy¶n
n y h¢y x†t b i to¡n v• sü tçn t⁄i a gi¡c •u nºi ti‚p ÷æc trong l÷îi.
ành ngh¾a 1.1. Ta nâi a gi¡c nºi ti‚p ÷æc trong l÷îi n‚u tçn t⁄i mºt a
gi¡c çng d⁄ng vîi nâ m c¡c ¿nh l nót cıa l÷îi.
B i to¡n nºi ti‚p mºt tam gi¡c •u tr¶n l÷îi nguy¶n l b i to¡n ìn
gi£n nh§t trong c¡c b i to¡n nºi ti‚p mºt a gi¡c •u tr¶n l÷îi nguy¶n. V“y
m k‚t qu£ "khæng düng ÷æc tam gi¡c •u nºi ti‚p tr¶n l÷îi Z2
" ÷æc ÷a
ra v o n«m 1878 do nh to¡n håc E. Lukacy cæng bŁ. — ¥y chóng tæi
÷a ra 5 c¡ch chøng minh k‚t qu£ n y: c¡ch thø nh§t (cıa Lukacy) sß
döng lþ thuy‚t chia h‚t c¡c sŁ nguy¶n; c¡ch thø hai düa tr¶n
Tải tài liệu tại sividoc.com
tài giá sinh viên – ZALO:0973.287.149-TEAMLUANVAN.COM
8
2
t‰nh væ t cıa sŁ tan n , c¡ch thø ba dòng l÷æng gi¡c, c¡ch thø t÷
dòng ph÷ìng ph¡p di»n t‰ch, c¡ch thø n«m dòng ph÷ìng ph¡p cüc
h⁄n. B i to¡n n y công ¢ l mºt trong nhœng b i to¡n hay trong ký thi
håc sinh giäi quŁc gia v quŁc t‚ trong nhœng n«m gƒn ¥y.
M»nh • 1.3. Tam gi¡c •u ABC khæng th” nºi ti‚p ÷æc trong l÷îi
nguy¶n.
Chøng minh.
C¡ch 1. (Ph÷ìng ph¡p chia h‚t, cüc h⁄n) Gi£ sß düng ÷æc tam gi¡c
•u tr¶n l÷îi Z2
v ta chån 1 tam gi¡c c⁄nh nhä nh§t, mºt ¿nh l gŁc tåa º,
hai ¿nh kia câ tåa º (a; b); (c; d). Khi â bŁn sŁ nguy¶n a; b; c; d
khæng câ ÷îc chung kh¡c 1 (tøc chóng nguy¶n tŁ còng nhau). Trong
tr÷íng hæp ng÷æc l⁄i s‡ d¤n tîi tam gi¡c vîi tåa º c¡c ¿nh l
Tải tài liệu tại sividoc.com
tài giá sinh viên – ZALO:0973.287.149-TEAMLUANVAN.COM
(0; 0); a ;b ; c ;d vîi k l ÷îc chung cıa bŁn sŁ a; b; c; d. Ta vi‚t
k k k k
flng thøc c¡c c⁄nh tam gi¡c vuæng ð d⁄ng tåa º:
a2
+ b2
= c2
+ d2
= (a c)2
+ (b d)2
:
Suy ra: a2
+b2
= c2
+d2
= 2(ac+bd). Do â, a2
+b2
+c2
+d2
= 4(ac+bd).
Ngh¾a l tŒng b…nh ph÷ìng bŁn sŁ nguy¶n chia h‚t cho 4.
M°t kh¡c b…nh ph÷ìng cıa sŁ nguy¶n khi chia cho 4 ch¿ cho d÷ l
0 ho°c 1. Bði v“y, ho°c t§t c£ 4 sŁ a; b; c; d l chfin ho°c t§t c£ •u l·.
Kh£ n«ng thø nh§t khæng x£y ra v… c¡c sŁ n y theo c¡ch chån ð
tr¶n l nguy¶n tŁ còng nhau. Kh£ n«ng thø hai công khæng x£y ra
v… lóc â khæng th” câ a2
+ b2
= (a c)2
+ (b d)2
. M¥u thu¤n n y suy
ra i•u ph£i chøng minh.
C¡ch 2. (Ph÷ìng ph¡p l÷æng gi¡c) Chó þ r‹ng n‚u 2 tia gŁc O k·
qua c¡c nót (a; b) v (c; d) cıa l÷îi th… tang cıa gâc giœa hai tia n y
l sŁ hœu t ho°c khæng x¡c ành v…
d b
tantan =ad bc
tan = tan( ) = = c a :
1 + tan : tan db ac + bd
1 + ca
Tải tài liệu tại sividoc.com
Viết đề tài giá sinh viên – ZALO:0973.287.149-TEAMLUANVAN.COM
9
÷æc tam gi¡c •u tr¶n l÷îi Z2
th… hai tia vîi gŁc l p
mºt trong c¡c ¿nh tam gi¡c s‡ t⁄o th nh gâc 60 m tan 60 = 3 l sŁ væ t
, tr¡i vîi v‚ ph£i l sŁ hœu t .
C¡ch 3. (Ph÷ìng ph¡p di»n t‰ch) Ta t‰nh di»n t‰ch tam gi¡c ABC theo
AB2
p
AB:AC 3
2 c¡ch: Mºt m°t, S ABC = sin A = sin 60 = AB2
:
2 2 4
Nh÷ng AB2
= sŁ nguy¶n (AB2
= AC2
+ CB2
), ngh¾a l S ABC l sŁ væ
t . M°t kh¡c, ngo⁄i ti‚p xung quanh tam gi¡c ABC l h…nh chœ nh“t
CB’C’A’ ta nh“n ÷æc: S ABC = SCB0CA0
S
AB0
C
S
AC0
B
S
A0
BC0
l sŁ hœu t v… t§t c£ c¡c di»n t‰ch ð v‚ ph£i •u l c¡c sŁ hœu t . Ta
nh“n ÷æc S ABC vła l sŁ hœu t vła l sŁ væ t . Væ lþ.
C¡ch 4. (Ph÷ìng ph¡p tåa º) Gi£ sß A(a1; a2), B(b1; b2), C(c1; c2) l
1 1 1 l mºt sŁ nguy¶n.
c¡c i”m nguy¶n. Khi â di»n t‰ch 2S = a1 b1
c1
a2 b2 c2
2p
AB 3
M°t kh¡c do ABC l tam gi¡c •u n¶n 2S = l sŁ væ t . M¥u
2
thu¤n â chøng minh b i to¡n.
Tł ¥y ta s‡ vi‚t k gi¡c thay cho a gi¡c k c⁄nh, c¡c c¡ch nâi sau ¥y
÷æc sß döng th÷íng xuy¶n: 5-gi¡c hay ngô gi¡c; 8-gi¡c hay b¡t gi¡c;
10-gi¡c hay th“p gi¡c; 12-gi¡c hay th“p nhà gi¡c,. . .
M»nh • 1.4. a gi¡c •u duy nh§t nºi ti‚p ÷æc trong l÷îi nguy¶n l h…nh
vuæng.
Chøng minh. D„ th§y tçn t⁄i h…nh vuæng nºi ti‚p l÷îi, chflng h⁄n â l
h…nh vuæng cì sð, câ tåa º c¡c ¿nh (0; 0); (0; 1); (1; 0); (1; 1).
Ti‚p theo ta chøng minh b‹ng ph£n chøng r‹ng ngo i h…nh vuæng
khæng cÆn a gi¡c •u n o kh¡c nºi ti‚p ÷æc trong l÷îi nguy¶n. Gi£ sß
D l a gi¡c nºi ti‚p ÷æc cıa l÷îi nguy¶n (tøc c¡c ¿nh •u l nót). X£y ra c¡c
kh£ n«ng sau:
a. D l tam gi¡c •u ABC.
b. D l ngô gi¡c •u ABCDE:
Tải tài liệu tại sividoc.com
Viết đề tài giá sinh viên – ZALO:0973.287.149-TEAMLUANVAN.COM
10
c. D l löc gi¡c •u ABCDEF:
d. D l n-gi¡c •u vîi n 7:
Ta x†t tłng kh£ n«ng:
a. Xem M»nh • 1.3
b. Gi£ sß ngô gi¡c •u ABCDE câ c⁄nh b‹ng a. Gåi A0
= AC  BE
th… A0
l nót cıa l÷îi. Th“t v“y, v… AA0
DE l h…nh b…nh h nh n¶n
AA0
=! 0
công l
! ED. Tł ¥y do E, D, A l c¡c i”m nguy¶n n¶n A
i”m nguy¶n. Kþ hi»u B0
; C0
; D0
; E0
t÷ìng tü nh÷ A0
th… B0
; C0
;
D0
; E0
công l c¡c i”m nguy¶n. V“y ngô gi¡c A0
B0
C0
D0
E0
l ngô gi¡c
nºi ti‚p ÷æc. Ta câ A’B’=C’D’=D’E’=E’A’. M°t kh¡c, ABE0
= BA0
C suy
000
000
000
000
000
ra A E D = B A E . T÷ìng tü, ta câ: A E D = B A E = C B A =
D

CB
0
= C

DE . Tł â suy ra A
0
B
0
C
0
D
0
E
0
l ngô gi¡c •u (nºi ti‚p
0 0 0 0 0
÷æc tr¶n l÷îi) câ c⁄nh b < a, (H…nh 1.5).
H…nh 1.5: Ngô gi¡c v löc gi¡c
Ti‚p töc qu¡ tr…nh tr¶n ta s‡ thu ÷æc væ sŁ i”m nguy¶n b¶n trong
ngô gi¡c ABCDE. i•u n y væ lþ v… trong mºt mi•n giîi h⁄n ch¿ tçn t⁄i
mºt sŁ hœu h⁄n i”m nguy¶n. V“y khæng tçn t⁄i ngô gi¡c •u nºi ti‚p ÷æc
l÷îi nguy¶n.
c. N‚u D l löc gi¡c •u A1B1C1D1E1F1. Trong löc gi¡c •u tçn t⁄i tam
gi¡c •u câ ¿nh nguy¶n. Væ lþ
d. N‚u D l n-gi¡c •u vîi n 7: Gi£ sß A1A2 : : : An l a gi¡c •u
Tải tài liệu tại sividoc.com
Viết đề tài giá sinh viên – ZALO:0973.287.149-TEAMLUANVAN.COM
11
nguy¶n ( a gi¡c •u câ ¿nh l c¡c i”m nguy¶n hay c¡c nót l÷îi) câ
c⁄nh nhä nh§t l a. Gåi O l
!
OB1
!
1 2
!
2
=
gŁc tåa º. °t = A A ;OB
!
A2A3
!
OBn
! n
A
1 1
B
2
:::B
n
l a gi¡c •u theo c¡ch
;:::; = A . Ta suy ra: B
düng v l a gi¡c nguy¶n. Khi â B1B2 = B2B3 = : : : = BnB1 = b.
 b
Trong tam gi¡c OB1B2, gåi B1OB2 = th… sin 2 = 2a . M°t kh¡c,
=
2
n ; n 7 n¶n < 60 =) sin 2 <
1
2 =) 2
b
a <
1
2 =) b < a.
M¥u thu¤n vîi gi£ thi‚t A1A2 : : : An l a gi¡c nguy¶n câ c⁄nh nhä nh§t.
M»nh • ÷æc chøng minh ho n to n.
Nh“n x†t 1.1.1. C¡ch chøng minh nh÷ tr¶n ÷æc gåi l ¡p döng nguy¶n
t›c cüc h⁄n. GiŁng nh÷ nguy¶n t›c Dirichle, c£ hai •u l ph÷ìng ph¡p
chøng minh ph£n chøng. Nºi dung cıa ph÷ìng ph¡p n y l ¡p döng
nguy¶n t›c sau: "Trong mºt t“p hæp hœu h⁄n (kh¡c rØng) c¡c sŁ thüc
tçn t⁄i sŁ nhä nh§t v tçn t⁄i sŁ lîn nh§t". Nhí nguy¶n lþ cüc h⁄n n y ta
câ th” x†t c¡c phƒn tß m mºt ⁄i l÷æng n o â câ gi¡ trà nhä nh§t ho°c
gi¡ trà lîn nh§t. Chflng h⁄n:
X†t o⁄n thflng câ º d i lîn nh§t (nhä nh§t) trong mºt sŁ hœu h⁄n
c¡c o⁄n thflng.
X†t gâc câ sŁ o lîn nh§t (nhä nh§t) trong mºt sŁ hœu h⁄n c¡c gâc.
X†t a gi¡c câ di»n t‰ch ho°c chu vi lîn nh§t (nhä nh§t) trong mºt
sŁ hœu h⁄n c¡c a gi¡c.
X†t kho£ng c¡ch lîn nh§t (nhä nh§t) trong mºt sŁ hœu h⁄n c¡c
kho£ng c¡ch.
Nh÷ v“y sŁ c¡c a gi¡c •u nºi ti‚p ÷æc tr¶n l÷îi nguy¶n r§t hi‚m hoi. Câ
hai c¡ch mð rºng t‰nh nºi ti‚p tr¶n l÷îi nguy¶n cıa a gi¡c nh÷ th‚: Mºt l
ta x†t c¡c a gi¡c nßa •u ( •u c⁄nh ho°c •u gâc), hai l ta giœ nguy¶n c¡c
a gi¡c •u nh÷ng t…m c¡ch "nºi ti‚p x§p x¿", hay cÆn gåi l "hƒu nºi
ti‚p". Phƒn ti‚p theo s‡ i theo hai h÷îng nh÷ v“y.
Tải tài liệu tại sividoc.com
Viết đề tài giá sinh viên – ZALO:0973.287.149-TEAMLUANVAN.COM
12
1.1.3 a gi¡c nßa •u nºi ti‚p l÷îi nguy¶n
Kh¡i ni»m a gi¡c nßa •u ÷æc chia l m hai lo⁄i: •u theo c⁄nh v •u
theo gâc (xem [4]).
ành ngh¾a 1.2. Mºt a gi¡c gåi l nßa •u n‚u nâ câ c¡c gâc b‹ng nhau
ho°c c¡c c⁄nh b‹ng nhau. a gi¡c câ c¡c gâc b‹ng nhau, c¡c c⁄nh câ th”
khæng b‹ng nhau, ÷æc gåi l a gi¡c nßa •u-gâc, kþ hi»u t“p hæp â l
G; a gi¡c câ c¡c c⁄nh b‹ng nhau, c¡c gâc câ th” khæng b‹ng nhau,
÷æc gåi l a gi¡c nßa •u-c⁄nh, kþ hi»u t“p hæp â l C.
D„ th§y giao GC l t“p hæp c¡c a gi¡c •u. Chó þ r‹ng trong ành
ngh¾a tr¶n ta khæng nâi g… ‚n a gi¡c lçi hay khæng. C¡c a gi¡c nßa
•u-gâc nºi ti‚p tr¶n l÷îi l h…nh vuæng v h…nh b¡t gi¡c.
H…nh 1.6: H…nh vuæng v h…nh b¡t gi¡c
a gi¡c giîi h⁄n bði ÷íng g§p khóc kh†p k‰n vîi º d i c¡c c⁄nh b‹ng 1
v gâc giœa hai o⁄n b‹ng 1v l tr÷íng hæp °c bi»t cıa a gi¡c nßa •u c⁄nh.
Ta x†t mºt sŁ d⁄ng.
a. Tam gi¡c •u.
B i to¡n nºi ti‚p mºt tam gi¡c •u tr¶n l÷îi nguy¶n l b i to¡n ìn
gi£n nh§t trong c¡c b i to¡n nºi ti‚p a gi¡c •u. V“y m k‚t qu£ l khæng
düng ÷æc tam gi¡c •u nºi ti‚p tr¶n l÷îi Z2
, ÷æc cæng bŁ ngay tł n«m
1878 bði nh to¡n håc E. Lukacy.
b. a gi¡c •u-c⁄nh.
M»nh • 1.5. ( ành lþ Boll 1, [4])
Tải tài liệu tại sividoc.com
Viết đề tài giá sinh viên – ZALO:0973.287.149-TEAMLUANVAN.COM
13
i. Tr¶n l÷îi nguy¶n Z2
khæng th” düng ÷æc a gi¡c •u-c⁄nh vîi sŁ c⁄nh
l sŁ l·.
ii. Tr¶n l÷îi nguy¶n Z2
câ th” düng ÷æc a gi¡c •u-c⁄nh vîi sŁ c⁄nh l sŁ
chfin.
Chøng minh. i. Ph£n chøng. Gi£ sß câ düng ÷æc a gi¡c •u-c⁄nh vîi sŁ
c⁄nh l·. Ta s‡ coi a gi¡c n y câ º d i c⁄nh lîn nh§t. Gi£ sß
! ~ ~
vk = xki + ykj; (k = 1; 2; : : : ; n) l c¡c v†c tì câ h÷îng dåc theo c¡c
~
câ º d i cıa chóng kh¡c
c⁄nh a gi¡c. TŒng c¡c v†c tì n y b‹ng 0 v
nhau. Ngh¾a l
x1 + x2 + + xn = 0
y1 + y2 + + yn = 0
x2
1 + y1
2
= x2
2 + y2
2
= = x2
n + yn
2
= a2
:
trong â, a l º d i c⁄nh a gi¡c ¢ cho. B…nh ph÷ìng hai v‚ 2 flng thøc ƒu,
cºng l⁄i ta thu ÷æc
n:a2
=
X
2 (xixj + yiyj):
i6=j
Tł ¥y, v… a2
l sŁ tü nhi¶n v n l· n¶n a2
l sŁ chfin.
.
N‚u a
2 .
4 th… khi â t§t c£ c¡c xi; yi l c¡c sŁ chfin v… tŒng æi mºt
.
c¡c b…nh ph÷ìng chia h‚t cho 4. i•u â l khæng th” v… trong c¡c v†c
1
tì 2v~i, tåa º nguy¶n, câ th” t⁄o th nh a gi¡c •u-c⁄nh gçm n c⁄nh.
a gi¡c n y câ º d i c⁄nh b‹ng mºt nßa º d i c⁄nh a gi¡c ¢ chån.
i•u n y væ lþ theo nguy¶n t›c cüc h⁄n.
B¥y gií gi£ sß a2 ..
. 2 v a2
khæng chia h‚t cho 4. Lóc n y c¡c xi; yi l
c¡c sŁ l· v… chóng thäa m¢n ph÷ìng tr…nh xi
2
+ yi
2
= a
2
. Nh÷ v“y tŒng
P
.
2 .
(xixj + yiyj) l sŁ chfin v bði v“y a . 4. Væ lþ.
i6=l
ii. H…nh 1.7 ch¿ ra h…nh vuæng gh†p vîi löc gi¡c, ta nh“n ÷æc
mºt th“p gi¡c (sŁ c⁄nh l 10) câ c¡c c⁄nh b‹ng nhau, khæng lçi. Tuy
nhi¶n câ th” chøng minh ÷æc r‹ng tr¶n l÷îi nguy¶n düng ÷æc mºt a
gi¡c •u-c⁄nh lçi vîi sŁ c⁄nh l sŁ chfin.
Tải tài liệu tại sividoc.com
Viết đề tài giá sinh viên – ZALO:0973.287.149-TEAMLUANVAN.COM
14
H…nh 1.7: a gi¡c •u-c⁄nh
c. a gi¡c •u gâc.
” nh“n ÷æc k‚t qu£ v• a gi¡c •u-gâc ta câ nh“n x†t sau: C¡c sŁ tan
2
n vîi n 2 N l sŁ væ t trł c¡c sŁ øng vîi n = 1; 2; 4; 8.
” câ k‚t qu£ â ta ¡p döng cæng thøc
tann
= Cn
1
tanCn
3
tan3
+ Cn
5
tan5
+ + ( 1)
n 1
tann
2
n 1
1 Cn
2
tan2
+ Cn
4
tan4
+ ( 1) tann
2
(1.1)
trong â, Ck
l h» sŁ trong khai tri”n nhà thøc Newton:
n
Ph†p chøng minh cæng thøc (1.1) nh“n ÷æc nhí quy n⁄p to¡n håc.
Ta s‡ coi n 5 v n = 8 v… c¡c tr÷íng hæp n = 1; 2; 3; 4 l hi”n nhi¶n.
Gi£ sß = 2 . Khi â tan n = 0 v theo (1.1) sŁ x = tan 2 l
n n
nghi»m cıa ph÷ìng tr…nh vîi h» sŁ nguy¶n
n Cn
3
x2
+ Cn
5
x4
+ ( 1)
n 1
xn 1
= 0 (1.2)
2
Trong ph÷ìng tr…nh n y h» sŁ cao nh§t l 1, do â, nghi»m hœu t cıa
nâ ph£i l sŁ nguy¶n. Bði v“y n‚u x l nghi»m hœu t th… x ph£i l sŁ
nguy¶n. X†t 2 tr÷íng hæp:
Tr÷íng hæp 1. N‚u n = mp vîi p l sŁ nguy¶n tŁ l·. Câ hai kh£ n«ng:
2
N‚u m = 1 tøc n = p th… tł (1.2) rót ra x2
jp v do â, x = tan n = 1.
Tải tài liệu tại sividoc.com
Viết đề tài giá sinh viên – ZALO:0973.287.149-TEAMLUANVAN.COM
15
Gi£ sß tan 2 l sŁ hœu t th… nh÷ ¢ th§y trong cæng thøc (1.1), sŁ
2 m
n
2
tan = tan công l sŁ hœu t . Væ lþ.
n n
2 p
k
4. V… tan
Tr÷íng hæp 2. B¥y gií, gi£ sß n = 2 ; k = 2 1
6
l sŁ væ t n¶n theo cæng thøc (1.1), t§t c£ c¡c sŁ d⁄ng tan
2
; k 4
2k
công l sŁ væ t . Khflng ành ÷æc chøng minh.
Chó þ r‹ng, hai tr÷íng hæp n y l t§t c£ c¡c kh£ n«ng câ th” x£y ra.
M»nh • 1.6. ( ành lþ Boll 2, [4]) Trong t§t c£ c¡c a gi¡c •u-gâc ch¿
düng ÷æc h…nh chœ nh“t v h…nh b¡t gi¡c nºi ti‚p tr¶n l÷îi nguy¶n
Chøng minh. Ta s‡ coi n 4 v… tr÷íng hæp tam gi¡c ¢ ÷æc chøng
minh. H…nh vuæng v b¡t gi¡c ¢ düng ÷æc nh÷ tr¶n h…nh v‡. B¥y gií
gi£ sß a gi¡c •u-gâc vîi n > 4 ¿nh l düng ÷æc tr¶n l÷îi nguy¶n Z2
. Khi
â, c¡c v†c tì bi”u di„n c¡c c⁄nh cıa nâ câ tåa º nguy¶n v gâc
2
giœa 2 v†c tì cıa hai c⁄nh k• nhau b§t ký •u b‹ng n . Ta s‡ coi O l
gŁc tåa º. Khi â, gâc giœa [OP) v [OQ) b‹ng
2
v tan 2
nh÷ ¢
n n
th§y ph£i l sŁ hœu t khi n > 4. Nh÷ng i•u n y x£y ra ch¿ khi n = 8,
xem h…nh v‡ 1.8.
H…nh 1.8: a gi¡c •u-gâc
Tải tài liệu tại sividoc.com
Viết đề tài giá sinh viên – ZALO:0973.287.149-TEAMLUANVAN.COM
16
C¡c b i to¡n sau ¥y bŒ sung th¶m cho kh¡i ni»m a gi¡c •u, nßa •u
tr¶n l÷îi nguy¶n.
B i to¡n 1.3. Tr¶n gi§y æ vuæng v‡ mºt ÷íng g§p khóc kh†p k‰n câ
c¡c ¿nh l c¡c nót. Chøng minh r‹ng sŁ c¡c o⁄n g§p khóc l chfin.
B i to¡n 1.4. Chøng minh r‹ng tr¶n l÷îi Z2
câ th” düng ÷æc a gi¡c lçi
•u c⁄nh vîi sŁ c⁄nh l sŁ chfin tòy þ.
B i to¡n 1.5. Chøng minh r‹ng a gi¡c •u duy nh§t tr¶n m°t phflng m
t§t c£ c¡c ¿nh câ tåa º hœu t l h…nh vuæng.
B i to¡n 1.6. Chøng minh flng thøc (1.1) nhí ¡p döng cæng thøc
Moivre (Moa-vì-rì):
cos n + i: sin n = (cos + i: sin )n
:
B i to¡n 1.7. a. Chøng minh r‹ng n‚u p v q l c¡c sŁ tü nhi¶n nguy¶n
tŁ còng nhau, cos(p =q) l sŁ hœu t th… sŁ cos( =q) công l sŁ hœu t .
b.Chøng minh r‹ng n‚u sŁ cos(p =q) vîi p,q nguy¶n tŁ còng nhau v
q > 3 l sŁ hœu t khi v ch¿ khi sŁ cos( =q) l sŁ hœu t .
c. Gi£ sß p,q nguy¶n tŁ còng nhau, q 3. Chøng minh r‹ng c¡c sŁ
sin(p =q) l sŁ væ t vîi q 6= 6
Ph÷ìng ph¡p gi£i: Sß döng k‚t qu£: 2 cos n câ th” vi‚t ÷æc d÷îi d⁄ng
fn(2 cos ), trong â, fn(x) l a thøc vîi h» sŁ nguy¶n v h» sŁ cao nh§t b‹ng
1.
1.2 a gi¡c hƒu nºi ti‚p l÷îi nguy¶n
Ta x†t b i to¡n thø hai: a gi¡c hƒu nºi ti‚p tr¶n l÷îi nguy¶n.
ành ngh¾a 1.3. a gi¡c gåi l nºi ti‚p trong l÷îi M n‚u tçn t⁄i mºt a gi¡c
çng d⁄ng vîi nâ (vîi h» sŁ çng d⁄ng l sŁ tü nhi¶n) sao cho kho£ng c¡ch
tł mØi ¿nh ‚n nót nhä hìn .
Chó þ r‹ng, khi = 0 ta nh“n ÷æc ành ngh¾a a gi¡c nºi ti‚p l÷îi ð
tr¶n, cÆn khi qu¡ lîn (lîn hìn kho£ng c¡ch giœ hai nót) th… ành
ngh¾a khæng x¡c ành.
Tải tài liệu tại sividoc.com
Viết đề tài giá sinh viên – ZALO:0973.287.149-TEAMLUANVAN.COM
17
Hìn nœa, ta s‡ gåi a gi¡c hƒu kh›p nºi ti‚p l÷îi M n‚u nâ nºi ti‚p trong
l÷îi vîi måi > 0. Rª r ng, a gi¡c s‡ hƒu kh›p nºi ti‚p n‚u nâ nºi ti‚p ÷æc
trong l÷îi. i•u ng÷æc l⁄i khæng óng:
M»nh• 1.7. Tam gi¡c •u hƒu kh›p nºi ti‚p trong l÷îi nguy¶n.
H…nh 1.9: Tam gi¡c •u hƒu nºi ti‚p l÷îi
Chøng minh. X†t tam gi¡c •u c⁄nh 2 nh÷ trong H…nh 1.9. Vîi ph†p và tü
t¥m O, t sŁ và tü k tü nhi¶n (kh¡c 0), A1B1C1 bi‚n th nh AkBkCk,
trong â, Ak; Bk l c¡c nót cıa l÷îi cÆn Ck câ t⁄o º (0; k
p
3) (tung
p ). V… Ak; Bk l
º yk cıa Ck b‹ng yk = ky 1 = k:OC1 = k 3 c¡c nót
vîi måi k n¶n b i to¡n quy v• t…m k ” kho£ng c¡ch tł Ck ‚n nót gƒn
nh§t nhä hìn .
L§y sŁ tü nhi¶n N sao cho N > 1 v chia kho£ng [0,1) th nh N nßa
kho£ng câ º d i nhä hìn :
N ;:::; ; 1 :
0; N ; N ; N
1 1 2 N 1
B¥y gií chó þ r‹ng tr¶n o⁄n [0; 1] câ c¡c i”m fk
p
3g, trong â, fxg
p
kþ hi»u phƒn ph¥n cıa x; k = 1; 2; : : : ; N + 1. MØi i”m fk 3g rìi v o
mºt trong c¡c nßa kho£ng. V… câ N nßa kho£ng m N + 1 i”m n¶n
câ hai i”m n o â rìi v o còng mºt nßa kho£ng. Gi£ sß â l i”m
p p p
fm 3g; fn 3g vîi m > n l c¡c sŁ tü nhi¶n. V… (m n) 3 kh¡c sŁ
Tải tài liệu tại sividoc.com
Viết đề tài giá sinh viên – ZALO:0973.287.149-TEAMLUANVAN.COM
18
1
nguy¶n nhä hìn N v do â nhä hìn . B‹ng c¡ch °t k b‹ng m n ta nh“n
÷æc i•u cƒn t…m.
M»nh • ÷æc chøng minh. Tuy nhi¶n x£y ra mºt sü ki»n tŒng qu¡t:
M»nh • 1.8. ( ành lþ cì b£n) Måi a gi¡c •u câ th” hƒu kh›p nºi ti‚p trong
mºt l÷îi b§t ký.
Chøng minh. Gi£ sß cho l÷îi M, sŁ > 0 v a gi¡c . Ta chøng minh r‹ng
qua mºt ph†p và tü n o â vîi t sŁ và tü l sŁ tü nhi¶n N, a gi¡c
bi‚n th nh a gi¡c 0
sao cho måi ¿nh cıa a gi¡c 0
•u c¡ch i”m nót mºt
kho£ng nhä hìn .
X†t ¿nh A cıa a gi¡c. Gi£ sß qua ph†p và tü t¥m O, h» sŁ và tü k
i”m A bi‚n th nh i”m v OA = x ~a+y ~ ~
Ak A
b, trong â, ~a; b l c¡c v†c
! A
!
~ ~
tì sinh ra l÷îi. Khi â, OAk = k(xA~a + yAb) = (kxA)~a + (kyA)b. i•u ki»n
Ak c¡ch i”m nót cıa l÷îi kho£ng c¡ch nhä hìn s‡ ÷æc thüc hi»n n‚u
mØi tåa º (kxA) v (kyA) s‡ kh¡c sŁ nguy¶n nhä hìn c , vîi h» sŁ
c khæng phö thuºc v o k, câ th” l§y chflng h⁄n c = 1 .
~
! ~ 2 max(j~aj; jbj)
j j j Aj jj 2
Th“t v“y, gi£ sß OF = m~a + nb. Khi â, n‚u m ~a kx ~a <
~ ~ !
OA ~
j j j
kx Aj jj
< 2 k
F j k
F j j
OF kj j
m~a nb
v m b b th… A A
= = +
=
!
!
~
k(xA~a yAb)j < . ~ ~
Nh÷ng c¡c b§t flng thøc jmj~aj kxAj~ajj < 2 v jnjbj kyAjbjj <2
t÷ìng ÷ìng vîi c¡c b§t flng thøc jm kxAj < v jn kyAj < .
2 ~a ~
j j 2 b
Bði v“y n‚u kxA v kyA kh¡c sŁ nguy¶n nhä hìn
j j
th…
~
AkF < vîi i”m nót F n o â v
2 max(j~aj; jbj)
ta câ i•u Æi häi. Ta cÆn ph£i
khflng ành t§t c£ c¡c ¿nh cıa a gi¡c 0
•u ð trong l¥n c“n cıa
i”m nót cıa l÷îi M.
Khflng ành â câ n‚u t§t c£ c¡c sŁ N xA; N yA; N xB; N yB : : : kh¡c c¡c
sŁ nguy¶n nhä hìn sŁ N(xA; yA; xB; yB; : : :)- h» sŁ trong ph¥n t‰ch c¡c
! ! ! !
b ;. SŁ N ÷æc x¡c ành nhí ành
v†c tì OA; OB; : : : theo c¡c v†c tì a ;
lþ Kroneker: "Gi£ sß 1; 2; : : : ; n l c¡c sŁ thüc. Khi â vîi måi sŁ
Tải tài liệu tại sividoc.com
Viết đề tài giá sinh viên – ZALO:0973.287.149-TEAMLUANVAN.COM
19
d÷ìng t…m ÷æc sŁ tü nhi¶n m sao cho mØi sŁ m i, i = 1; 2; : : : n kh¡c
sŁ nguy¶n kho£ng , tøc l : fm ig 2 [0; ] [ [1 ; 1]" (Xem [6]).
B i to¡n 1.8. Tr¶n gi§y æ vuæng v‡ mºt ÷íng g§p khóc câ ¿nh l c¡c
nót sao cho c¡c o⁄n thflng •u b‹ng nhau. Chøng minh r‹ng sŁ c¡c o⁄n
thflng l sŁ chfin.
B i to¡n 1.9. Chøng minh r‹ng tr¶n l÷îi Z2
câ th” °t a gi¡c lçi •u c⁄nh
vîi sŁ c⁄nh chfin b§t ký.
1.3 Cæng thøc Picard
1.3.1 Tam gi¡c ìn nguy¶n thıy
Ng÷íi ta ph¥n lo⁄i a gi¡c th nh a gi¡c ìn v a gi¡c khæng ìn ( a gi¡c
phøc) nh÷ sau:
a gi¡c ìn (Simple polygon): a gi¡c m c¡c c⁄nh ch¿ câ th” c›t
nhau t⁄i c¡c ƒu mót ( ¿nh a gi¡c), khæng câ hai c⁄nh khæng k•
nhau c›t nhau. a gi¡c ìn câ th” l a gi¡c lçi ho°c a gi¡c lªm.
a gi¡c khæng ìn hay a gi¡c phøc hay(Complex polygon) l a gi¡c
câ hai c⁄nh khæng k• nhau c›t nhau, i”m c›t nhau â khæng ph£i l
¿nh cıa a gi¡c.
Chó þ r‹ng a gi¡c ìn câ th” l a gi¡c lªm ho°c a gi¡c lçi. a gi¡c phøc l a
gi¡c lªm. H…nh v‡ sau ¥y ch¿ ra a gi¡c ìn v khæng ìn:
H…nh 1.10: a gi¡c ìn v a gi¡c khæng ìn
Tải tài liệu tại sividoc.com
Viết đề tài giá sinh viên – ZALO:0973.287.149-TEAMLUANVAN.COM
20
1.3.2 Cæng thøc Picard
Mºt øng döng trüc ti‚p cıa l÷îi nguy¶n l t‰nh di»n t‰ch h…nh phflng
b‹ng c¡ch phı nâ bði l÷îi æ vuæng. Cæng thøc sau ¥y mang t¶n nh
to¡n håc ng÷íi Ph¡p Picard, ÷æc t…m ra v o n«m 1899.
M»nh • 1.9. (Cæng thøc Picard) Cho a gi¡c ìn D câ c¡c ¿nh l c¡c i”m
nguy¶n. SŁ i”m nguy¶n tr¶n c¡c c⁄nh l m, sŁ i”m nguy¶n b¶n trong a
gi¡c l n. Khi â di»n t‰ch cıa D b‹ng
SD = n +
m
1: (1.3)
2
Chøng minh. — ¥y tr…nh b y hai c¡ch chøng minh cæng thøc
Picard: C¡ch chøng minh truy•n thŁng v c¡ch chøng minh trüc
ti‚p(tham kh£o trong [3]). B i to¡n d¤n ra sau ¥y.
C¡ch 1. Cì sð cıa vi»c chøng minh cæng thøc Picard l x¡c ành di»n
t‰ch cıa h…nh tam gi¡c ìn nguy¶n thıy (câ c¡c ¿nh nguy¶n v khæng
chøa b¶n trong nâ ho°c tr¶n bi¶n mºt i”m nguy¶n n o). C¡c b÷îc
chøng minh qua c¡c k‚t qu£ sau:
B i to¡n 1.10. Trong mºt a gi¡c ìn vîi sŁ ¿nh lîn hìn ho°c b‹ng 4 luæn
tçn t⁄i ÷íng ch†o n‹m ð mi•n trong cıa nâ.
B i to¡n 1.11. Câ th” chia n-gi¡c ìn nguy¶n thıy th nh n gi¡c
sao cho c¡c tam gi¡c n y khæng câ i”m trong chung v cıa
tam gi¡c n‹m trong n-gi¡c â.
2 tam
c¡c c⁄nh
B i to¡n 1.12. Cho mºt a gi¡c ìn D m måi ¿nh l c¡c i”m nguy¶n. Khi â
câ th” nŁi c¡c i”m nguy¶n n‹m trong D v tr¶n c⁄nh cıa D ” ÷æc c¡c tam
gi¡c ìn.
B i to¡n 1.13. SŁ c¡c tam gi¡c ìn trong mºt tam gi¡c ph¥n cıa a gi¡c ìn
D l b‹ng nhau vîi måi ki”u tam gi¡c ph¥n. N‚u b¶n trong D câ k i”m
nguy¶n v tr¶n c⁄nh cıa D (k” c£ ¿nh) câ m i”m nguy¶n
th… sŁ c¡c tam gi¡c ìn l s = 2k + m 2
Tł c¡c k‚t qu£ tr¶n ta thu ÷æc m»nh • cì b£n sau:
Tải tài liệu tại sividoc.com
Viết đề tài giá sinh viên – ZALO:0973.287.149-TEAMLUANVAN.COM
21
M»nh • 1.10. Cho ABC l tam gi¡c ìn nguy¶n thıy b§t ký tr¶n
l÷îi nguy¶n mØi æ vuæng câ di»n t‰ch 1 ìn và. Khi â SABC =
1
. ”
2
chøng minh SABC =
1
ta s‡ chøng minh hai b§t flng:SABC
1
v
2 2
1
S
ABC .
2
Phı tam gi¡c ABC bði h…nh chœ nh“t MNPQ. Khi â, SABC = SMNPQ-
l m
tŒng di»n t‰ch c¡c tam gi¡c phö vîi ABC = k = vîi k; l; m 2 Z.
2 2
m 1
V“y S
ABC
=
2 2
:
X†t c¡c tam gi¡c ph¥n Łi vîi tam gi¡c phö vîi tam gi¡c ABC trong
h…nh chœ nh“t MNPQ. Khi â ta câ tam gi¡c ph¥n cho h…nh chœ nh“t
MNPQ vîi sŁ tam gi¡c t⁄o th nh l s 2(m 1)(n 1)+2(m+n) 2 = 2m:n,
trong â m; n l hai c⁄nh h…nh chœ nh“t. V… di»n t‰ch h…nh chœ nh“t
MNPQ b‹ng m:n n¶n tł flng thøc â suy ra måi tam gi¡c ph¥n
1
•u ph£i câ di»n t‰ch 2. Ta câ i•u ph£i chøng minh.
CuŁi còng, theo B i to¡n 1.13 ta câ th” chia D th nh 2k + m 2 tam
gi¡c ìn bði c¡c o⁄n thflng nŁi c¡c i”m nguy¶n n‹m trong ho°c tr¶n bi¶n
cıa nâ. Theo M»nh • 1.10 mØi mºt tam gi¡c ph¥n câ di»n t‰ch
1
b‹ng . V“y di»n t‰ch cıa a gi¡c D l
2
(2k + m 2): 1 = k + m 1:
2 2
Cæng thøc ÷æc chøng minh
C¡ch 2. (Tham kh£o trong [2])
Chøng minh. Vîi mØi a gi¡c D, vîi quy ÷îc m; k nh÷ gi£ thi‚t ta kþ
hi»u h m f(D) = k + m 1. Gi£ sß a gi¡c M ÷æc c›t th nh hai a
2
gi¡c M1; M2 vîi c¡c ¿nh l c¡c nót l÷îi. Ta s‡ chøng minh r‹ng cæng
thøc Picard n‚u óng Łi vîi 2 trong 3 a gi¡c M; M1; M2 th… công s‡
óng Łi vîi a gi¡c cÆn l⁄i. ” câ i•u â ta i chøng minh flng thøc
f(M) = f(M1) + f(M2) (1.4)
Tải tài liệu tại sividoc.com
Viết đề tài giá sinh viên – ZALO:0973.287.149-TEAMLUANVAN.COM
22
C¡ch l“p lu“n ð ¥y kh¡ °c bi»t nh÷ sau (xem [2])
+ Gåi o⁄n thflng c›t a gi¡c M th nh 2 a gi¡c M1; M2 l L. C¡c nót n‹m
ngo i L cæng l⁄i b‹ng f(M) v f(M1) + f(M2); c¡c nót khæng ph£i ƒu mót
cıa L cºng vîi 1 th nh f(M) v cºng vîi 0,5 th nh f(M1)
v f(M2). MØi mºt trong hai i”m nót m l ƒu mót cıa L cºng th¶m 0,5 v o
mØi sŁ f(M); f(M1); f(M2) v do â phƒn th¶m cıa c¡c nót t⁄i 2 ƒu mót L v
sŁ f(M) ‰t hìn 1 so vîi phƒn th¶m cıa c¡c nót t⁄i 2
ƒu mót L v o f(M1) + f(M2). V… chóng ta ph£i gi£m i 1 tł mØi lƒn
th¶m v o f(M) v gi£m i 2 tł mØi lƒn th¶m v o f(M1) + f(M2) n¶n suy ra
f(M) = f(M1) + f(M2).
+ B¥y gií ta i chøng minh cæng thøc Picard Łi vîi tam gi¡c tòy þ.
N‚u M l h…nh chœ nh“t vîi c¡c c⁄nh p; q th… f(M) = (p 1)(q 1)+
2(p + q)
1 = pq = Shcn, tøc l cæng thøc Picard óng Łi vîi h…nh
2
chœ nh“t. C›t h…nh chœ nh“t M th nh hai h…nh tam gi¡c (vuæng)
M1; M2 b‹ng ÷íng ch†o cıa nâ v sß döng k‚t qu£ f(M) = f(M1) + f(M2)
v f(M1) = f(M2) ta suy ra cæng thøc Picard óng Łi vîi tam gi¡c vuæng
câ hai c⁄nh gâc vuæng thuºc ÷íng thflng l÷îi.
+ B¥y gií tł tam gi¡c b§t ký, phı nâ b‹ng mºt h…nh chœ nh“t ta suy
ra i•u ph£i chøng minh.
+ CuŁi còng chó þ r‹ng måi a gi¡c •u câ th” c›t th nh c¡c tam
gi¡c bði c¡c ÷íng ch†o. Ta câ i•u ph£i chøng minh.
1.3.3 Mºt sŁ b i to¡n ¡p döng cæng thøc Picard
B i to¡n 1.14. Trong tam gi¡c ABC câ óng mºt i”m nguy¶n. Chøng
minh r‹ng i”m â l trång t¥m tam gi¡c.
Trong tam gi¡c ABC câ óng mºt i”m nguy¶n D. Khi â tam gi¡c ABC
÷æc ph¥n chia th nh 3 tam gi¡c ABD, ACD, BCD m mØi tam gi¡c
khæng chøa i”m nguy¶n n o ngo i ¿nh. Theo m»nh • tr¶n SABC =
1
SACD = SBCD = 2. Suy ra kho£ng c¡ch tł B v C ‚n AD b‹ng nhau,
v D thuºc ÷íng trung tuy‚n k· tł A. T÷ìng tü, D n‹m tr¶n trung tuy‚n k·
tł B. Ta k‚t lu“n D l trång t¥m tam gi¡c ABC.
Tải tài liệu tại sividoc.com
Viết đề tài giá sinh viên – ZALO:0973.287.149-TEAMLUANVAN.COM
23
B i to¡n 1.15. a. H¢y t‰nh di»n t‰ch nhä nh§t cıa ngô gi¡c lçi
nguy¶n câ th” ⁄t ÷æc.
b. N‚u ngô gi¡c tr¶n khæng lçi th… di»n t‰ch nhä nh§t b‹ng bao nhi¶u?
Theo V‰ dö 1.4.3 mºt a gi¡c lçi nguy¶n câ ‰t nh§t mºt i”m nguy¶n l
i”m trong, ngh¾a k 1; m 5. V“y di»n t‰ch ngô gi¡c b‹ng k+
m
1
2
1+ 5 1 =5 . M°t kh¡c luæn tçn t⁄i a gi¡c lçi nguy¶n câ di»n t‰ch b‹ng
2 2
5
2, chflng h⁄n c¡c ¿nh ngô gi¡c l O(0,0);A(0,1); B(1,2); C(2,1);D(1,0).
Ngô gi¡c n y câ 5 i”m nguy¶n tr¶n bi¶n v mºt i”m nguy¶n I(1,1) l
5
i”m trong. V“y theo cæng thøc Picard: SOABCD = .
2
3
Gåi l ngô gi¡c ìn khæng lçi th… sŁ i”m trong l k 0. S .
2
3 3
Ngô gi¡c ìn ABCDI câ di»n t‰ch b‹ng . V“y di»n t‰ch b‹ng .
2 2
H…nh 1.11: Mºt sŁ b i to¡n li¶n quan
B i to¡n 1.16. Câ th” °t ÷æc hay khæng mºt tam gi¡c vîi c⁄nh nguy¶n
( º d i c¡c c⁄nh l sŁ nguy¶n) sao cho c¡c ¿nh l c¡c nót l÷îi nh÷ng
khæng câ c⁄nh n o thuºc ÷íng thflng l÷îi?
Chøng minh. C¥u tr£ líi l câ. D„ ki”m tra ÷æc tam gi¡c vîi c¡c ¿nh câ
tåa º (0,0), (12,16), (-12,9) thäa m¢n y¶u cƒu • b i
Tải tài liệu tại sividoc.com
Viết đề tài giá sinh viên – ZALO:0973.287.149-TEAMLUANVAN.COM
24
B i to¡n 1.17. Câ hay khæng mºt ÷íng g§p khóc kh†p k‰n gçm mºt
sŁ l· c¡c o⁄n thflng nguy¶n b‹ng nhau m c¡c ¿nh l c¡c nót cıa l÷îi?
Chøng minh. C¥u tr£ líi l khæng. Gi£ sß tçn t⁄i ÷íng g§p khóc A1A2 :
: : An nh÷ y¶u cƒu, gçm c¡c o⁄n thflng câ º d i c. X†t v†c
tì . Gåi a
i
v b
i
l º d i ( ⁄i sŁ) h…nh chi‚u cıa v†c tì
! AiAi+1
! AiAi+1
t÷ìng øng tr¶n c¡c tröc tåa º Oy, Ox. Ta câ flng thøc c2
= a2
i + b2
i vîi
mØi i = 1 : : : n:: D÷ cıa ph†p chia c2
cho 4 l 0,1,2. N‚u c2
chia h‚t
cho 4 th… tł flng thøc tr¶n ta suy ra ai v bi công chia h‚t cho 4 ( x†t
c¡c d÷ khi chia cho 4 cıa ai v bi). Do â, ph†p và tü t¥m A1, t sŁ và tü
0,5 bi‚n ÷íng g§p khóc A1A2 : : : An th nh ÷íng g§p khóc B1B2 : : : Bn
ng›n hìn m c¡c ¿nh v¤n l nót cıa l÷îi. Ti‚p töc l m nh÷ v“y, d¤n tîi mºt
÷íng g§p khóc m c2
cıa nâ khæng chia h‚t cho 4. Chó þ r‹ng vîi måi
÷íng g§p khóc nh÷ th‚ th…
a1 + a2 + : : : + an = b1 + b2 + : : : + bn = 0
Tr÷íng hæp 1. N‚u d÷ khi chia c2
cho 4 l 1 th… mºt trong c¡c sŁ ai,bi
l l· v sŁ kia l chfin n¶n tŒng a1 + a2 + : : : + an 6= 0
Tr÷íng hæp 2. N‚u d÷ khi chia c2
cho 4 l 2 th… c£ hai sŁ ai,bi •u l· n¶n
tŒng a1 + a2 + : : : + an + b1 + b2 + : : : + bn l sŁ l·, kh¡c 0. Væ lþ.
B i to¡n 1.18. X†t tam gi¡c nguy¶n sao cho tr¶n c⁄nh cıa nâ khæng
chøa c¡c i”m nguy¶n n o kh¡c c¡c ¿nh. Chøng minh r‹ng n‚u mºt tam
gi¡c nh÷ v“y chøa trong nâ óng mºt i”m nguy¶n th… i”m n y ch‰nh l
trång t¥m cıa tam gi¡c.
B i to¡n 1.19. Gi£ sß mºt n-gi¡c nguy¶n lçi sao cho b¶n trong v c⁄nh
cıa nâ khæng chøa mºt i”m nguy¶n n o kh¡c c¡c ¿nh, chøng tä n 4.
B i to¡n 1.20. Chøng minh r‹ng vîi b§t k… hai i”m nguy¶n A,B sao
cho giœa o⁄n nŁi chóng khæng chøa i”m nguy¶n n o kh¡c, t…m ÷æc
i”m C sao cho tam gi¡c ABC l nguy¶n thıy. Kho£ng c¡ch tł C ‚n AB
b‹ng bao nhi¶u n‚u bi‚t º d i AB = d?
B i to¡n 1.21. X†t n i”m nguy¶n(n 3) sao cho b§t k… ba i”m n o
giœa chóng •u l“p th nh ÷æc tam gi¡c m c¡c trung tuy‚n cıa nâ
khæng c›t mºt i”m nguy¶n n o kh¡c. T…m sŁ n lîn nh§t câ th” ÷æc.
Tải tài liệu tại sividoc.com
Viết đề tài giá sinh viên – ZALO:0973.287.149-TEAMLUANVAN.COM
25
B i to¡n 1.22. Chøng minh r‹ng mØi h…nh vuæng câ c⁄nh b‹ng n
n‹m trong m°t phflng chøa khæng nhi•u hìn (n + 1)2
i”m nguy¶n.
1.4 Mºt sŁ øng döng cıa l÷îi tåa º nguy¶n
Vîi nhœng hi”u bi‚t v• l÷îi, ta câ th” câ mºt sŁ øng döng câ ‰ch sau.
1.4.1 Gi¡ trà væ t cıa h m l÷æng gi¡c
Ta thß °t c¥u häi: c¡c sŁ sau l sŁ hœu t hay sŁ væ t :
sin 1 ; cos ; tan 7 ?
19 55
p
Mºt c¡ch tŒng qu¡t: Vîi mØi gâc = (p; q 2 Z), c¡c sŁ cos ; sin ;
q
tan l sŁ hœu t hay sŁ væ t ? Rª r ng sin hœu t vîi = k ; =
k
2
(3k 1); cos hœu t vîi = (6k 1); tan hœu t vîi
; =
3 2 6
+ k (k 2 Z).
= k ; = 4
p
Ta s‡ th§y vîi c¡c gâc cÆn l⁄i = q gi¡ trà cıa c£ 3 h m sŁ •u l
sŁ væ t . ” chøng minh i•u â ta nh›c l⁄i hai k‚t qu£ ¢ bi‚t trong l÷æng
gi¡c.
K‚t qu£ 1.Vîi måi sŁ n 2 N h m sŁ cos nx l mºt a thøc h» sŁ nguy¶n,
b“c n Łi vîi cos x, tøc l cos nx = Pn(cos x), trong â, Pn(y) l a thøc h»
sŁ nguy¶n b“c n.
K‚t qu£ 2. H m sŁ sin nx l t‰ch cıa h m sin x vîi mºt a thøc b“c
n 1 cıa cos x; h» sŁ nguy¶n, tøc l : sin nx = sin x:Qn 1(cos x); trong
â, Qn 1(y) l a thøc h» sŁ nguy¶n b“c n 1.
M»nh • 1.11. Ta câ c¡c h» qu£ sau:
a. N‚u cos x l sŁ hœu t th… cos nx công l
v sin nx thæng ÷îc (khi sin x =6 0).
sŁ hœu t , cÆn c¡c sŁ sin x
b. N‚u (p; q) = 1 v cos q l sŁ hœu t th… cos q công l sŁ
hœu t .
Tải tài liệu tại sividoc.com
Viết đề tài giá sinh viên – ZALO:0973.287.149-TEAMLUANVAN.COM
26
Chøng minh.
a. Hi”n nhi¶n.
b. Nh“n x†t r‹ng n‚u p; q > 1; (p; q) = 1 th… tçn t⁄i sŁ tü nhi¶n k sao
cho kp khi chia cho q ÷æc d÷ l 1, tøc l : kp = rq + 1. Bði v“y n‚u
cospl sŁ hœu t th… sŁ sau công l sŁ hœu t :
q
cos k q = cos = ( 1)r
cos q
q
p rq + 1
p
Tł flng thøc â công suy ra : N‚u cos l sŁ hœu t th… cos công
q q
l sŁ hœu t .
p
Nh÷ v“y, sŁ cos vîi p; q > 1; (p; q) = 1 hœu t khi v ch¿ khi cos
q q
hœu t .
B¥y gií ta chøng minh øng döng
M»nh • 1.12. SŁ cos l sŁ væ t vîi måi sŁ tü nhi¶n q > 3.
q
p m1
2 N). Tr¶n m°t
Chøng minh. Gi£ sß cos = l sŁ hœu t (m1; n1
q n
1
phflng l§y h» tåa º Descarte vuæng gâc v tł gŁc tåa º O ta k· c¡c
tia t⁄o vîi h÷îng d÷ìng cıa tröc Ox c¡c gâc ; 2 ;:::; (2q 1) . Gåi
q
q q
A0; A1; A2; : : : ; A2q 1
l
giao cıa c¡c tia n y vîi ÷íng trÆn ìn và. a
gi¡c A0A1A2 : : : A2q 1
l
a gi¡c •u, i”m Ak câ tåa º cos
k
; sin
k
.
q q
V… sŁ cos l sŁ hœu t n¶n ho nh º cıa i”m Ak l sŁ hœu t :
q
k mk
cos = pk(cos ) =
q q n
k
rk
Tung º công b‹ng t‰ch c¡c sŁ hœu t Qk(cos ) = vîi sŁ sin ; k =
q s
k q
1; 2; : : : ; 2q 1; tåa º i”m A0 = (1; 0).
Tải tài liệu tại sividoc.com
tài giá sinh viên – ZALO:0973.287.149-TEAMLUANVAN.COM
27
Tải tài liệu tại sividoc.com
tài giá sinh viên – ZALO:0973.287.149-TEAMLUANVAN.COM
Nh÷ v“y, Ak = mk ; rk: sin . Kþ hi»u m¤u sŁ chung cıa hai ph¥n
n q
s k
k
sŁ mk ; rk l D th… Ak = Mk ;Nk : sin , trong â Mk; Nk l c¡c sŁ
n D D q
s
k
k
nguy¶n, k = 1; 2; : : : ; 2q 1. D; D: sin q (i; j l c¡c sŁ
B¥y gií x†t t§t c £ c¡ c i” m câ t åa º
i j
nguy¶n), c¡c i”m n y t⁄o th nh mºt l÷îi, hìn nœa c¡c ¿nh cıa 2q gi¡c
•u A0A1A2 : : : A2q 1 l c¡c nót cıa l÷îi. Nh÷ng i•u â khæng th” ÷æc
(xem phƒn tr÷îc) v… 2q 8. Bði v“y i•u gi£ sß "vîi q > 3 sŁ cos l
q
sŁ hœu t " l sai, v“y t§t c£ c¡c sŁ cos p(q > 3; (p; q) = 1) l væ t .
q
B i to¡n 1.23. Cho q 3; (p; q) = 1, chøng minh r‹ng:
p
a/ c¡c sŁ sin vîi q 6= 6 l c¡c sŁ væ t ,
q
p
vîi q 6= 4 l
b/ c¡c sŁ tan c¡c sŁ væ t .
q
TŒng hæp l⁄i câ th” ph¡t bi”u th nh
p p p
6= 0;
1
; 1 công nh÷
M»nh • 1.13. C¡c sŁ arcsin ; arccos vîi
q q q 2
p p
c¡c sŁ arctan vîi 6= 0; 1; ((p; q) = 1) •u l væ ÷îc vîi sŁ .
q q
B i to¡n 1.24. (IMO-2016,#3) Gi£ sß P = A1A2 : : : Ak l a gi¡c lçi trong
m°t phflng vîi c¡c ¿nh A1; A2; : : : ; Ak câ tåa º nguy¶n v n‹m tr¶n mºt
÷íng trÆn. Gåi S l di»n t‰ch cıa a gi¡c P. Mºt sŁ tü nhi¶n l· n thäa
m¢n b…nh ph÷ìng º d i c¡c c⁄nh cıa P •u chia h‚t cho n. Chøng minh
r‹ng 2S l sŁ nguy¶n chia h‚t cho n.
1.4.2 C¡c b i to¡n tr¶n æ vuæng
B£ng æ vuæng l mºt h…nh chœ nh“t ÷æc chia th nh m:n æ vuæng
b‹ng nhau, ta gåi l b£ng æ vuæng k‰ch th÷îc m n. Gi§y æ vuæng l
v‰ dö v• b£ng æ vuæng. B£ng æ vuæng l mºt phƒn cıa l÷îi nguy¶n
v… th‚ ta câ th” x†t c¡c b i to¡n tr¶n æ vuæng vîi nhœng hi”u bi‚t v• l÷îi
tåa º nguy¶n. Tuy nhi¶n do °c i”m cıa c¡c b i to¡n n y l chóng
Tải tài liệu tại sividoc.com
tài giá sinh viên – ZALO:0973.287.149-TEAMLUANVAN.COM
28
câ t‰nh hœu h⁄n n¶n ta cÆn ¡p döng ÷æc c¡c ph÷ìng ph¡p nh÷
ph÷ìng ph¡p tæ m u, ph÷ìng ph¡p Dirichle, ph÷ìng ph¡p cüc h⁄n,...
Nguy¶n lþ Dirichle Thüc ch§t l ph÷ìng ph¡p ph£n chøng: N‚u em
nhŁt m thä v o n chi‚c lçng vîi m > n th… ‰t nh§t công câ mºt lçng
nhŁt khæng ‰t hìn 2 thä.
V‰ dö 1.4.1. Trong mºt b£ng æ vuæng 8 8 ng÷íi ta vi‚t v o mØi æ
vuæng mºt sŁ tü nhi¶n tł 1 ‚n 64. Chøng minh r‹ng luæn tçn t⁄i 2 æ
vuæng câ chung c⁄nh m hi»u c¡c sŁ ð hai æ â khæng nhä hìn 5.
V‰ dö 1.4.2. Tr¶n m°t phflng l§y 5 i”m nguy¶n ph¥n bi»t. Chøng
minh r‹ng tçn t⁄i trong chóng hai i”m nguy¶n sao cho o⁄n thflng nŁi
hai i”m n y i qua mºt i”m nguy¶n n o â.
Chøng minh. °t X1 = 2k + 1; X2 = 2k vîi k 2 Z. Måi i”m M(X1; X2) thuºc
mºt trong 4 t“p hæp: d1 = (X1; X1); d2 = (X1; X2); d3 = (X2; X1); d4 =
(X2; X2). BŁn t“p hæp n y ríi nhau vîi mØi k 2 Z. Theo nguy¶n t›c Dirichle
khi câ 5 i”m nguy¶n th… câ ‰t nh§t 2 i”m thuºc còng mºt trong bŁn t“p
hæp d1; d2; d3; d4. Gi£ sß hai i”m A1(x1; y1), A2(x2; y2) thuºc còng
trong 4 t“p hæp tr¶n. Khi â trung i”m I cıa A1A2 câ tåa
Tải tài liệu tại sividoc.com
tài giá sinh viên – ZALO:0973.287.149-TEAMLUANVAN.COM
º I
x1 + x2
;
y1 + y2
. V… A1; A2 thuºc v o 1 t“p hæp trong 4 t“p hæp
2 2
tr¶n n¶n d„ ki”m tra ÷æc tåa º cıa I l c¡c sŁ nguy¶n. V“y trong 5 i”m
luæn tçn t⁄i hai i”m sao cho o⁄n thflng nŁi chóng i qua mºt i”m
nguy¶n n o â ( â ch‰nh l trung i”m cıa o⁄n thflng nŁi hai i”m).
V‰ dö 1.4.3. Chøng minh r‹ng trong ngô gi¡c lçi nguy¶n luæn tçn t⁄i
câ ‰t nh§t mºt i”m nguy¶n l i”m trong.
Chøng minh. Theo v‰ dö tr¶n trong 5 i”m A, B, C, D, E tçn t⁄i trung
i”m cıa o⁄n thflng nŁi 2 i”m l i”m nguy¶n. Câ hai kh£ n«ng cıa o⁄n
thflng:
Kh£ n«ng 1: o⁄n thflng n y l ÷íng ch†o ngô gi¡c, hi”n nhi¶n b i to¡n
÷æc gi£i.
Kh£ n«ng 2: o⁄n thflng n y l c⁄nh cıa ngô gi¡c, gi£ sß â l c⁄nh DE v E1 l
trung i”m cıa DE. Khi â ngô gi¡c ABCDE1 câ 5 i”m
Tải tài liệu tại sividoc.com
Viết đề tài giá sinh viên – ZALO:0973.287.149-TEAMLUANVAN.COM
29
nguy¶n v công l ngô gi¡c lçi. GiŁng nh÷ ngô gi¡c ABCDE, câ 2 tr÷íng
hæp: N‚u rìi v o kh£ n«ng 1 nh÷ tr¶n th… ta dłng l⁄i, n‚u rìi v o kh£
n«ng thø hai ta l°p l⁄i qu¡ tr…nh. Qu¡ tr…nh n y hœu h⁄n bði v“y sau
mØi b÷îc ta l⁄i t…m ÷æc mºt i”m nguy¶n n‹m tr¶n c⁄nh ngô gi¡c. Nh÷
v“y t…m ÷æc væ sŁ i”m nguy¶n n‹m tr¶n c⁄nh cıa ngô gi¡c. i•u væ lþ
n y chøng minh b i to¡n.
Nh“n x†t 1.4.1. Câ th” x†t a gi¡c lçi b§t ký mi„n sao câ 5 ¿nh câ tåa º
nguy¶n.
p p
V‰ dö 1.4.4. a. Tr¶n m°t phflng tåa º l§y i”m A( 2; 3). Chøng minh
r‹ng kho£ng c¡ch tł A ‚n c¡c i”m nguy¶n l æi mºt ph¥n bi»t. b.
Chøng minh r‹ng vîi måi sŁ nguy¶n d÷ìng n tçn t⁄i mºt h…nh trÆn
câ óng n i”m nguy¶n.
Chøng minh. a. L§y hai i”m B(x; y); C(u; v) vîi x; y; u; v 2 Z. Ta câ:
AB2
= (x
p
2)2
+ (y
p
3)2
; AC2
= (u
p
2)2
+ (v
p
3)2
. Ta s‡ chøng
minh n‚u câ hai i”m nguy¶n còng c¡ch •u i”m A th… hai i”m
§y tròng nhau.
p p p p
N‚u AB = AC =) (x 2)2
+(y 3)2
= (u 2)2
+(y 3)2
=)
x
2
+ y
2
u
2
v
2 p
u) + 2
p
sŁ nguy¶n
2 2(x 3(y v). V… v‚ tr¡i l
n¶n v‚ ph£i công l sŁ nguy¶n. V“y x = u; y = v. Suy ra B C.
p p
b. Do A( 2; 3) c¡ch c¡c i”m nguy¶n vîi kho£ng c¡ch æi mºt
kh¡c nhau n¶n ta s›p ÷æc c¡c i”m nguy¶n theo thø tü c¡c kho£ng
c¡ch t«ng dƒn: d1 < d2 < : : : < dn < : : :. L§y b¡n kinh R thäa m¢n dn
< R < dn+1. Khi â h…nh trÆn t¥m A, b¡n k‰nh R chøa óng n i”m
nguy¶n. (Ta câ th” chån A l i”m nguy¶n b§t ký).
B i to¡n 1.25. Cho h…nh vuæng c⁄nh d i 15 cm. Ng÷íi ta chia h…nh
vuæng â th nh 255 h…nh vuæng nhä k‰ch th÷îc 1 cm v ¡nh d§u
t¥m cıa mØi h…nh vuæng nhä b‹ng mºt d§u ch§m. Häi r‹ng trong
m°t phflng chøa h…nh vuæng ¢ cho vîi 27 ÷íng thflng li»u câ th” chia
h…nh vuæng â th nh c¡c phƒn sao cho mØi phƒn câ khæng qu¡ 1
d§u ch§m ÷æc khæng?
Tải tài liệu tại sividoc.com
Viết đề tài giá sinh viên – ZALO:0973.287.149-TEAMLUANVAN.COM
30
B i to¡n 1.26. Trong h…nh vuæng c⁄nh 1m ng÷íi ta v‡ c¡c ÷íng trÆn
b§t ký m tŒng c¡c b¡n k‰nh cıa chóng b‹ng 0,6 m. Chøng minh r‹ng
câ th” t…m ÷æc ÷íng thflng song song vîi c⁄nh cıa h…nh vuæng m tr¶n
÷íng thflng â câ nhœng i”m chung cıa ‰t nh§t 2 ÷íng trÆn ¢ cho.
V‰ dö 1.4.5. (BŒ • Minkovski) GŁc tåa º l t¥m Łi xøng cıa mºt
h…nh lçi H câ di»n t‰ch lîn hìn 4 ( ìn và æ vuæng). Chøng minh
r‹ng h…nh H chøa mºt i”m nguy¶n kh¡c gŁc tåa º.
Chøng minh. X†t t§t c£ nhœng h…nh lçi nh“n ÷æc tł H qua ph†p
tành ti‚n theo c¡c v†c tì m c£ hai tåa º •u chfin. Ta s‡ chøng minh khi
â tçn t⁄i ‰t nh§t 2 trong sŁ c¡c h…nh thu ÷æc câ giao kh¡c rØng.
Câ th” giîi h⁄n h…nh H bði mºt h…nh trÆn t¥m t⁄i gŁc tåa º, b¡n
k‰nh R 2 Z. Trong sŁ c¡c h…nh ang x†t, l§y c¡c h…nh vîi t¥m Łi xøng
câ tåa º khæng ¥m, nhä hìn ho°c b‹ng 2n. Khi â ta câ (n + 1)2
h…nh v
t§t c£ c¡c h…nh n y •u n‹m trong h…nh vuæng câ c⁄nh 2:(n + R).
N‚u c¡c h…nh n y khæng giao nhau vîi måi n th… (n + 1)2
SH 4(n +
R)2
.Nh÷ng v… SH > 4 n¶n câ th” chån n sao cho
n + R
<
r SH
. Ngh¾a
n + 1 4
l tçn t⁄i n sao cho câ ‰t nh§t hai h…nh câ giao kh¡c rØng.
Gi£ sß H1 câ t¥m Łi xøng O1; H2 câ t¥m Łi xøng O2 câ i”m chung
A. Ta s‡ khflng ành ÷æc trung i”m M cıa O1O2 thuºc c£ hai h…nh.
O B O B. V… h…nh câ t¥m Łi xøng
Th“t v“y l§y i”m B sao cho
!
1 =
!
2
n¶n B 2 H1. Do A; B 2 H1 n¶n trung i”m cıa AB công thuºc H1 (v…
t‰nh ch§t lçi) v trung i”m cıa AB công l trung i”m cıa O1O2. V“y
!
M 2 H1 v M khæng tròng vîi O1. Dòng ph†p tành ti‚n theo v†c tì O1O
bi‚n M th nh M0
, ta ־c M0
l i”m cƒn t…m.
V‰ dö 1.4.6. Tr¶n tí gi§y câ v‚t müc di»n t‰ch nhä hìn 1 ( ìn và di»n
t‰ch). Chøng minh r‹ng câ th” thi‚t k‚ mºt m⁄ng æ vuæng phı l¶n tí
gi§y sao cho khæng câ ¿nh n o cıa m⁄ng l÷îi rìi v o v‚t müc.
Chøng minh. Tr÷îc h‚t dòng mºt tí gi§y tr›ng rçi phı tí gi§y b‹ng mºt
m⁄ng l÷îi æ vuæng câ c⁄nh song song vîi c⁄nh tí gi§y. C›t c¡c æ
vuæng ìn và ra khäi tí gi§y v °t chóng chçng l¶n nhau. Gi£ sß müc
câ th” th§m tł æ vuæng n y sang æ vuæng kh¡c. Theo gi£ thi‚t, di»n
Tải tài liệu tại sividoc.com
Viết đề tài giá sinh viên – ZALO:0973.287.149-TEAMLUANVAN.COM
31
t‰ch v‚t müc nhä hìn 1 n¶n câ 1 i”m cıa h…nh vuæng khæng bà
th§m müc. Ti‚p theo ta tr£i c¡c æ vuæng n y v• và tr‰ ban ƒu th…
nhœng i”m n y s‡ t⁄o n¶n mºt h» thŁng ¿nh m⁄ng l÷îi æ vuæng ìn và.
Hi”n nhi¶n khæng câ i”m n o trong chóng n‹m trong v‚t müc.
B i to¡n 1.27. Cho h…nh phflng F câ di»n t‰ch lîn hìn 1. Chøng minh
!
AB câ tåa º nguy¶n.r‹ngtçnt⁄ihaii”mA,BthuºcFsaocho
Ph÷ìng ph¡p gi£i: p döng nguy¶n t›c Dirichle.
B i to¡n 1.28. Trong mºt a gi¡c lçi câ chøa khæng ‰t hìn m2
+1 i”m
nguy¶n. Chøng minh r‹ng trong a gi¡c tçn t⁄i m+1 i”m nguy¶n thflng h
ng.
B i to¡n 1.29. (VMO-2010,#5) Cho b£ng 3 3 v n l sŁ tü nhi¶n cho
tr÷îc. T…m sŁ c¡c c¡ch tæ m u khæng nh÷ nhau khi tæ mØi æ bði 1
trong n m u (hai c¡ch tæ m u gåi l nh÷ nhau n‚u c¡ch n y nh“n ÷æc tł
c¡ch kia b‹ng c¡ch quay quanh t¥m).
Ph÷ìng ph¡p gi£i: Thß vîi n = 1, 2 ” t…m ra h÷îng gi£i. Ta s‡ t…m
c¡ch ‚m sŁ c¡c lîp t÷ìng ÷ìng trong t§t c£ n9
c¡ch tæ m u. K‚t qu£:
n3
+ n5 n3
+ n9 n5
=n9
+ n5
+ 2n3
:
2 4 4
B i to¡n 1.30. Cho ngô gi¡c lçi ABCDE c¡c ¿nh câ tåa º nguy¶n.
Chøng minh r‹ng
a. Tçn t⁄i ‰t nh§t mºt i”m nguy¶n n‹m trong ho°c tr¶n c⁄nh ngô gi¡c
kh¡c c¡c ¿nh.
b. Tçn t⁄i ‰t nh§t mºt i”m nguy¶n n‹m trong ngô gi¡c kh¡c c¡c ¿nh.
c. Tçn t⁄i ‰t nh§t mºt i”m nguy¶n n‹m trong ho°c tr¶n c⁄nh ngô gi¡c
lçi A1B1C1D1E1 t⁄o th nh tł c¡c ÷íng ch†o cıa ngô gi¡c lçi ¢ cho.
Ph÷ìng ph¡p gi£i: Dòng nguy¶n t›c Dirichle v nguy¶n t›c cüc h⁄n. Nºi
dung cıa Ch÷ìng 1 kh¡ phong phó • c“p ‚n l÷îi nguy¶n vîi a gi¡c •u,
hai b i to¡n v• a gi¡c nßa •u v a gi¡c hƒu nºi ti‚p l÷îi nguy¶n còng
cæng thøc Picard. C¡c k‚t qu£ ð ¥y ÷æc ¡p döng v o b i
Tải tài liệu tại sividoc.com
Viết đề tài giá sinh viên – ZALO:0973.287.149-TEAMLUANVAN.COM
32
to¡n tr¶n æ vuæng, t‰nh di»n t‰ch a gi¡c tr¶n l÷îi, t‰nh gi¡ trà væ t cıa
h m l÷æng gi¡c,. . . Ch÷ìng ti‚p theo nâi v• quan h» giœa l÷îi
nguy¶n v ÷íng trÆn.
Tải tài liệu tại sividoc.com
Viết đề tài giá sinh viên – ZALO:0973.287.149-TEAMLUANVAN.COM
33
Ch֓ng 2
L÷îi tåa º nguy¶n v ÷íng trÆn
Sau khi x†t c¡c a gi¡c tr¶n l÷îi thu ÷æc mºt sŁ k‚t qu£ tŁt, trong
ch÷ìng n y ta công °t ÷íng trÆn tr¶n l÷îi v x†t sŁ c¡c i”m nguy¶n trong
v tr¶n ÷íng trÆn. Thüc ch§t b i to¡n li¶n quan ‚n sŁ håc, câ th” gi£i
quy‚t ÷æc c¡c v§n • nh÷ bº ba Pythagoras, sŁ c¡c bi”u di„n mºt sŁ tü
nhi¶n th nh tŒng hai b…nh ph÷ìng... Düa tr¶n t i li»u tham kh£o [2] v
[5].
2.1 i”m nguy¶n nguy¶n tŁ
¢ câ nhi•u t i li»u giîi thi»u v• " i”m nguy¶n-l÷îi tåa º nguy¶n".
— ¥y chóng tæi nh›c l⁄i v bŒ sung th¶m v• c¡c i”m n y tr¶n cì sð k‚t
hæp giœa sŁ håc, ⁄i sŁ v h…nh håc, ÷a ra øng döng cıa c¡c kh¡i
ni»m v o mºt sŁ v§n • cıa sŁ håc v h…nh håc. C¡c i”m nguy¶n ìn
gi£n l c¡c c°p sŁ nguy¶n m ta kþ hi»u l t“p hæp
Z2
= f(a; b)ja; b 2 Zg:
Câ hai ph†p to¡n cºng v nh¥n tr¶n Z2
x¡c ành nh÷ sau
(a; b) + (c; d) = (a + c; b + d); (a; b):(c; d) = (ac bd; ad + bc): (2.1)
” d„ quan s¡t câ th” bi”u di„n t“p hæp Z2
nh÷ l mºt "l÷îi nguy¶n tr¶n
m°t phflng", tøc l t“p hæp t§t c£ c¡c i”m tr¶n m°t phflng tåa º vîi ho nh
º v tung º •u l c¡c sŁ nguy¶n ho°c t“p hæp c¡c v†ctì vîi gŁc O, c¡c i”m
ngån l c¡c i”m n y. Khi â tŒng cıa hai i”m
Tải tài liệu tại sividoc.com
Viết đề tài giá sinh viên – ZALO:0973.287.149-TEAMLUANVAN.COM
34
nguy¶n bi”u di„n nh÷ tŒng v†ctì thæng th÷íng theo quy t›c h…nh b…nh
h nh cÆn ph†p nh¥n l ph†p to¡n n o â m ta bi”u di„n nh÷ tr¶n h…nh
v‡ 1.
H…nh 2.1: Cºng v nh¥n c¡c i”m nguy¶n
C¡ch thø ba bi”u di„n i”m nguy¶n z = (a; b) l d⁄ng tŒng a + bi,
trong â i kþ hi»u v†ctì ìn và cıa tröc Oy, tøc l c°p (0,1) = i. C°p (a; 0) l
c¡c i”m nguy¶n n‹m tr¶n tröc Ox, c°p (0; b) tøc l c¡c i”m nguy¶n cıa
tröc Oy câ d⁄ng bi. Chó þ r‹ng i2
= (0; 1):(0; 1) =
( 1; 0) = 1. Nh÷ v“y, trong G æi khi xu§t hi»n c«n b“c hai cıa sŁ
pp
¥m: 1 = i; 9 = 3i; .... MØi i”m nguy¶n z = a + bi 2 G
câ th” l“p i”m nguy¶n li¶n hæp vîi nâ: z = a bi 2 G v chu'n l : N(z) = a2
+ b2
2 Z
2
. Sß döng c¡c sŁ li¶n hæp, chu'n v cæng thøc ph†p nh¥n d„
nh“n ÷æc quy t›c ” chia i”m nguy¶n z = a + bi cho i”m nguy¶n w = c +
di. Cö th”,
z zw zw (a + bi)(c di) ac + bd bc ad
= = = = + i: (2.2)
w N(w) c
2
+ d
2
c
2 2
c
2 2
w:w + d + d
Công nh÷ c¡c sŁ nguy¶n: 1; 2 = 1 + 1; 3 = 1 + 1 + 1; :::; c¡c i”m nguy¶n
câ th” bi”u di„n qua sŁ 1 v i. Chflng h⁄n, 5 3i = 1+1+1+1+
1 + ( i) + ( i) + ( i). Nh÷ th‚, vîi c¡c i”m nguy¶n câ th” câ ành lþ
cì b£n giŁng nh÷ ành lþ cì b£n cıa sŁ håc ” ÷a vi»c nghi¶n cøu c¡c
Tải tài liệu tại sividoc.com
Viết đề tài giá sinh viên – ZALO:0973.287.149-TEAMLUANVAN.COM
35
i”m nguy¶n v• vi»c nghi¶n cøu c¡c " i”m nguy¶n nguy¶n tŁ"
2.1.1 Sü chia h‚t cıa c¡c i”m nguy¶n
N‚u vîi hai i”m nguy¶n a; b 2 Z2
t…m ÷æc c 2 Z2
sao cho a = b:c
th… ta nâi a chia h‚t cho b, kþ hi»u l a
..
.b hay b chia h‚t a, kþ hi»u l b|a.
V‰ dö 2
..
.1 + i v… 2 = (1 + i)(1 i). Tr¶n H…nh 2.2 ch¿ ra c¡c i”m
nguy¶n chia h‚t cho 2 + 3i.
Ta câ c¡c k‚t qu£ hi”n nhi¶n sau: Quan h» chia h‚t trong Z2
câ c¡c
t‰nh ch§t chia h‚t thæng th÷íng:
N‚u a
..
.b th… ac
..
.bc;
N‚u ac
..
.bc; c 6= 0 th… a
..
.b;
N‚u a
..
.b; b
..
.c th… a
..
.c.
. .
Trong lþ thuy‚t c¡c sŁ nguy¶n ta câ tł a; b 2
. .
Z, a.b; b.a k†o theo
a
. = b.. Trong c¡c i”m nguy¶n Z
2
i•u â khæng óng, chflng h⁄n:
. .
a. a, a. ia. Tuy nhi¶n ta câ
. .
. .
ib: (2.3)
a.b; b. a =) a = b ho°c a =
Chøng minh. Th“t v“y, gi£ sß a = c khi â c v c 1
= b theo gi£
a
b
thi‚t, l c¡c i”m nguy¶n. Hi”n nhi¶n 1 = N(1) = N(c:c 1
). Tł â, N(c) = N(c
1
) = 1 (v… N(c) v N(c 1
) l c¡c sŁ nguy¶n d÷ìng thæng th÷íng). Nh÷ng
ch¿ tçn t⁄i 4 i”m nguy¶n vîi chu'n b‹ng 1, â l c¡c sŁ 1, -1, i, i m trong
phƒn ti‚p theo ta s‡ gåi l c¡c ìn và.
Hai i”m nguy¶n ÷æc gåi l li¶n k‚t n‚u th÷ìng cıa chóng l mºt trong
c¡c ìn và 1; 1; i; i. i”m nguy¶n p ÷æc gåi l i”m nguy¶n nguy¶n tŁ n‚u p
khæng câ ÷îc kh¡c khæng n o kh¡c trł c¡c li¶n k‚t vîi chóng v c¡c ìn
và. Chflng h⁄n, c¡c sŁ 3 v 2 + 3i l c¡c i”m nguy¶n nguy¶n tŁ cÆn 5
khæng ph£i l i”m nguy¶n nguy¶n tŁ v… 5 = (2+i)(2 i).
” chøng minh 3 v 2 + 3i l i”m nguy¶n nguy¶n tŁ cƒn ph£i sß döng
h» thøc N(a)N(b) = N(ab) v ch⁄y lƒn l÷æt qua c¡c i”m nguy¶n m câ
chu'n l ÷îc cıa chu'n cıa sŁ tr÷îc nâ.
Tải tài liệu tại sividoc.com
Viết đề tài giá sinh viên – ZALO:0973.287.149-TEAMLUANVAN.COM
36
.
.
H…nh 2.2: a.C¡c i”m nguy¶n.(2 + 3i); b. C¡c i”m nguy¶n li¶n k‚t
2.1.2 ành lþ cì b£n cıa c¡c i”m nguy¶n
Nh›c l⁄i ành lþ cì b£n cıa sŁ håc c¡c sŁ nguy¶n:
ành lþ 2.1. MØi sŁ nguy¶n a 6= 0 •u bi”u di„n ÷æc d÷îi d⁄ng t‰ch
a = :pe
11
pe
22
: : : pe
nn
trong â, n > 0, l mºt trong c¡c ìn và, pi l c¡c sŁ nguy¶n tŁ ph¥n bi»t
(tøc l khæng li¶n k‚t) cÆn e1; e2; : : : en l c¡c sŁ nguy¶n d÷ìng. Ph†p
ph¥n t‰ch nh÷ tr¶n l duy nh§t sai kh¡c thø tü c¡c nh¥n tß.
T÷ìng tü ta câ k‚t qu£ Łi vîi c¡c i”m nguy¶n nguy¶n tŁ:
ành lþ 2.2. MØi i”m nguy¶n g 6= 0 •u bi”u di„n ÷æc d÷îi d⁄ng
g = :g1
e1
g2
e2
: : : gn
en
trong â, l mºt trong c¡c ìn và, g1; g2; : : : ; gn l c¡c i”m nguy¶n
nguy¶n tŁ ph¥n bi»t (tøc l khæng li¶n k‚t) cÆn e1; e2; : : : ; en l c¡c
sŁ
mô d÷ìng. Ph†p ph¥n t‰ch nh÷ tr¶n l duy nh§t theo ngh¾a: måi ph¥n
e0 e0 e0
t‰ch kh¡c cıa g th nh c¡c nh¥n tß nguy¶n tŁ câ d⁄ng = 0:h11 h22 : : : hnn
vîi bº c¡c sŁ nguy¶n tŁ h1; h2; : : : ; hn kh¡c vîi bº c¡c sŁ nguy¶n tŁ g1;
g2; : : : ; gn b‹ng c¡ch thay Œi c¡c phƒn tß v thay Œi phƒn tß th nh
phƒn tß li¶n k‚t, cÆn bº c¡c sŁ mô ch¿ kh¡c v• thø tü.
Tải tài liệu tại sividoc.com
Viết đề tài giá sinh viên – ZALO:0973.287.149-TEAMLUANVAN.COM
37
Chøng minh. ” chøng minh sü tçn t⁄i cıa ph¥n t‰ch tr¶n ta ti‚n h nh
quy n⁄p theo sŁ N(g) cıa i”m nguy¶n g:
+Ph†p ph¥n t‰ch óng vîi c¡c ìn và vîi chu'n l 1: â l c¡c ìn và 1, i. Gi£
sß N(g) = n v ph†p ph¥n t‰ch óng vîi måi i”m nguy¶n câ chu'n nhä hìn
n. N‚u g l i”m nguy¶n nguy¶n tŁ th… ành lþ ÷æc chøng minh, tr¡i l⁄i th…
g = b:c vîi N(b) > 1; N(c) > 1. V… N(g) = N(b):N(c) ta nh“n ÷æc N(b) <
n; N(c) < n. Theo gi£ thi‚t quy n⁄p, b v c ph¥n t‰ch ÷æc th nh c¡c nh¥n
tß nguy¶n tŁ, ngh¾a l i”m nguy¶n g ÷æc ph¥n t‰ch nh÷ tr¶n.
+ T‰nh duy nh§t ÷æc chøng minh phøc t⁄p hìn v chia l m ba b÷îc
B÷îc 1. X†t bŒ • sau (BŒ • v• ph†p chia câ d÷): Vîi mØi c°p i”m
nguy¶n a 6= 0 v b •u tçn t⁄i i”m nguy¶n c (th÷ìng khæng ı cıa b
cho a) sao cho N(b ca) < N(a).
Th“t v“y, gi£ sß
b = x + iy l th÷ìng cıa b cho a (nh“n ÷æc tł cæng
a
thøc ph†p chia). i”m x + iy khæng nh§t thi‚t l i”m nguy¶n (x ho°c y câ
th” khæng nguy¶n) nh÷ng nâ câ th” l mºt trong c¡c i”m (x; y) tr¶n m°t
phflng Oxy. Ta x†t i”m cƒn t…m c ð l¥n c“n i”m nguy¶n (x; y). Khi â
(x; y) n‹m trong h…nh vuæng ìn và t¥m c, v“y kho£ng c¡ch giœa
c v
b = x + iy nhä hìn 1, v i•u â óng c£ Łi vîi chu'n cıa hi»u
a
cıa chóng(b…nh ph÷ìng kho£ng c¡ch n y), tøc l : N( b) c) < 1. Bði
v“y,
a
a(a c) = N(a)N a c < N(a)
N(b ca) = N
b b
B÷îc 2. ” thu“n ti»n ta °t a = a0; b = a1 v vi‚t d¢y c¡c ph†p chia câ d÷
sau:
a0 = q1a1 + a2 trong â; N(a2) < N(a1)
a
1 = q2a1 + a3 trong â; N(a3) < N(a2)
a
n 2
= q
n 1
a
n 1
+ a
n
;
trong â; N(an) < N(an 1)
a
n
= q
n 1
a
n 1
Tải tài liệu tại sividoc.com
Viết đề tài giá sinh viên – ZALO:0973.287.149-TEAMLUANVAN.COM
38
Qu¡ tr…nh n y k†o d i khi d÷ cıa chu'n kh¡c 0. V… c¡c chu'n n y l c¡c
sŁ nguy¶n, ìn i»u gi£m n¶n qua mºt sŁ hœu h⁄n b÷îc ta nh“n ÷æc d÷
b‹ng 0. Ta kþ hi»u an l d÷ cuŁi còng kh¡c 0. N‚u tł d¢y tr¶n, i
ng÷æc tł d÷îi l¶n tr¶n ta ÷æc:
anjan 1; anjan 2; : : : ; anja0:
v d„ chøng minh ÷æc
N‚u xnja0; xja1 th… xja2; xja3; : : : ; xjan:
Công theo d¢y tr¶n ta th§y c¡c i”m a2; : : : ; a2 bi”u di„n ÷æc d÷îi d⁄ng
aj = xja0 + yja1, °c bi»t:
an = xna0 + yna1: (2.6)
(Nh÷ v“y i”m an l "÷îc chung lîn nh§t" cıa a0 v a1 theo ngh¾a måi
i”m nguy¶n chia h‚t cho a0; a1 ho°c li¶n k‚t vîi an ho°c câ chu'n nhä
hìn N(an)).
Thu“t to¡n t…m ÷îc chung lîn nh§t công ÷æc gåi l thu“t to¡n Euclid. Tł
â d„ d ng chøng minh ÷æc m»nh • "N‚u p l i”m nguy¶n nguy¶n tŁ
th… ab chia h‚t cho p k†o theo a chia h‚t cho p ho°c b chia h‚t cho
p". B÷îc 3. Ta quay l⁄i k‚t thóc ph†p chøng minh ành lþ. N‚u x£y ra
flng thøc p1 : : : pn = q1 : : : qn trong â pi; qi l c¡c i”m nguy¶n nguy¶n
tŁ, sß döng bŒ • câ th” lƒn l÷æt rót gån c¡c nh¥n tß b¶n tr¡i v b¶n
ph£i, suy ra m = n v c¡c nh¥n tß nguy¶n tŁ tròng nhau.
Nh“n x†t 2.1.1. Chó þ r‹ng vi»c tr…nh b y ph†p chøng minh tr¶n
kh¡c vîi ph†p chøng minh ành lþ cì b£n ð chØ ta ph£i dòng so s¡nh
chu'n trong thu“t to¡n Euclid
2.2 ÷íng trÆn tr¶n l÷îi tåa º nguy¶n
B i to¡n °t ra l : H¢y ‚m sŁ i”m nguy¶n N(R) n‹m trong ÷íng
trÆn
K(R) = f(x; y)jx2
+ y2
Rg:
(2.5)
(2.4)
Tải tài liệu tại sividoc.com
Viết đề tài giá sinh viên – ZALO:0973.287.149-TEAMLUANVAN.COM
39
trong â,R 0 công l sŁ nguy¶n. SŁ N(R) b‹ng di»n t‰ch cıa h…nh F
(R) t⁄o th nh tł c¡c h…nh vuæng ìn và cıa l÷îi m ð gâc tr¡i d÷îi n‹m trong
K(R).
H…nh 2.3: F(R) v N(R)
V… kho£ng c¡ch lîn nh§t giœa hai i”m cıa h…nh vuæng ìn và
khæng p
v÷æt qu¡ 2 n¶n t§t c£ c¡c h…nh vuæng giao vîi ÷íng trÆn x2
+y2
=
R2
s›p x‚p trong h…nh v nh kh«n
f(x; y)j(R p 2)2
< x2
+ y2
< (R +
p
2)2
g:
(H…nh v‡ bi”u di„n vîi R = 4)Di»n t‰ch cıa h…nh v nh kh«n b‹ng
(R +
p
p 2)2 = 4
p
R:
2)2
(R 2
v bði v“y, jSF (R) R2
j < 4
p
2R, trong â, SF (R) kþ hi»u di»n t‰ch
N(R)
p
N(R) R
cıa h…nh F. Nh÷ v“y, 2
4 2
. Ngh¾a l , vîi måi R ı lîn ta
R
câ flng thøc gƒn óng . Ch‰nh x¡c hìn câ th” ph¡t bi”u th nh
R2
k‚t qu£ sau:
M»nh • 2.1. (K.Gauss) Vîi c¡c kþ hi»u nh÷ tr¶n ta câ:
lim N(R) = (2.7)
R2
!
R !1
Tải tài liệu tại sividoc.com
Viết đề tài giá sinh viên – ZALO:0973.287.149-TEAMLUANVAN.COM
40
Nh to¡n håc thi¶n t i K. Gauss ¢ x§p x¿ cæng thøc 2.7 b‹ng c¡ch l“p
b£ng t÷ìng øng vîi c¡c gi¡ trà gƒn óng cıa sŁ
R 10 20 30 100 200 300
N(R) 317 1257 2831 31517 125629 282697
3,17 3,1425 3,134 3,1417 3,140725 3,14107
Ph†p chøng minh flng thøc (2.7) g›n li•n vîi mºt trong c¡c t‰nh ch§t
cì b£n cıa l÷îi Z2
: Di»n t‰ch måi h…nh b…nh h nh sinh ra l÷îi Z2
, •u
b‹ng 1. H…nh b…nh h nh nh÷ v“y công ÷æc gåi l h…nh b…nh h nh
cì sð cıa l÷îi Z2
. Nh›c l⁄i r‹ng l h…nh b…nh h nh cì sð cıa l÷îi Z2
n‚u
nâ chia m°t phflng th nh c¡c h…nh b…nh h nh b‹ng nhau, t“p hæp
c¡c ¿nh cıa c¡c h…nh b…nh h nh tròng vîi t“p hæp c¡c nót cıa l÷îi Z2
Gi¡ trà tuy»t Łi cıa hi»u giœa di»n t‰ch h…nh trÆn K(R) v di¶n
t‰ch h…nh F t⁄o tł t“p hæp t§t c£ c¡c h…nh b…nh h nh øng vîi c¡c
nót trong K(R), nhä hìn di»n t‰ch h…nh v nh kh«n
f(x; y)j(R a)2
x2
+ y2
(R + a)2
g:
trong â, a pl ÷íng ch†o lîn nh§t cıa h…nh b…nh h nh (tr¶n h…nh v‡, R = 4;
a = 13). N‚u S = th… SF = N(R) v do â,
j :N(R) R2
j < (R + a)2
(R a)2
= 4a R
N(R)
Ngh¾a l < 4a
. Cho R ti‚n ra væ t“n, theo chøng minh tr¶n
R2
R:
ta nh“n ÷æc: N(R)
= lim = ; tøc l = 1
R2
n!1
Ta cÆn nh“n ÷æc mºt h» qu£ cıa (2.7): ⁄i l÷æng N(R) bi”u di„n sŁ
t§t c£ c¡c c°p sŁ nguy¶n s›p thø tü (x; y) 2 Z2
thäa m¢n x2
+ y2
R2
.
Vîi mØi nót (x; y) 2 Z2
sŁ x2
+y2
l sŁ nguy¶n, bði v“y n‚u kþ hi»u r(k)
l sŁ c¡c c¡ch bi”u di„n ph¥n bi»t cıa sŁ tü nhi¶n k th nh tŒng cıa hai
b…nh ph÷ìng c¡c sŁ nguy¶n (c¡c bi”u di„n k = a2
+ b2
= ( a)2
+ b2
=
a2
+ ( b)2
= ( a)2
+ ( b)2
coi l æi mºt ph¥n bi»t) th…
N(R) = r(0) + r(1) + : : : + r(n); vîi n = R2
:
Tải tài liệu tại sividoc.com
Viết đề tài giá sinh viên – ZALO:0973.287.149-TEAMLUANVAN.COM
41
M»nh • 2.2. (K.Gauss) Ta câ flng thøc sau
lim r(0) + r(1) + : : : + r(n) =
!
n
n!1
Chó þ r‹ng b£n th¥n h m r(n) khæng ch‰nh quy. Chflng h⁄n r(0) = 1;
r(1) = 4; r(2) = 4; r(3) = 0; r(4) = 4; r(5) = 8; r(6) = 0; r(7) = 0; r(8) = 4;
: : : ; r(21) = 0; r(22) = 0; r(23) = 0; r(24) = 0; r(25) = 12.
2.2.1 SŁ c¡c bi”u di„n cıa mºt sŁ tü nhi¶n th nh tŒng hai b…nh
ph֓ng
” t…m sŁ c¡c ph¥n t‰ch n = a2
+ b2
vîi n 2 N tòy þ ta câ k‚t qu£ sau
M»nh • 2.3. Vîi måi n 2 N, sŁ c¡c i”m nguy¶n trong ÷íng trÆn p
b¡n k‰nh n b‹ng sŁ c¡c c¡ch ph¥n t‰ch sŁ nguy¶n n th nh tŒng hai
b…nh ph÷ìng, óng b‹ng sŁ r(n) x¡c ành nh÷ sau:
N‚u n khæng chia h‚t cho 2 th… r(n) = 4[d1(n) d3(n)], vîi d1(n); d3(n)
l c¡c ÷îc cıa n thø tü câ d⁄ng 4n + 1; 4n + 3;
N‚u n = 2 ps
11
ps
22
: : : ps
kk
:q1
t1
: : : te
ll
l ph¥n t‰ch ch‰nh t›c cıa n
th nh c¡c nh¥n tß nguy¶n tŁ vîi pi câ d⁄ng 4k + 1; qj câ d⁄ng 4k + 3 th…
r(n) =
(
4(s + 1) : : : (s + 1) khi t ; : : : ; t •u chfin
0 1 k trong1
tr֒ngl
hæp ng÷æc l⁄i.
Tr¶n h…nh v‡ 2.4:
H…nh 2.4: r(4)=4;r(25)=12
Tải tài liệu tại sividoc.com
Viết đề tài giá sinh viên – ZALO:0973.287.149-TEAMLUANVAN.COM
42
Vîi n = 4, ta câ 4 c¡ch ph¥n t‰ch
22
=02
+22
=22
+02
=02
+( 2)2
= ( 2)2
+ 02
:
Vîi n = 25 ta câ 12 c¡ch ph¥n t‰ch 52
= ( 5)2
+02
= ( 4)2
+ ( 3)2
=
( 3)2
+( 4)2
=(0)2
+( 5)2
=32
+( 4)2
=42
+( 3)2
=02
+52
=
42
+32
= 32
+42
=02
+52
=( 3)2
+42
=( 4)2
+(3)2
.
Tł ành lþ 2.3 ta rót ra: Ph÷ìng tr…nh x2
+ y2
= 5k
; (k 0) câ 4(k + 1)
k
nghi»m nguy¶n, nâi c¡ch kh¡c, ÷íng trÆn vîi b¡n k‰nh 52 , t¥m ð gŁc
tåa º s‡ i qua óng 4(k + 1) nót cıa l÷îi Z2
.
2.2.2 ÷íng trÆn Sinhsel
Tr…nh b y phƒn n y düa theo [2]. ƒu ti¶n ta câ nh“n x†t:Vîi måi
p 1
sŁ tü nhi¶n n tçn t⁄i h…nh trÆn vîi t¥m t⁄i i”m ( 2; ) chøa trong nâ
3
óng n i”m nguy¶n cıa l÷îi.
” chøng minh nh“n x†t n y ta h¢y chøng tä: n‚u (x1; y1); (x2; y2) l
hai nót ph¥n bi»t cıa l÷îi th… kho£ng c¡ch tł chóng ‚n i”m (
p
;1 )
2
p 2 1 2 p 2
3
khæng b‹ng nhau. Th“t v“y, n‚u (x1 2) + (y1 ) = (x2 2) +
1 2 p
3
2 2 2 2 2 1
(y2 ) th… x1 x2 + y1 y2 y 2 2(x1 + x2) = 0. Tł â rót ra:
3
3
2 2
x1 = x2 v y2
2
y1
2
y1 y2 = 0. flng thøc thø hai t÷ìng ÷ìng
3 3
y1
1 2
= y2
1 2
() 3y1 1 = (3y2 1):
3 3
Tøc l ho°c y1 = y2 ho°c 3(y1 + y2) = 2, nh÷ng flng thøc sau khæng
x£y ra. Nh÷ v“y, x1 = x2; y1 = y2. Ta l§y d¢y c¡c b¡n k‰nh R1 < R2 <
: : : < Rn < : : :, n‚u Rn < R < Rn+1 th… ÷íng trÆn t¥m p 2; 3 , b¡n
1
k‰nh R chøa trong nâóng n i”m nguy¶n.
C¥u häi cƒn quan t¥m v khâ hìn l : vîi sŁ n cho tr÷îc düng ÷æc
hay khæng ÷íng trÆn i qua n i”m nót l÷îi?.
D„ t…m ÷æc ÷íng trÆn i qua 1,2,3 v 4 i”m. Công d„ ÷a ra c¡c v‰
dö vîi n = 8; n = 12 (H…nh v‡ 2.5, 2.6). t tƒm th÷íng hìn l tr÷íng
Tải tài liệu tại sividoc.com
Viết đề tài giá sinh viên – ZALO:0973.287.149-TEAMLUANVAN.COM
43
hæp n = 6 (H…nh v‡ 2.5). Tuy nhi¶n vîi n = 5; 7; 17; : : : th… vi»c
düng ÷íng trÆn khæng h• ìn gi£n.
M»nh • 2.4. (A. Singsel) Vîi måi sæ tü nhi¶n n •u tçn t⁄i ÷íng trÆn i
qua óng n nót l÷îi
Chøng minh. K‚t qu£ thø hai trong (2.3) cho ph†p k‚t lu“n r‹ng: vîi
måi n •u düng ÷æc ÷íng trÆn m tr¶n â câ óng 4n nót l÷îi. ”
l m i•u â ch¿ cƒn l§y t¥m ÷íng trÆn l gŁc tåa º cÆn b¡n k‰nh chån
l R = 5(k 1)=2
(Xem chó þ sau ành lþ 2.3).
H…nh 2.5: n=8, n=6
H…nh 2.6: n=12
Tải tài liệu tại sividoc.com
Viết đề tài giá sinh viên – ZALO:0973.287.149-TEAMLUANVAN.COM
44
H…nh v‡ 2.5 vîi 6 i”m n‹m tr¶n ÷íng trÆn gæi þ cho ta suy ngh¾ x†t
÷íng trÆn t¥m l i”m
1
; 0 . N‚u l§y b¡n k‰nh l sŁ R =
5(k 1)=2
2 2
th… ph÷ìng tr…nh ÷íng trÆn câ d⁄ng
1 2 5k 1
x + y2
= (2.8)
2 4
hay
(2x 1)2
+ (2y)2
= 5k
1 (2.9)
Công nh÷ chó þ ð tr¶n, ph÷ìng tr…nh
a2
+ b2
= 5k 1
câ 4 nghi»m. Rª r ng trong ph÷ìng tr…nh (2.10) mºt trong c¡c sŁ a; b ph£i
chfin, sŁ kia l·. Trong ph÷ìng tr…nh (2.9), t‰nh ch§t cıa mØi sŁ h⁄ng
l cŁ ành v… v“y mØi nghi»m trong hai nghi»m (a; b); (b; a) cıa (2.10)
nh“n ÷æc óng mºt nghi»m cıa (2.9). Nh÷ v“y ph÷ìng tr…nh (2.9) câ óng
2k nghi»m ( b‹ng mºt nßa sŁ nghi»m cıa ph÷ìng tr…nh (2.10)).
Nh÷ v“y ta câ th” düng ÷æc ÷íng trÆn i qua n = 2k i”m. Chflng
h⁄n vîi 10 i”m tr¶n l÷îi, ch¿ vi»c düng ÷íng trÆn (2.9) øng vîi k = 5.
T§t nhi¶n khæng th” l§y i”m
1 ; 0 l m t¥m ÷íng trÆn khi sŁ n
2
l·, n = 2k + 1. Tr÷íng hæp n y ta s‡ düng ÷íng trÆn t¥m
1
; 0 , b¡n
3
k‰nh R =5k
: (x 1 )2
+ y2
= 52k
hay
3 3 9
(3x 1)2
+ (3y)2
= 52k
: (2.11)
Nh÷ng k‚t lu“n cıa m»nh • (2.3), ph÷ìng tr…nh
a2
+ b2
= 52k
(2.12)
câ 4(2k+1) nghi»m. B‹ng c¡ch x†t d÷ cıa ph†p chia cho 3 (b…nh
ph÷ìng cıa sŁ nguy¶n khi chia cho 3, câ d÷ l 0 v 1), ta rót ra mºt trong
c¡c sŁ a; b chia h‚t cho 3, sŁ cÆn l⁄i khæng chia h‚t cho 3. Gi£ sß a
0(mod3); b 1(mod3). Khi â, trong 4 c°p (a; b); (a; b); (b; a); ( b; a)
(2.10)
Tải tài liệu tại sividoc.com
Viết đề tài giá sinh viên – ZALO:0973.287.149-TEAMLUANVAN.COM
45
ch¿ câ óng mºt c°p l nghi»m cıa ph÷ìng tr…nh (2.11). Do â sŁ
nghi»m cıa nâ ‰t hìn 4 lƒn sŁ nghi»m cıa ph÷ìng tr…nh (2.12), tøc l
(2.11) câ 2k + 1 nghi»m. Chflng h⁄n vîi k = 2 tøc n = 5, sau khi düng
÷íng trÆn (3x 1)2
+ (3y)2
= 54
, ta ÷æc 5 i”m nguy¶n tr¶n nâ:
(7,5),(7,-5),(-2,8),(-8,0),(-2,-8).
C¡c ph÷ìng tr…nh (2.8), (2.11) ÷æc gåi l c¡c ÷íng trÆn Singsel.
Chó þ r‹ng vîi sŁ n cho tr÷îc c¡c ph÷ìng tr…nh n y ch÷a ch›c ¢ cho
÷íng trÆn nhä nh§t i qua n i”m cıa l÷îi. Chflng h⁄n vîi n = 4 câ
p 2
th” l§y b¡n k‰nh ÷íng trÆn l , vîi n = 9, ÷íng trÆn Sinhsel câ
2
b¡n k‰nh 625 nh÷ng ÷íng trÆn vîi t¥m ( 1; 0), b¡n k‰nh 65 công i
3
3 3
qua 9 nót cıa l÷îi.
C¡c b i to¡n kh¡c:
B i to¡n 2.1. Chøng minh r‹ng vîi mØi sŁ tü nhi¶n n tçn t⁄i qu£ cƒu
p p 1
) chøa trong nâ óng n
i”m cıa l÷îi Z3
.t¥m(2;3;
3
B i to¡n 2.2. Gi£ sß ÷íng trÆn (x x0)2
+ y2
= R
2
l ÷íng trÆn
Singsel i qua óng n i”m nguy¶n cıa Z2. Chøng minh r‹ng tr¶n m°t p
0
B i to¡n 2.3. Chøng minh r‹ng vîi måi sŁ tü nhi¶n n •u tçn t⁄i ÷íng
trÆn i qua óng n i”m cıa m°t phflng thuºc l÷îi tam gi¡c •u; l÷îi löc gi¡c
•u.
B i to¡n 2.4. Chøng minh r‹ng n‚u ‰t nh§t mºt tåa º cıa t¥m ÷íng
trÆn l sŁ væ t th… tr¶n ch‰nh ÷íng trÆn t…m ÷æc khæng qu¡ hai
i”m câ tåa º hœu t .
B i to¡n 2.5. Chøng minh r‹ng
a. Tçn t⁄i ÷íng trÆn câ t¥m t⁄i nót l÷îi nguy¶n m tr¶n nâ khæng câ
i”m hœu t n o.
b. Tçn t⁄i ÷íng trÆn m tr¶n nâ chøa óng mºt i”m hœu t .
c. Tçn t⁄i ÷íng trÆn m tr¶n nâ chøa óng hai i”m hœu t .
Tải tài liệu tại sividoc.com
Viết đề tài giá sinh viên – ZALO:0973.287.149-TEAMLUANVAN.COM
46
d. N‚u ÷íng trÆn t¥m ð gŁc tåa º (0,0) i qua ‰t nh§t mºt i”m hœu t
th… tr¶n ÷íng trÆn â s‡ chøa væ sŁ c¡c i”m hœu t cıa m°t
phflng.
B i to¡n 2.6. Tr¶n gi§y k· æ, k‰ch th÷îc mØi æ l 1 1 v‡ ÷íng trÆn
b¡n k‰nh R, t¥m l i”m nót. Chøng minh r‹ng n‚u tr¶n ÷íng trÆn câ
p
1988 i”m nót cıa l÷îi th… ho°c R ho°c R 2 l sŁ nguy¶n.
B i to¡n 2.7. Chøng minh r‹ng vîi måi n chfin, tçn t⁄i ÷íng trÆn t¥m
(
1
3; 0) i qua óng n nót cıa l÷îi Z2
.
2.3 Mºt sŁ øng döng v o sŁ håc
2.3.1 Bº ba Pythagoras
Bº ba sŁ tü nhi¶n (a; b; c) mØi sŁ l º d i c⁄nh tam gi¡c vuæng, tøc
l thäa m¢n i•u ki»n cıa ành lþ Pythagore a2
+ b2
= c2
. Ta s‡ sß
döng ành lþ cì b£n v• i”m nguy¶n ” t…m t§t c£ c¡c bº ba â.
Ta câ c2
= (a + bi)(a bi). Ph¥n t‰ch c v a + bi th nh c¡c nh¥n tß
nguy¶n tŁ:
c = pe1
pe2
: : : pe
n ; a + bi = ql1
ql2
: : : ql
m sk1
sk2
: : : skl
; qi = qi; si 6=
si 1 2 n 1 2 m 1 2 l
= ( q1
l1 q2
l2 : : : qm
l
m ):(sk
11 (sk
22 (sk
ll ):( :q1
l
1 q2
l
2 : : : qm
l
m ):(q1
l
1 s2
k
2 : : : sl
k
l ) =
= :q1
2l1
q2
2l2
: : : qm
2l
m :sk
11
sk
22
: : : sk
ll
:s1
k
1 s2
k
2 : : : sl
k
l . Tł ¥y
ki = 2ri. Bði v“y, c = d(u+iv)(u iv) vîi d = q1
2l1
q2
2l2
: : : qm
2l
m ; (u+iv) =
sr
11
r2
k2
: : : sr
ll
. Khi â, c2
= d2
(u + iv)2
(u + iv)2
= [d(u + iv)]2
:[d(u +
iv)]2
. ‚n ¥y sß döng sü ph¥n t‰ch duy nh§t th nh c¡c nh¥n tß nguy¶n
tŁ( ành lþ cì b£n) ta ÷æc:
a + bi = d(u + iv)2
= d(u2
v2
) + (2duv)i
Tł â suy ra:
M»nh • 2.5. MØi bº ba Pythagore bi”u di„n ÷æc d⁄ng
a = d(u2
v2
); b = 2duv; c = d(u2
+ v2
); u; v 2 N:
Tải tài liệu tại sividoc.com
Viết đề tài giá sinh viên – ZALO:0973.287.149-TEAMLUANVAN.COM
47
2.3.2 C¡c d⁄ng cıa i”m nguy¶n nguy¶n tŁ
Mºt øng döng thø hai quan trång l ta câ th” mæ t£ t§t c£ c¡c i”m
nguy¶n nguy¶n tŁ.
M»nh • 2.6. C¡c i”m nguy¶n nguy¶n tŁ câ mºt trong ba d⁄ng sau:
(A) 1+i v c¡c i”m li¶n k‚t vîi nâ,
(B) C¡c sŁ nguy¶n tŁ thæng th÷íng d⁄ng p=4k+3 v c¡c li¶n k‚t,
(C) Nh¥n tß a+bi cıa sŁ nguy¶n tŁ p thæng th÷íng d⁄ng 4k + 1; k 2 N
Chøng minh. X†t i”m nguy¶n nguy¶n tŁ q.
BŒ •: i”m nguy¶n nguy¶n tŁ q l ÷îc cıa óng mºt sŁ nguy¶n tŁ p. Th“t
v“y, ta luæn câ qjN(q) n¶n q chia h‚t mºt nh¥n tß nguy¶n tŁ n o
â cıa N(q). M°t kh¡c q khæng th” chia h‚t 2 sŁ nguy¶n tŁ ph¥n bi»t l
v r v… ta luæn t…m ÷æc c¡c sŁ nguy¶n m v n sao cho ml + rn = 1.
BŒ • ÷æc chøng minh.
Nh÷ v“y, i”m nguy¶n tŁ q cho tr÷îc luæn luæn chia h‚t mºt sŁ
nguy¶n tŁ p(thæng th÷íng). Tøc l q = a + bijp hay q:t = p, vîi t 2 Z2
.
Ta suy ra:N(q)N(t) = N(p) = p2
. Bði v“y câ hai kh£ n«ng
"
N(q) = a2 + b2 = p;
t l
t = q = a bi (b)
N(t) = 1; tøc l ìn và v q li¶n k‚t vîip (a)
B¥y gií ta x†t 3 tr÷íng hæp Łi vîi sŁ nguy¶n tŁ p:
N‚u p = 2 = 12
+ 12
= (1 + i)(1 i) = i(1 i2
). Khi â, c¡c i”m
nguy¶n
1 + i = (1; 1); 1 i = (1; 1); 1 + i = ( 1; 1); 1 i = ( 1; 1) l
c¡c i”m li¶n k‚t vîi nhau. Ta câ d⁄ng (A).
N‚u p = 4k + 3, flng thøc trong (b) khæng x£y ra v… b…nh ph÷ìng
cıa mºt sŁ nguy¶n b§t ký khi chia cho 4 câ d÷ l 0 ho°c 1. Bði v“y sŁ
nguy¶n tŁ d⁄ng 4k + 3 l i”m nguy¶n tŁ. Ta câ d⁄ng (B).
N‚u p = 4k + 1. Sß döng bŒ • Eluler: "N‚u p = 4k + 1; k 2 Z l sŁ
nguy¶n tŁ th… tçn t⁄i x 2 N sao cho pjx2
+ 1, tøc pj(x + i):(x i)" th…
trong tr÷íng hæp cıa ta p khæng th” l i”m nguy¶n nguy¶n tŁ. Ta suy
Tải tài liệu tại sividoc.com
Viết đề tài giá sinh viên – ZALO:0973.287.149-TEAMLUANVAN.COM
48
ra p ph£i chia h‚t cho mºt i”m nguy¶n n o â g = (a + bi). Tł ¥y
. .
. .
(c + di) khæng li¶n
theo ành lþ cì b£n p.(a bi). N‚u câ p.(c + di) m
.
.
k‚t vîi a + bi th… p.(c di). Suy ra:
. 2 2 2 2
.
di) = (a )(c ). Tr¡i vîi gi£ thi‚t
p.(a + bi)(a bi)(c + bi)(c + b + d
p l sŁ nguy¶n tŁ. Ngh¾a l
p = (a + bi):(a bi) = a2
+ b2
= N(g): (2.13)
M»nh • ÷æc chøng minh ho n to n.
Nºi dung ch÷ìng 2 l " ‚m" sŁ i”m nguy¶n tr¶n ÷íng trÆn v trong
÷íng trÆn. Phƒn øng döng v o sŁ håc cho ta mºt sŁ k‚t qu£ nh÷ bº
ba Pythagoras, c¡c d⁄ng cıa i”m nguy¶n nguy¶n tŁ,: : : cho th§y sü
g›n k‚t, h i hÆa giœa sŁ håc v h…nh håc.
Tải tài liệu tại sividoc.com
Viết đề tài giá sinh viên – ZALO:0973.287.149-TEAMLUANVAN.COM
49
Ch֓ng 3
C¡c b i to¡n kh¡c
3.1 i”m nguy¶n tr¶n ÷íng cong phflng
Phı mºt l÷îi tåa º nguy¶n l¶n m°t phflng ta x†t mºt ÷íng cong tr¶n
l÷îi v °t b i to¡n t…m c¡c i”m nguy¶n m ÷íng cong i qua. C¡c i”m n y
câ th” rØng, hœu h⁄n ho°c væ h⁄n. Thüc ch§t b i to¡n l gi£i ph÷ìng
tr…nh nghi»m nguy¶n, tuy nhi¶n x†t tr¶n l÷îi b i to¡n s‡ câ trüc quan
h…nh håc hìn. Ph÷ìng ph¡p gi£i c¡c b i to¡n n y sß döng c¡c ki‚n thøc
v• sŁ håc c¡c sŁ nguy¶n. Sau ¥y l c¡c v‰ dö.
V‰ dö 3.1.1. Chøng minh r‹ng câ væ sŁ c¡c i”m nguy¶n n‹m tr¶n
Hypecbol (H): x2
2y2
= 1.
p p
Líi gi£i.Ta câ x2
2y2
= 1 () (x + y 2)(x y 2) = 1. Ta câ ngay
i”m (x1; y1) = (3; 2) l i”m n‹m tr¶n Hypecbol.
Vîi n 1, ta °t xn+1 = 3xn + 4yn; yn+1 = 2xn + 3yn. B‹ng c¡ch
quy n⁄p chøng minh ÷æc vîi måi n
(x1 + y1
p n p n p
2) = (3 + 2 2) = xn + yn 2;
p p p :
(x1 y1 2)n
= (3 2 2)n
= xn yn 2
p p p n p n
, t֓ng ֓ng
Suy ra (xn + yn 2)(xn yn 2) = (3 + 2 2) (3 2 2)
vîi xn
2
2yn
2
= (9 8)2
= 1. Nh÷ v“y ngo i i”m (3,2) ta t…m ÷æc væ
sŁ i”m nguy¶n Xn(xn; yn) n‹m tr¶n Hypecbol x2
2y2
= 1.
V‰ dö 3.1.2. (IMO-2006,#4) T…m t§t c£ c¡c c°p sŁ nguy¶n (x,y) sao
cho 1 + 2x
+ 22x+1
= y2
.
Tải tài liệu tại sividoc.com
Viết đề tài giá sinh viên – ZALO:0973.287.149-TEAMLUANVAN.COM
50
Chøng minh. B i to¡n câ th” ph¡t tri”n nh÷ sau: T…m t§t c£ c¡c i”m
nguy¶n x; y) n‹m tr¶n ÷íng cong 1 + 2x
+ 22x+1
= y2
.
N‚u (x; y) l nghi»m th… hi”n nhi¶n x 0 v (x; y) công l nghi»m.
Vîi x = 0 ta câ hai nghi»m (0; 2) v (0; 2).
Gi£ sß (x; y) l nghi»m vîi x > 0. Khæng m§t t‰nh ch§t tŒng qu¡t
ta coi y > 0. Vi‚t l⁄i ph÷ìng tr…nh nh÷ sau: 2x
(1 + 2x+1
= (y 1)(y + 1
chøng tä y 1 v y + 1 l chfin, ch‰nh x¡c l mºt trong chóng chia h‚t cho
4. Do â x 3 v mºt trong c¡c nh¥n tß n y chia h‚t cho 2x 1
nh÷ng
khæng chia h‚t cho 2x
. V“y
y = 2x 1
m + ; m l·; = 1: (3.1)
Thay v o ph÷ìng tr…nh ¢ cho thu ÷æc:
2x
(1 + 2x+1
) = (2x 1
m + )2
1 = 22x 2
+ 2x
m (3.2)
Vîi = 1 k†o theo m2
8 0, tøc m = 1, khæng thäa m¢n (3.4).
Vîi = 1, ph÷ìng tr…nh (3.4) cho ta
1 + m = 2x 2
(m2
8) 2(m2
8):
K†o theo 2m2
m 17 0. Tł â, m 3. M°t kh¡c theo (3.4), m 6= 1. V… m
l· n¶n m = 3, cho ta x = 4. Tł (3.3) ta câ y = 23. Ki”m tra l⁄i ta câ c¡c
gi¡ trà n y thäa m¢n ph÷ìng tr…nh. Nhî r‹ng y = 23 công thäa m¢n.
Nh÷ v“y, ta câ c¡c i”m nguy¶n (0; 2); (0; 2); (4; 23); (4; 23) tr¶n ÷íng
cong ¢ cho.
V‰ dö 3.1.3. (IMO-1997,#5) T…m t§t c£ c¡c c°p sŁ nguy¶n (a,b) vîi
a; b 1 thäa m¢n ph÷ìng tr…nh ab2
= ba
.
Chøng minh. B i to¡n câ ph¡t tri”n nh÷ sau: T…m t§t c£ c¡c i”m
nguy¶n (x; y) vîi x; y 1 n‹m tr¶n ÷íng cong xy2
= yx
Tr÷îc h‚t chó þ r‹ng: n‚u ta câ am
= bn
th… luæn tçn t⁄i c n o â sao
cho a = ce
; b = cf
vîi m = f d; n = ed v d l ÷îc chung lîn nh§t cıa m v n
(Chøng minh b‹ng c¡ch ph¥n t‰ch a; b ra thła sŁ nguy¶n tŁ).
Trong tr÷íng hæp b i to¡n, gi£ sß d l ÷îc chung lîn nh§t cıa a v b2
v
ta °t a = de; b2
= df. Khi â theo chó þ tr¶n, a = ce
; b = cf
vîi
Tải tài liệu tại sividoc.com
Viết đề tài giá sinh viên – ZALO:0973.287.149-TEAMLUANVAN.COM
51
c n o â. Do â, f ce
= ec2f
. Khæng th” x£y ra e = 2f v… khi â, gi£n ÷îc
hai v‚ cho e = f. Væ lþ.
Gi£ sß 2f > e th… f = ec2f e
. Suy ra e = 1 v f = c2f 1
. N‚u c = 1
th… f = 1 v ta câ nghi»m a = b = 1. N‚u c 2 th… c2f 1
2f
> f, væ
nghi»m.
CuŁi còng, gi£ sß 2f < e th… e = f:ce 2f
. Tł â, f = 1 v e = ce 2
:ce 2
2e 2
e vîi e 5 n¶n ta ph£i câ e = 3 ho°c e = 4(e > 2f = 2). Gi¡ trà e = 3
cho nghi»m a = 27; b = 3 cÆn gi¡ trà e = 4 cho nghi»m a = 16; b = 2.
Ta thu ÷æc 3 i”m nguy¶n (1; 1); (16; 2); (27; 3).
3.2 Mºt sŁ to¡n thi håc sinh giäi v thi Olympic
Ta s‡ x†t øng döng cıa l÷îi tåa º v o mºt sŁ b i to¡n thi håc sinh giäi
v thi Olympic quŁc t‚, mð ƒu l b i to¡n thi Olympic To¡n quŁc t‚ n«m
2016. B i to¡n • c“p ‚n mºt a gi¡c nguy¶n, c¡c ¿nh n‹m tr¶n ÷íng trÆn
v câ t‰nh ch§t °c bi»t. ” gi£i nâ, ki‚n thøc dòng ‚n kh¡ quen thuºc: i”m
nguy¶n, chia h‚t c¡c sŁ nguy¶n, cæng thøc Picard,: : : vîi sü t÷ duy
r§t linh ho⁄t.
V‰ dö 3.2.1. (IMO.2016,#3) Cho P = A1A2 : : : Ak l mºt a gi¡c lçi
trong m°t phflng. C¡c ¿nh câ tåa º nguy¶n v n‹m tr¶n mºt ÷íng trÆn.
Mºt sŁ tü nhi¶n n l· thäa m¢n b…nh ph÷ìng º d i mØi c⁄nh cıa P •u
chia h‚t cho n. Gåi S l di»n t‰ch cıa P, chøng minh 2S l sŁ tü nhi¶n
chia h‚t cho n.
Chøng minh. Khæng m§t t‰nh ch§t tŒng qu¡t, gi£ sß a gi¡c P l a
gi¡c khæng câ ÷íng ch†o n o m b…nh ph÷ìng º d i chia h‚t cho n v…
n‚u tr¡i l⁄i ta s‡ chia a gi¡c th nh hai a gi¡c câ sŁ c⁄nh nhä hìn.
+ Ta x¡c ành vp(x) = maxk 2 Zjpk
jx vîi måi x 2 Q(pk
jx (= 9q 2
Z; x = qpk
, v‰ dö v3(81) = 4; v5( 1 ) = 3). Nh›c l⁄i r‹ng måi tam
125
gi¡c ABC vîi c⁄nh
p
a;
p
b;
p
, di»n t‰ch S = S(ABC) v b¡n k‰nh
c
÷íng trÆn ngo⁄i ti‚p R, ta câ hai cæng thøc:
abc = 4(2S)2
R2
: (3.3)
Tải tài liệu tại sividoc.com
Viết đề tài giá sinh viên – ZALO:0973.287.149-TEAMLUANVAN.COM
52
4(2S)2
= 2ab + 2bc + 2ca a2
b2
c2
: (3.4)
Vîi måi a gi¡c nguy¶n P, ành lþ Picard cho ta
S(F ) = n + m 1
2
vîi n v m l sŁ c¡c i”m nguy¶n ð trong v tr¶n c⁄nh a gi¡c P. Do
â, 2S(P) = 2n + m 2 2 Z+
. Ta câ Ai l c¡c i”m nguy¶n n¶n AiAj
2
l sŁ nguy¶n. Ta ch¿ cƒn chøng minh b i to¡n Łi vîi tr÷íng hæp n l
lôy thła cıa mºt sŁ nguy¶n tŁ l· p. °t n = pm
v do â gi£ thi‚t l
pm
jAiAj
2
vîi måi i; j 2 1; 2; : : : ; n v cƒn chøng minh pm
j2S(P).
a. Tr÷íng hæp k = 3. Gi£ sß
p
p = A2A3;
p
= A3A1
= A1A2; b
a c
sao cho pm
ja; b; c. Theo cæng thøc (3.4),
4(2S )2
= 2ab + 2bc + 2ca a2
b2
c2
:
M
Måi sŁ h⁄ng b¶n ph£i chia h‚t cho p2m
, do â, p2m
j4(2SM)2
, k†o theo
pm
j2SM.
b. Tr÷íng hæp k 4. Ta quy n⁄p theo k: Gi£ sß b i to¡n ÷æc chøng
minh Łi vîi (k 1 gi¡c, ta i chøng minh nâ óng vîi måi k gi¡c.
p p p m
ja; b. Vîi SM l di»n
x = A3A1 vîi p
°t a = A1A2; b = A2A3;
t‰ch tam gi¡c A1A2A3, O l t¥m ÷íng trÆn i qua c¡c ¿nh a gi¡c P. N‚u
pm
jx th… M A1A2A3 v a gi¡c Q = A1A3A4 : : : Ak l tam gi¡c
v (k 1) gi¡c thäa m¢n gi£ thi‚t quy n⁄p, nh÷ ¢ ch¿ ra ð tr¶n pm
j2SM;
pm
j2SQ. Do â: pm
j2SM + 2SQ = 2S nh÷ y¶u cƒu.
B¥y gií, gi£ sß pm
khæng chia h‚t x. Khi â, c = vp(c) < m. Theo
cæng thøc (3.4) ta câ
4(S)2
= 2ab + 2ax + 2bx a2
b2
x2
Lôy thła cao nh§t cıa p khi chia cho x2
nhä hìn (nghi¶m ng°t) lôy thła
cao nh§t cıa p khi chia cho sŁ h⁄ng b§t ký trong 5 sŁ h⁄ng cıa v‚ ph£i
flng thøc tr¶n n¶n suy ra:
vp((2S)2
) = vp( x2
) = 2c:
B¥y gií cæng thøc (3.3) cho ta: abx = 4(2S)2
R2
, do â, R2
l sŁ hœu t
v câ th” l§y gi¡ trà øng vîi lôy thła cao nh§t cıa p chia cho R2
nh÷
Tải tài liệu tại sividoc.com
Viết đề tài giá sinh viên – ZALO:0973.287.149-TEAMLUANVAN.COM
53
thæng th÷íng. Theo t‰nh ch§t cıa vp(x) ta câ:
vp(a) + vp(b) + c = vp((2S)2
) + vp(R2
) = 2c + vp(R2
):
m i•u â k†o thoe vp(R2
) = vp(a) + vp(b) c 2m c > m. Do â, pm
jR2
(theo ành ngh¾a cıa vp(R2
)).
X†t M AiOAi+1 vîi i = 1; 2; : : : ; k. °t AiAi+1 =
p
y thäa m¢n pm
jy.
L⁄i theo cæng thøc (3.4) ta câ:
4(2SMAiOAi+1
)2
= 2R2
y + 2R2
R2
R4
R4
y2
= y(4R2
y)
V… pm
jy; pm
jR2
n¶n pm
j2SMAiOAi+1 vîi måi i. TŒng c¡c sŁ h⁄ng
2SMAiOAi+1 ( nh÷ c¡c di»n t‰ch tam gi¡c ành h÷îng) vîi i = 1; 2; : : : ;
k quy v• 2S. Suy ra pm
j2S.
B…nh lu“n
Ta câ th” x†t th¶m tr÷íng hæp P l tø gi¡c b‹ng c¡ch dòng ành lþ
Ptolemy coi nh÷ mºt b i to¡n nhä.
Theo nh÷ nhi•u ºi tuy”n Olympic to¡n QuŁc t‚ nh“n x†t: ¥y l mºt b i
to¡n khâ nh§t trong • thi n«m 2016, do Nga • nghà. Thüc ch§t l b
i to¡n sŁ håc nh÷ng ng÷íi gi£i cƒn hi”u bi‚t s¥u s›c v• c¡c cæng
thøc, ành lþ h…nh håc ¡p döng tr¶n l÷îi tåa º nguy¶n.
Sau ¥y l mºt v‰ dö nœa li¶n quan d‚n l÷îi æ vuæng:
V‰ dö 3.2.2. (IMO-1996,#1) Cho sŁ d÷ìng r v mºt b£ng h…nh chœ
nh“t ABCD vîi AB=20,BC=12. H…nh chœ nh“t ÷æc chia th nh 20 12
ævuæng. X†t ph†p dàch chuy”n sau: Dàch chuy”n tł æ vuæng n y sang
ævuæng kia ch¿ ÷æc thüc hi»n n‚u kho£ng c¡ch giœa hai t¥m æ vuæng p
b‹ng r. Nhi»m vö °t ra l t…m mºt d¢y c¡c ph†p dàch chuy”n i tł æ
vuæng câ A l ¿nh ‚n æ vuæng câ B l ¿nh.
a. Chøng minh r‹ng nhi»m vö l b§t kh£ thi n‚u r chia h‚t cho 2 ho°c
chia h‚t cho 3.
b. Chøng minh r‹ng nhi»m vö l kh£ thi n‚u r = 73.
c. Nhi»m vö câ thüc hi»n ÷æc khæng n‚u r = 97?
Tải tài liệu tại sividoc.com
Viết đề tài giá sinh viên – ZALO:0973.287.149-TEAMLUANVAN.COM
54
Chøng minh. (a) Gi£ sß dàch chuy”n a ìn và theo mºt h÷îng v b ìn và
theo h÷îng vuæng gâc, khi â a2
+ b2
= r. N‚u r chia h‚t cho 2 th… c£
a v b •u chfin ho°c •u l·. Nh÷ng i•u â ngh¾a l ta ch¿ câ th” câ c¡c æ
en ho°c æ tr›ng (gi£ sß c¡c æ ÷æc tæ m u nh÷ tr¶n b n cí vua). Hai
gâc h…nh chœ nh“t còng mºt m u n¶n nhi»m vö khæng thüc hi»n
־c.
X†t ph†p chia c¡c sŁ a; b; r cho 3: n‚u r chia h‚t cho 3 th… a v b c£
hai •u chia h‚t cho 3. i•u â ngh¾a l n‚u b›t ƒu tł æ câ tåa º (0,0) th…
ta ch¿ câ th” dàch chuy”n tîi æ câ tåa º d⁄ng (3m; 3n). i”m Æi häi câ
tåa º (19,0) khæng ð d⁄ng â n¶n nhi»m vö khæng thüc hi»n ÷æc.
(b) N‚u r = 73 th… ta câ a = 8; b = 3 (ho°c ng÷æc l⁄i). Câ 4 ki”u dàch
chuy”n:
A : (x; y) 7! (x + 8; y + 3)
B : (x; y) 7! (x + 3; y + 8)
C : (x; y) 7! (x + 8; y 3)
D : (x; y) 7! (x + 3; y 8)
Ta l§y (x 8; y 3) 7! (x; y) trong ph†p dàch chuy”n ki”u A,v.v: : : th…
khi câ a ph†p dàch chuy”n ki”u A, b ph†p dàch chuy”n ki”u B v v.v: : :
s‡ câ c¡c ph÷ìng tr…nh sau:
8(a + c) + 3(b + d) = 19; 3(ac) + 8(b d) = 0
Mºt nghi»m d„ th§y l a = 5; b = 1; c = 3; d = 2. B›t ƒu vîi nghi»m n y,
b‹ng mºt sŁ t‰nh to¡n ta t…m ÷æc c¡c b÷îc thüc hi»n nhi»m
vö (0; 0) 7! (8; 3) 7! (16; 6) 7! (8; 9) 7! (11; 1) 7! (19; 4) 7! (11; 7) 7!
(19; 10) 7! (16; 2) 7! (8; 5) 7! (16; 8) 7! (19; 0).
(c) N‚u r = 97 ta cƒn câ a = 9; b = 4. GiŁng nh÷ tr¶n, gi£ sß ta b›t ƒu
ð (0,0). B‹ng c¡ch lþ lu“n "chfin, l·" ta rót ra nhi»m vö l b§t kh£
thi.
V‰ dö 3.2.3. (IMO-1997,#1) Trong m°t phflng c¡c i”m vîi tåa º
nguy¶n l c¡c ¿nh cıa c¡c h…nh vuæng ìn và. C¡c h…nh vuæng ÷æc
tæ m u en, tr›ng ( nh÷ tr¶n b n cí vua). Vîi mØi c°p sŁ nguy¶n d÷ìng
(m,n) x†t mºt tam gi¡c vuæng m c¡c ¿nh câ tåa º nguy¶n, c¡c c⁄nh
Tải tài liệu tại sividoc.com
Viết đề tài giá sinh viên – ZALO:0973.287.149-TEAMLUANVAN.COM
55
b¶n chi•u d i m v n, n‹m dåc theo c⁄nh h…nh vuæng. Kþ hi»u S1 l tŒng
di»n t‰ch phƒn m u en cıa tam gi¡c v S2 l tŒng di»n t‰ch phƒn m u
tr›ng cıa tam gi¡c. °t f(m; n) = jS1
(a) T‰nh f(m,n) vîi måi m v n nguy¶n d÷ìng còng chfin ho°c còng l·.
(b) Chøng minh r‹ng f(m; n)
1
maxfm; ng vîi måi m v n.
2
H…nh 3.1: H…nh chœ nh“t 20 12
Chøng minh. (a) N‚u m v n •u chfin th… f(m; n) = 0. Th“t v“y, gåi M l
trung i”m cıa c⁄nh huy•n th… M l mºt i”m nót cıa l÷îi. N‚u ta quay tam
gi¡c mºt gâc 1800
th… tam gi¡c tròng vîi nßa kia cıa h…nh chœ nh“t
v ta th§y sŁ m u en tr›ng tr¶n c¡c æ l nh÷ nhau. Do â, S1 v S2 Łi vîi
tam gi¡c l mØi nßa gi¡ trà cıa chóng Łi vîi h…nh chœ nh“t. Nh÷ng sŁ
c¡c æ en, tr›ng tr¶n h…nh chœ nh“t th… b‹ng nhau n¶n
f(m; n) = jS1 S2j = 0.
N‚u m v n •u l· th… trung i”m c⁄nh huy•n l t¥m cıa mºt h…nh vuæng v
ta v¤n câ sŁ m u en, tr›ng cıa mØi nßa h…nh chœ nh“t b‹ng
nhau. Lóc n y S1 v S2 hìn 1 so vîi h…nh chœ nh“t, bði v“y f(m; n) =
1
.
2
(b) K‚t qu£ l“p tøc suy ra tł (a) n‚u m v n còng t‰nh chfin l·. L“p lu“n
trong (a) s‡ sai khi m v n câ d§u tr¡i nhau v… hai nßa cıa h…nh
S2j.
Tải tài liệu tại sividoc.com
Viết đề tài giá sinh viên – ZALO:0973.287.149-TEAMLUANVAN.COM
56
chœ nh“t câ m u ng÷æc nhau. Gi£ sß m l c⁄nh l·. Khi â n‚u ta k†o
d i c⁄nh º d i m bði 1 ( ìn và) th… s‡ t⁄o ra mºt tam gi¡c mîi chøa tam
gi¡c ban ƒu nh÷ng nâ câ c£ hai c⁄nh •u chfin v do â S1 = S2.
Tr÷íng hæp x§u nh§t l t§t c£ c¡c di»n t‰ch n y còng m u. Trong tr÷íng
hæp â ta câ f(m; n) =
n
2. Nh÷ng n max(m; n) n¶n ta câ i•u ph£i
chøng minh.
B i to¡n 3.1. (Thi væ àch Moscow, 1961) Cho mºt b£ng æ vuæng câ
k‰ch th÷îc 4 4. Chøng minh r‹ng câ th” °t 7 d§u v o c¡c æ cıa b£ng
sao cho n‚u xâa 2 h ng ho°c 2 cºt b§t ký cıa b£ng th… v¤n câ ‰t
nh§t 1 d§u ch÷a bà xâa. Chøng minh r‹ng n‚u sŁ d§u nhä hìn 7 th…
luæn câ th” xâa 2 h ng v 2 cºt ” måi d§u •u bà xâa.
B i to¡n 3.2. (Thi v o lîp chuy¶n To¡n-Tin H Nºi, Amsterdam, 1998)
Cho h…nh vuæng c⁄nh n(n l sŁ nguy¶n lîn hìn 1) ÷æc chia th nh
n n æ vuæng nhä. Trong mØi æ vuæng nhä n y ch¿ ghi mºt trong 3 sŁ
1,0,-1. H…nh vuæng nh÷ th‚ ÷æc gåi l "b£ng sŁ æ vuæng c⁄nh n".
a. H¢y l“p mºt b£ng sŁ vuæng c⁄nh 6 sao cho tŒng c¡c sŁ ghi trong
b£ng theo måi h ng, måi cºt •u kh¡c nhau.
b. Câ hay khæng mºt b£ng sŁ c⁄nh n n o â m tŒng c¡c sŁ ghi trong
b£ng theo måi h ng, måi cºt v theo hai ÷íng ch†o •u kh¡c nhau.
B i to¡n 3.3. (Thi v o lîp chuy¶n To¡n-Tin H Nºi, ⁄i håc tŒng hæp
th nh phŁ Hç Ch‰ Minh,1994) Cho b£ng k‰ch th÷îc 2n 2n æ
vuæng. Ng÷íi ta ¡nh d§u v o 3n æ b§t ký cıa b£ng. Chøng minh r‹ng
câ th” chån ra n h ng v n cºt cıa b£ng sao cho c¡c æ ÷æc ¡nh d§u •u
n‹m tr¶n n h ng ho°c n cºt n y.
B i to¡n 3.4. (IMO-1987,#5) Cho sŁ tü nhi¶n n 3. Chøng minh r‹ng
câ mºt t“p hæp gçm n i”m trong m°t phflng sao cho kho£ng c¡ch
giœa hai i”m b§t ký l mºt sŁ væ t v mØi bº 3 i”m x¡c ành mºt tam
gi¡c khæng suy bi‚n câ di»n t‰ch hœu t .
Ph÷ìng Ph¡p gi£i: Gåi xn l i”m câ tåa º (n; n2
) vîi n = 1; 2; 3; : : :. Ta i
chøng minh kho£ng c¡ch giœa hai i”m b§t ký l sŁ hœu t v tam gi¡c
x¡c ành bði 3 i”m b§t ký câ di»n t‰ch hœu t kh¡c khæng.
Lưới tọa độ Và một số bài toán liên quan Luận văn thạc sĩ toán học.doc
Lưới tọa độ Và một số bài toán liên quan Luận văn thạc sĩ toán học.doc
Lưới tọa độ Và một số bài toán liên quan Luận văn thạc sĩ toán học.doc

More Related Content

Similar to Lưới tọa độ Và một số bài toán liên quan Luận văn thạc sĩ toán học.doc

Nhà ở cán bộ trung ương - Đại Học Xây Dựng Hà Nội
Nhà ở cán bộ trung ương - Đại Học Xây Dựng Hà NộiNhà ở cán bộ trung ương - Đại Học Xây Dựng Hà Nội
Nhà ở cán bộ trung ương - Đại Học Xây Dựng Hà NộiĐồ án Xây Dựng
 
hệ thống công thức trong cơ học đất
hệ thống công thức trong cơ học đấthệ thống công thức trong cơ học đất
hệ thống công thức trong cơ học đấtAnh Anh
 
Hệ thống công thức cơ học đất
Hệ thống công thức cơ học đấtHệ thống công thức cơ học đất
Hệ thống công thức cơ học đấtTtx Love
 
An sinh xa hoi o Viet Nam luy tien den muc nao?
An sinh xa hoi o Viet Nam luy tien den muc nao?An sinh xa hoi o Viet Nam luy tien den muc nao?
An sinh xa hoi o Viet Nam luy tien den muc nao?foreman
 
10 chuyên đề hình học tổ hợp
10 chuyên đề hình học tổ hợp10 chuyên đề hình học tổ hợp
10 chuyên đề hình học tổ hợpThế Giới Tinh Hoa
 
5 cuong-toan van-luan_an_130107144049
5 cuong-toan van-luan_an_1301071440495 cuong-toan van-luan_an_130107144049
5 cuong-toan van-luan_an_130107144049Phong Tân
 
Luận văn: Hấp thụ sóng điện từ trong Graphene đơn lớp dưới ảnh hưởng của từ t...
Luận văn: Hấp thụ sóng điện từ trong Graphene đơn lớp dưới ảnh hưởng của từ t...Luận văn: Hấp thụ sóng điện từ trong Graphene đơn lớp dưới ảnh hưởng của từ t...
Luận văn: Hấp thụ sóng điện từ trong Graphene đơn lớp dưới ảnh hưởng của từ t...Dịch vụ viết thuê Luận Văn - ZALO 0932091562
 
Chuong 3 he thong mang luoi cong trinh cong cong
Chuong 3   he thong mang luoi cong trinh cong congChuong 3   he thong mang luoi cong trinh cong cong
Chuong 3 he thong mang luoi cong trinh cong congHi House
 

Similar to Lưới tọa độ Và một số bài toán liên quan Luận văn thạc sĩ toán học.doc (20)

Cac ham so so hoc
Cac ham so so hocCac ham so so hoc
Cac ham so so hoc
 
Về hệ số nhị thức, hệ số đa thức Và một số bài toán liên quan.doc
Về hệ số nhị thức, hệ số đa thức Và một số bài toán liên quan.docVề hệ số nhị thức, hệ số đa thức Và một số bài toán liên quan.doc
Về hệ số nhị thức, hệ số đa thức Và một số bài toán liên quan.doc
 
Nhà ở cán bộ trung ương - Đại Học Xây Dựng Hà Nội
Nhà ở cán bộ trung ương - Đại Học Xây Dựng Hà NộiNhà ở cán bộ trung ương - Đại Học Xây Dựng Hà Nội
Nhà ở cán bộ trung ương - Đại Học Xây Dựng Hà Nội
 
hệ thống công thức trong cơ học đất
hệ thống công thức trong cơ học đấthệ thống công thức trong cơ học đất
hệ thống công thức trong cơ học đất
 
Hệ thống công thức cơ học đất
Hệ thống công thức cơ học đấtHệ thống công thức cơ học đất
Hệ thống công thức cơ học đất
 
An sinh xa hoi o Viet Nam luy tien den muc nao?
An sinh xa hoi o Viet Nam luy tien den muc nao?An sinh xa hoi o Viet Nam luy tien den muc nao?
An sinh xa hoi o Viet Nam luy tien den muc nao?
 
Asxhvn
AsxhvnAsxhvn
Asxhvn
 
10 chuyên đề hình học tổ hợp
10 chuyên đề hình học tổ hợp10 chuyên đề hình học tổ hợp
10 chuyên đề hình học tổ hợp
 
5 cuong-toan van-luan_an_130107144049
5 cuong-toan van-luan_an_1301071440495 cuong-toan van-luan_an_130107144049
5 cuong-toan van-luan_an_130107144049
 
Luận văn: Hấp thụ sóng điện từ trong Graphene đơn lớp dưới ảnh hưởng của từ t...
Luận văn: Hấp thụ sóng điện từ trong Graphene đơn lớp dưới ảnh hưởng của từ t...Luận văn: Hấp thụ sóng điện từ trong Graphene đơn lớp dưới ảnh hưởng của từ t...
Luận văn: Hấp thụ sóng điện từ trong Graphene đơn lớp dưới ảnh hưởng của từ t...
 
Luận văn: Hấp thụ sóng điện từ trong Graphene đơn lớp, HAY
Luận văn: Hấp thụ sóng điện từ trong Graphene đơn lớp, HAYLuận văn: Hấp thụ sóng điện từ trong Graphene đơn lớp, HAY
Luận văn: Hấp thụ sóng điện từ trong Graphene đơn lớp, HAY
 
Pháp luật về hoạt động thanh toán bằng thẻ ngân hàng ở Việt Nam.doc
Pháp luật về hoạt động thanh toán bằng thẻ ngân hàng ở Việt Nam.docPháp luật về hoạt động thanh toán bằng thẻ ngân hàng ở Việt Nam.doc
Pháp luật về hoạt động thanh toán bằng thẻ ngân hàng ở Việt Nam.doc
 
Ky thuat lap_trinh
Ky thuat lap_trinhKy thuat lap_trinh
Ky thuat lap_trinh
 
Kỹ thuật lập trình.
Kỹ thuật lập trình.Kỹ thuật lập trình.
Kỹ thuật lập trình.
 
Đề tài: Nhà làm việc của đại sứ quán nước ngoài tại Việt Nam, HOT
Đề tài: Nhà làm việc của đại sứ quán nước ngoài tại Việt Nam, HOTĐề tài: Nhà làm việc của đại sứ quán nước ngoài tại Việt Nam, HOT
Đề tài: Nhà làm việc của đại sứ quán nước ngoài tại Việt Nam, HOT
 
Nguyên Tắc Hai Cấp Xét Xử Trong Tố Tụng Dân Sự.doc
Nguyên Tắc Hai Cấp Xét Xử Trong Tố Tụng Dân Sự.docNguyên Tắc Hai Cấp Xét Xử Trong Tố Tụng Dân Sự.doc
Nguyên Tắc Hai Cấp Xét Xử Trong Tố Tụng Dân Sự.doc
 
Về tổng gauss Và một số ứng dụng.doc
Về tổng gauss Và một số ứng dụng.docVề tổng gauss Và một số ứng dụng.doc
Về tổng gauss Và một số ứng dụng.doc
 
Đề tài: Nhà điều hành trung tâm viện khoa học và công nghệ, HAY
Đề tài: Nhà điều hành trung tâm viện khoa học và công nghệ, HAYĐề tài: Nhà điều hành trung tâm viện khoa học và công nghệ, HAY
Đề tài: Nhà điều hành trung tâm viện khoa học và công nghệ, HAY
 
Luận văn tốt nghiệp: Công trình Khu nhà ở Linh Đàm, HAY
Luận văn tốt nghiệp: Công trình Khu nhà ở Linh Đàm, HAYLuận văn tốt nghiệp: Công trình Khu nhà ở Linh Đàm, HAY
Luận văn tốt nghiệp: Công trình Khu nhà ở Linh Đàm, HAY
 
Chuong 3 he thong mang luoi cong trinh cong cong
Chuong 3   he thong mang luoi cong trinh cong congChuong 3   he thong mang luoi cong trinh cong cong
Chuong 3 he thong mang luoi cong trinh cong cong
 

More from DV Viết Luận văn luanvanmaster.com ZALO 0973287149

More from DV Viết Luận văn luanvanmaster.com ZALO 0973287149 (20)

Ảnh Hưởng Của Marketing Quan Hệ Đến Lòng Trung Thành Của Khách Hàng.Tình Huốn...
Ảnh Hưởng Của Marketing Quan Hệ Đến Lòng Trung Thành Của Khách Hàng.Tình Huốn...Ảnh Hưởng Của Marketing Quan Hệ Đến Lòng Trung Thành Của Khách Hàng.Tình Huốn...
Ảnh Hưởng Của Marketing Quan Hệ Đến Lòng Trung Thành Của Khách Hàng.Tình Huốn...
 
Phát triển nguồn nhân lực tại Uỷ ban nhân dân huyện Trà Bồng, tỉnh Quảng Ngãi...
Phát triển nguồn nhân lực tại Uỷ ban nhân dân huyện Trà Bồng, tỉnh Quảng Ngãi...Phát triển nguồn nhân lực tại Uỷ ban nhân dân huyện Trà Bồng, tỉnh Quảng Ngãi...
Phát triển nguồn nhân lực tại Uỷ ban nhân dân huyện Trà Bồng, tỉnh Quảng Ngãi...
 
Báo cáo tốt Nghiệp tài chính hợp nhất tại tổng công ty Indochina gol...
Báo cáo tốt Nghiệp  tài chính hợp nhất tại tổng công ty Indochina gol...Báo cáo tốt Nghiệp  tài chính hợp nhất tại tổng công ty Indochina gol...
Báo cáo tốt Nghiệp tài chính hợp nhất tại tổng công ty Indochina gol...
 
Tạo động lực thúc đẩy nhân viên làm việc tại ngân hàng TMCP Ngoại Thương Việt...
Tạo động lực thúc đẩy nhân viên làm việc tại ngân hàng TMCP Ngoại Thương Việt...Tạo động lực thúc đẩy nhân viên làm việc tại ngân hàng TMCP Ngoại Thương Việt...
Tạo động lực thúc đẩy nhân viên làm việc tại ngân hàng TMCP Ngoại Thương Việt...
 
Phát triển công nghiệp trên địa bàn Thành phố Tam Kỳ, Tỉnh Quảng Na...
Phát triển công nghiệp trên địa bàn Thành phố Tam Kỳ, Tỉnh Quảng Na...Phát triển công nghiệp trên địa bàn Thành phố Tam Kỳ, Tỉnh Quảng Na...
Phát triển công nghiệp trên địa bàn Thành phố Tam Kỳ, Tỉnh Quảng Na...
 
Giải pháp phát triển cho vay xuất nhập khẩu tại ngân hàng NN&PTNN ch...
Giải pháp phát triển cho vay xuất nhập khẩu tại ngân hàng NN&PTNN ch...Giải pháp phát triển cho vay xuất nhập khẩu tại ngân hàng NN&PTNN ch...
Giải pháp phát triển cho vay xuất nhập khẩu tại ngân hàng NN&PTNN ch...
 
Hoàn thiện công tác lập báo cáo tài chính hợp nhất tại tổng công ...
Hoàn thiện công tác lập báo cáo tài chính hợp nhất tại tổng công ...Hoàn thiện công tác lập báo cáo tài chính hợp nhất tại tổng công ...
Hoàn thiện công tác lập báo cáo tài chính hợp nhất tại tổng công ...
 
Luận Văn Thạc Sĩ Quản trị thành tích nhân viên tại Cục Hải quan TP Đà Nẵng.doc
Luận Văn Thạc Sĩ  Quản trị thành tích nhân viên tại Cục Hải quan TP Đà Nẵng.docLuận Văn Thạc Sĩ  Quản trị thành tích nhân viên tại Cục Hải quan TP Đà Nẵng.doc
Luận Văn Thạc Sĩ Quản trị thành tích nhân viên tại Cục Hải quan TP Đà Nẵng.doc
 
Hoàn thiện công tác quản lý thuế thu nhập cá nhân tại cục thuế Tỉ...
Hoàn thiện công tác quản lý thuế thu nhập cá nhân tại cục thuế Tỉ...Hoàn thiện công tác quản lý thuế thu nhập cá nhân tại cục thuế Tỉ...
Hoàn thiện công tác quản lý thuế thu nhập cá nhân tại cục thuế Tỉ...
 
Đề Tài Phát triển bền vững nông nghiệp Huyện Ba Tơ, Tỉnh Quảng Ngãi....
Đề Tài Phát triển bền vững nông nghiệp Huyện Ba Tơ, Tỉnh Quảng Ngãi....Đề Tài Phát triển bền vững nông nghiệp Huyện Ba Tơ, Tỉnh Quảng Ngãi....
Đề Tài Phát triển bền vững nông nghiệp Huyện Ba Tơ, Tỉnh Quảng Ngãi....
 
Hoàn thiện công tác bảo trợ xã hội trên địa bàn huyện Phong Điền, tỉnh Thừa T...
Hoàn thiện công tác bảo trợ xã hội trên địa bàn huyện Phong Điền, tỉnh Thừa T...Hoàn thiện công tác bảo trợ xã hội trên địa bàn huyện Phong Điền, tỉnh Thừa T...
Hoàn thiện công tác bảo trợ xã hội trên địa bàn huyện Phong Điền, tỉnh Thừa T...
 
Đề Tài Luận VănPhát triển sản phẩm du lịch tại thành phố Đà Nẵng.doc
Đề Tài Luận VănPhát triển sản phẩm du lịch tại thành phố Đà Nẵng.docĐề Tài Luận VănPhát triển sản phẩm du lịch tại thành phố Đà Nẵng.doc
Đề Tài Luận VănPhát triển sản phẩm du lịch tại thành phố Đà Nẵng.doc
 
Đào tạo nghề cho lao động thuộc diện thu hồi đất trên địa bàn Thàn...
Đào tạo nghề cho lao động thuộc diện thu hồi đất trên địa bàn Thàn...Đào tạo nghề cho lao động thuộc diện thu hồi đất trên địa bàn Thàn...
Đào tạo nghề cho lao động thuộc diện thu hồi đất trên địa bàn Thàn...
 
Tóm Tắt Luận Văn Thạc Sĩ Quản Trị Kinh Doanh Xây dựng chính sách Marketing tạ...
Tóm Tắt Luận Văn Thạc Sĩ Quản Trị Kinh Doanh Xây dựng chính sách Marketing tạ...Tóm Tắt Luận Văn Thạc Sĩ Quản Trị Kinh Doanh Xây dựng chính sách Marketing tạ...
Tóm Tắt Luận Văn Thạc Sĩ Quản Trị Kinh Doanh Xây dựng chính sách Marketing tạ...
 
Đề Tài Nghiên cứu rủi ro cảm nhận đối với mua hàng thời trang trực tuyến.docx
Đề Tài Nghiên cứu rủi ro cảm nhận đối với mua hàng thời trang trực tuyến.docxĐề Tài Nghiên cứu rủi ro cảm nhận đối với mua hàng thời trang trực tuyến.docx
Đề Tài Nghiên cứu rủi ro cảm nhận đối với mua hàng thời trang trực tuyến.docx
 
Giải pháp nâng cao động lực thúc đẩy người lao động tại công ty khai...
Giải pháp nâng cao động lực thúc đẩy người lao động tại công ty khai...Giải pháp nâng cao động lực thúc đẩy người lao động tại công ty khai...
Giải pháp nâng cao động lực thúc đẩy người lao động tại công ty khai...
 
Giải pháp phát triển dịch vụ ngân hàng điện tử tại ngân hàng đầu ...
Giải pháp phát triển dịch vụ ngân hàng điện tử tại ngân hàng đầu ...Giải pháp phát triển dịch vụ ngân hàng điện tử tại ngân hàng đầu ...
Giải pháp phát triển dịch vụ ngân hàng điện tử tại ngân hàng đầu ...
 
Giải pháp phát triển dịch vụ ngân hàng điện tử tại ngân hàng đầu ...
Giải pháp phát triển dịch vụ ngân hàng điện tử tại ngân hàng đầu ...Giải pháp phát triển dịch vụ ngân hàng điện tử tại ngân hàng đầu ...
Giải pháp phát triển dịch vụ ngân hàng điện tử tại ngân hàng đầu ...
 
Quản trị quan hệ khách hàng tại Chi nhánh Viettel Đà Nẵng – Tập đoàn Viễn thô...
Quản trị quan hệ khách hàng tại Chi nhánh Viettel Đà Nẵng – Tập đoàn Viễn thô...Quản trị quan hệ khách hàng tại Chi nhánh Viettel Đà Nẵng – Tập đoàn Viễn thô...
Quản trị quan hệ khách hàng tại Chi nhánh Viettel Đà Nẵng – Tập đoàn Viễn thô...
 
Đề Tài Đánh giá thành tích đội ngũ giảng viên trường Đại Học Phạm ...
Đề Tài Đánh giá thành tích đội ngũ giảng viên trường Đại Học Phạm ...Đề Tài Đánh giá thành tích đội ngũ giảng viên trường Đại Học Phạm ...
Đề Tài Đánh giá thành tích đội ngũ giảng viên trường Đại Học Phạm ...
 

Recently uploaded

26 Truyện Ngắn Sơn Nam (Sơn Nam) thuviensach.vn.pdf
26 Truyện Ngắn Sơn Nam (Sơn Nam) thuviensach.vn.pdf26 Truyện Ngắn Sơn Nam (Sơn Nam) thuviensach.vn.pdf
26 Truyện Ngắn Sơn Nam (Sơn Nam) thuviensach.vn.pdfltbdieu
 
bài tập lớn môn kiến trúc máy tính và hệ điều hành
bài tập lớn môn kiến trúc máy tính và hệ điều hànhbài tập lớn môn kiến trúc máy tính và hệ điều hành
bài tập lớn môn kiến trúc máy tính và hệ điều hànhdangdinhkien2k4
 
Trắc nghiệm CHƯƠNG 5 môn Chủ nghĩa xã hội
Trắc nghiệm CHƯƠNG 5 môn Chủ nghĩa xã hộiTrắc nghiệm CHƯƠNG 5 môn Chủ nghĩa xã hội
Trắc nghiệm CHƯƠNG 5 môn Chủ nghĩa xã hộiNgocNguyen591215
 
SD-05_Xây dựng website bán váy Lolita Alice - Phùng Thị Thúy Hiền PH 2 7 8 6 ...
SD-05_Xây dựng website bán váy Lolita Alice - Phùng Thị Thúy Hiền PH 2 7 8 6 ...SD-05_Xây dựng website bán váy Lolita Alice - Phùng Thị Thúy Hiền PH 2 7 8 6 ...
SD-05_Xây dựng website bán váy Lolita Alice - Phùng Thị Thúy Hiền PH 2 7 8 6 ...ChuThNgnFEFPLHN
 
TUYỂN TẬP 50 ĐỀ LUYỆN THI TUYỂN SINH LỚP 10 THPT MÔN TOÁN NĂM 2024 CÓ LỜI GIẢ...
TUYỂN TẬP 50 ĐỀ LUYỆN THI TUYỂN SINH LỚP 10 THPT MÔN TOÁN NĂM 2024 CÓ LỜI GIẢ...TUYỂN TẬP 50 ĐỀ LUYỆN THI TUYỂN SINH LỚP 10 THPT MÔN TOÁN NĂM 2024 CÓ LỜI GIẢ...
TUYỂN TẬP 50 ĐỀ LUYỆN THI TUYỂN SINH LỚP 10 THPT MÔN TOÁN NĂM 2024 CÓ LỜI GIẢ...Nguyen Thanh Tu Collection
 
xemsomenh.com-Vòng Thái Tuế và Ý Nghĩa Các Sao Tại Cung Mệnh.pdf
xemsomenh.com-Vòng Thái Tuế và Ý Nghĩa Các Sao Tại Cung Mệnh.pdfxemsomenh.com-Vòng Thái Tuế và Ý Nghĩa Các Sao Tại Cung Mệnh.pdf
xemsomenh.com-Vòng Thái Tuế và Ý Nghĩa Các Sao Tại Cung Mệnh.pdfXem Số Mệnh
 
xemsomenh.com-Vòng Lộc Tồn - Vòng Bác Sĩ và Cách An Trong Vòng Lộc Tồn.pdf
xemsomenh.com-Vòng Lộc Tồn - Vòng Bác Sĩ và Cách An Trong Vòng Lộc Tồn.pdfxemsomenh.com-Vòng Lộc Tồn - Vòng Bác Sĩ và Cách An Trong Vòng Lộc Tồn.pdf
xemsomenh.com-Vòng Lộc Tồn - Vòng Bác Sĩ và Cách An Trong Vòng Lộc Tồn.pdfXem Số Mệnh
 
Đề thi tin học HK2 lớp 3 Chân Trời Sáng Tạo
Đề thi tin học HK2 lớp 3 Chân Trời Sáng TạoĐề thi tin học HK2 lớp 3 Chân Trời Sáng Tạo
Đề thi tin học HK2 lớp 3 Chân Trời Sáng Tạowindcances
 
SLIDE - Tu van, huong dan cong tac tuyen sinh-2024 (đầy đủ chi tiết).pdf
SLIDE - Tu van, huong dan cong tac tuyen sinh-2024 (đầy đủ chi tiết).pdfSLIDE - Tu van, huong dan cong tac tuyen sinh-2024 (đầy đủ chi tiết).pdf
SLIDE - Tu van, huong dan cong tac tuyen sinh-2024 (đầy đủ chi tiết).pdfhoangtuansinh1
 
Giáo trình nhập môn lập trình - Đặng Bình Phương
Giáo trình nhập môn lập trình - Đặng Bình PhươngGiáo trình nhập môn lập trình - Đặng Bình Phương
Giáo trình nhập môn lập trình - Đặng Bình Phươnghazzthuan
 
30 ĐỀ PHÁT TRIỂN THEO CẤU TRÚC ĐỀ MINH HỌA BGD NGÀY 22-3-2024 KỲ THI TỐT NGHI...
30 ĐỀ PHÁT TRIỂN THEO CẤU TRÚC ĐỀ MINH HỌA BGD NGÀY 22-3-2024 KỲ THI TỐT NGHI...30 ĐỀ PHÁT TRIỂN THEO CẤU TRÚC ĐỀ MINH HỌA BGD NGÀY 22-3-2024 KỲ THI TỐT NGHI...
30 ĐỀ PHÁT TRIỂN THEO CẤU TRÚC ĐỀ MINH HỌA BGD NGÀY 22-3-2024 KỲ THI TỐT NGHI...Nguyen Thanh Tu Collection
 
xemsomenh.com-Vòng Tràng Sinh - Cách An 12 Sao Và Ý Nghĩa Từng Sao.pdf
xemsomenh.com-Vòng Tràng Sinh - Cách An 12 Sao Và Ý Nghĩa Từng Sao.pdfxemsomenh.com-Vòng Tràng Sinh - Cách An 12 Sao Và Ý Nghĩa Từng Sao.pdf
xemsomenh.com-Vòng Tràng Sinh - Cách An 12 Sao Và Ý Nghĩa Từng Sao.pdfXem Số Mệnh
 
bài thi bảo vệ nền tảng tư tưởng của Đảng.docx
bài thi bảo vệ nền tảng tư tưởng của Đảng.docxbài thi bảo vệ nền tảng tư tưởng của Đảng.docx
bài thi bảo vệ nền tảng tư tưởng của Đảng.docxTrnHiYn5
 
Danh sách sinh viên tốt nghiệp Đại học - Cao đẳng Trường Đại học Phú Yên năm ...
Danh sách sinh viên tốt nghiệp Đại học - Cao đẳng Trường Đại học Phú Yên năm ...Danh sách sinh viên tốt nghiệp Đại học - Cao đẳng Trường Đại học Phú Yên năm ...
Danh sách sinh viên tốt nghiệp Đại học - Cao đẳng Trường Đại học Phú Yên năm ...hoangtuansinh1
 
Giáo trình xây dựng thực đơn. Ths Hoang Ngoc Hien.pdf
Giáo trình xây dựng thực đơn. Ths Hoang Ngoc Hien.pdfGiáo trình xây dựng thực đơn. Ths Hoang Ngoc Hien.pdf
Giáo trình xây dựng thực đơn. Ths Hoang Ngoc Hien.pdf4pdx29gsr9
 
Access: Chuong III Thiet ke truy van Query.ppt
Access: Chuong III Thiet ke truy van Query.pptAccess: Chuong III Thiet ke truy van Query.ppt
Access: Chuong III Thiet ke truy van Query.pptPhamThiThuThuy1
 
ĐỀ CHÍNH THỨC KỲ THI TUYỂN SINH VÀO LỚP 10 THPT CÁC TỈNH THÀNH NĂM HỌC 2020 –...
ĐỀ CHÍNH THỨC KỲ THI TUYỂN SINH VÀO LỚP 10 THPT CÁC TỈNH THÀNH NĂM HỌC 2020 –...ĐỀ CHÍNH THỨC KỲ THI TUYỂN SINH VÀO LỚP 10 THPT CÁC TỈNH THÀNH NĂM HỌC 2020 –...
ĐỀ CHÍNH THỨC KỲ THI TUYỂN SINH VÀO LỚP 10 THPT CÁC TỈNH THÀNH NĂM HỌC 2020 –...Nguyen Thanh Tu Collection
 
30 ĐỀ PHÁT TRIỂN THEO CẤU TRÚC ĐỀ MINH HỌA BGD NGÀY 22-3-2024 KỲ THI TỐT NGHI...
30 ĐỀ PHÁT TRIỂN THEO CẤU TRÚC ĐỀ MINH HỌA BGD NGÀY 22-3-2024 KỲ THI TỐT NGHI...30 ĐỀ PHÁT TRIỂN THEO CẤU TRÚC ĐỀ MINH HỌA BGD NGÀY 22-3-2024 KỲ THI TỐT NGHI...
30 ĐỀ PHÁT TRIỂN THEO CẤU TRÚC ĐỀ MINH HỌA BGD NGÀY 22-3-2024 KỲ THI TỐT NGHI...Nguyen Thanh Tu Collection
 
Giới thiệu Dự án Sản Phụ Khoa - Y Học Cộng Đồng
Giới thiệu Dự án Sản Phụ Khoa - Y Học Cộng ĐồngGiới thiệu Dự án Sản Phụ Khoa - Y Học Cộng Đồng
Giới thiệu Dự án Sản Phụ Khoa - Y Học Cộng ĐồngYhoccongdong.com
 
ĐỀ KIỂM TRA CUỐI KÌ 2 BIÊN SOẠN THEO ĐỊNH HƯỚNG ĐỀ BGD 2025 MÔN TOÁN 10 - CÁN...
ĐỀ KIỂM TRA CUỐI KÌ 2 BIÊN SOẠN THEO ĐỊNH HƯỚNG ĐỀ BGD 2025 MÔN TOÁN 10 - CÁN...ĐỀ KIỂM TRA CUỐI KÌ 2 BIÊN SOẠN THEO ĐỊNH HƯỚNG ĐỀ BGD 2025 MÔN TOÁN 10 - CÁN...
ĐỀ KIỂM TRA CUỐI KÌ 2 BIÊN SOẠN THEO ĐỊNH HƯỚNG ĐỀ BGD 2025 MÔN TOÁN 10 - CÁN...Nguyen Thanh Tu Collection
 

Recently uploaded (20)

26 Truyện Ngắn Sơn Nam (Sơn Nam) thuviensach.vn.pdf
26 Truyện Ngắn Sơn Nam (Sơn Nam) thuviensach.vn.pdf26 Truyện Ngắn Sơn Nam (Sơn Nam) thuviensach.vn.pdf
26 Truyện Ngắn Sơn Nam (Sơn Nam) thuviensach.vn.pdf
 
bài tập lớn môn kiến trúc máy tính và hệ điều hành
bài tập lớn môn kiến trúc máy tính và hệ điều hànhbài tập lớn môn kiến trúc máy tính và hệ điều hành
bài tập lớn môn kiến trúc máy tính và hệ điều hành
 
Trắc nghiệm CHƯƠNG 5 môn Chủ nghĩa xã hội
Trắc nghiệm CHƯƠNG 5 môn Chủ nghĩa xã hộiTrắc nghiệm CHƯƠNG 5 môn Chủ nghĩa xã hội
Trắc nghiệm CHƯƠNG 5 môn Chủ nghĩa xã hội
 
SD-05_Xây dựng website bán váy Lolita Alice - Phùng Thị Thúy Hiền PH 2 7 8 6 ...
SD-05_Xây dựng website bán váy Lolita Alice - Phùng Thị Thúy Hiền PH 2 7 8 6 ...SD-05_Xây dựng website bán váy Lolita Alice - Phùng Thị Thúy Hiền PH 2 7 8 6 ...
SD-05_Xây dựng website bán váy Lolita Alice - Phùng Thị Thúy Hiền PH 2 7 8 6 ...
 
TUYỂN TẬP 50 ĐỀ LUYỆN THI TUYỂN SINH LỚP 10 THPT MÔN TOÁN NĂM 2024 CÓ LỜI GIẢ...
TUYỂN TẬP 50 ĐỀ LUYỆN THI TUYỂN SINH LỚP 10 THPT MÔN TOÁN NĂM 2024 CÓ LỜI GIẢ...TUYỂN TẬP 50 ĐỀ LUYỆN THI TUYỂN SINH LỚP 10 THPT MÔN TOÁN NĂM 2024 CÓ LỜI GIẢ...
TUYỂN TẬP 50 ĐỀ LUYỆN THI TUYỂN SINH LỚP 10 THPT MÔN TOÁN NĂM 2024 CÓ LỜI GIẢ...
 
xemsomenh.com-Vòng Thái Tuế và Ý Nghĩa Các Sao Tại Cung Mệnh.pdf
xemsomenh.com-Vòng Thái Tuế và Ý Nghĩa Các Sao Tại Cung Mệnh.pdfxemsomenh.com-Vòng Thái Tuế và Ý Nghĩa Các Sao Tại Cung Mệnh.pdf
xemsomenh.com-Vòng Thái Tuế và Ý Nghĩa Các Sao Tại Cung Mệnh.pdf
 
xemsomenh.com-Vòng Lộc Tồn - Vòng Bác Sĩ và Cách An Trong Vòng Lộc Tồn.pdf
xemsomenh.com-Vòng Lộc Tồn - Vòng Bác Sĩ và Cách An Trong Vòng Lộc Tồn.pdfxemsomenh.com-Vòng Lộc Tồn - Vòng Bác Sĩ và Cách An Trong Vòng Lộc Tồn.pdf
xemsomenh.com-Vòng Lộc Tồn - Vòng Bác Sĩ và Cách An Trong Vòng Lộc Tồn.pdf
 
Đề thi tin học HK2 lớp 3 Chân Trời Sáng Tạo
Đề thi tin học HK2 lớp 3 Chân Trời Sáng TạoĐề thi tin học HK2 lớp 3 Chân Trời Sáng Tạo
Đề thi tin học HK2 lớp 3 Chân Trời Sáng Tạo
 
SLIDE - Tu van, huong dan cong tac tuyen sinh-2024 (đầy đủ chi tiết).pdf
SLIDE - Tu van, huong dan cong tac tuyen sinh-2024 (đầy đủ chi tiết).pdfSLIDE - Tu van, huong dan cong tac tuyen sinh-2024 (đầy đủ chi tiết).pdf
SLIDE - Tu van, huong dan cong tac tuyen sinh-2024 (đầy đủ chi tiết).pdf
 
Giáo trình nhập môn lập trình - Đặng Bình Phương
Giáo trình nhập môn lập trình - Đặng Bình PhươngGiáo trình nhập môn lập trình - Đặng Bình Phương
Giáo trình nhập môn lập trình - Đặng Bình Phương
 
30 ĐỀ PHÁT TRIỂN THEO CẤU TRÚC ĐỀ MINH HỌA BGD NGÀY 22-3-2024 KỲ THI TỐT NGHI...
30 ĐỀ PHÁT TRIỂN THEO CẤU TRÚC ĐỀ MINH HỌA BGD NGÀY 22-3-2024 KỲ THI TỐT NGHI...30 ĐỀ PHÁT TRIỂN THEO CẤU TRÚC ĐỀ MINH HỌA BGD NGÀY 22-3-2024 KỲ THI TỐT NGHI...
30 ĐỀ PHÁT TRIỂN THEO CẤU TRÚC ĐỀ MINH HỌA BGD NGÀY 22-3-2024 KỲ THI TỐT NGHI...
 
xemsomenh.com-Vòng Tràng Sinh - Cách An 12 Sao Và Ý Nghĩa Từng Sao.pdf
xemsomenh.com-Vòng Tràng Sinh - Cách An 12 Sao Và Ý Nghĩa Từng Sao.pdfxemsomenh.com-Vòng Tràng Sinh - Cách An 12 Sao Và Ý Nghĩa Từng Sao.pdf
xemsomenh.com-Vòng Tràng Sinh - Cách An 12 Sao Và Ý Nghĩa Từng Sao.pdf
 
bài thi bảo vệ nền tảng tư tưởng của Đảng.docx
bài thi bảo vệ nền tảng tư tưởng của Đảng.docxbài thi bảo vệ nền tảng tư tưởng của Đảng.docx
bài thi bảo vệ nền tảng tư tưởng của Đảng.docx
 
Danh sách sinh viên tốt nghiệp Đại học - Cao đẳng Trường Đại học Phú Yên năm ...
Danh sách sinh viên tốt nghiệp Đại học - Cao đẳng Trường Đại học Phú Yên năm ...Danh sách sinh viên tốt nghiệp Đại học - Cao đẳng Trường Đại học Phú Yên năm ...
Danh sách sinh viên tốt nghiệp Đại học - Cao đẳng Trường Đại học Phú Yên năm ...
 
Giáo trình xây dựng thực đơn. Ths Hoang Ngoc Hien.pdf
Giáo trình xây dựng thực đơn. Ths Hoang Ngoc Hien.pdfGiáo trình xây dựng thực đơn. Ths Hoang Ngoc Hien.pdf
Giáo trình xây dựng thực đơn. Ths Hoang Ngoc Hien.pdf
 
Access: Chuong III Thiet ke truy van Query.ppt
Access: Chuong III Thiet ke truy van Query.pptAccess: Chuong III Thiet ke truy van Query.ppt
Access: Chuong III Thiet ke truy van Query.ppt
 
ĐỀ CHÍNH THỨC KỲ THI TUYỂN SINH VÀO LỚP 10 THPT CÁC TỈNH THÀNH NĂM HỌC 2020 –...
ĐỀ CHÍNH THỨC KỲ THI TUYỂN SINH VÀO LỚP 10 THPT CÁC TỈNH THÀNH NĂM HỌC 2020 –...ĐỀ CHÍNH THỨC KỲ THI TUYỂN SINH VÀO LỚP 10 THPT CÁC TỈNH THÀNH NĂM HỌC 2020 –...
ĐỀ CHÍNH THỨC KỲ THI TUYỂN SINH VÀO LỚP 10 THPT CÁC TỈNH THÀNH NĂM HỌC 2020 –...
 
30 ĐỀ PHÁT TRIỂN THEO CẤU TRÚC ĐỀ MINH HỌA BGD NGÀY 22-3-2024 KỲ THI TỐT NGHI...
30 ĐỀ PHÁT TRIỂN THEO CẤU TRÚC ĐỀ MINH HỌA BGD NGÀY 22-3-2024 KỲ THI TỐT NGHI...30 ĐỀ PHÁT TRIỂN THEO CẤU TRÚC ĐỀ MINH HỌA BGD NGÀY 22-3-2024 KỲ THI TỐT NGHI...
30 ĐỀ PHÁT TRIỂN THEO CẤU TRÚC ĐỀ MINH HỌA BGD NGÀY 22-3-2024 KỲ THI TỐT NGHI...
 
Giới thiệu Dự án Sản Phụ Khoa - Y Học Cộng Đồng
Giới thiệu Dự án Sản Phụ Khoa - Y Học Cộng ĐồngGiới thiệu Dự án Sản Phụ Khoa - Y Học Cộng Đồng
Giới thiệu Dự án Sản Phụ Khoa - Y Học Cộng Đồng
 
ĐỀ KIỂM TRA CUỐI KÌ 2 BIÊN SOẠN THEO ĐỊNH HƯỚNG ĐỀ BGD 2025 MÔN TOÁN 10 - CÁN...
ĐỀ KIỂM TRA CUỐI KÌ 2 BIÊN SOẠN THEO ĐỊNH HƯỚNG ĐỀ BGD 2025 MÔN TOÁN 10 - CÁN...ĐỀ KIỂM TRA CUỐI KÌ 2 BIÊN SOẠN THEO ĐỊNH HƯỚNG ĐỀ BGD 2025 MÔN TOÁN 10 - CÁN...
ĐỀ KIỂM TRA CUỐI KÌ 2 BIÊN SOẠN THEO ĐỊNH HƯỚNG ĐỀ BGD 2025 MÔN TOÁN 10 - CÁN...
 

Lưới tọa độ Và một số bài toán liên quan Luận văn thạc sĩ toán học.doc

  • 1. Tải tài liệu tại sividoc.com Viết đề tài giá sinh viên – ZALO:0973.287.149-TEAMLUANVAN.COM ĐẠI HỌC THÁI NGUYÊN TRƯỜNG ĐẠI HỌC KHOA HỌC --------------  ------------- NGUYỄN THỊ THANH MAI LƯỚI TỌA ĐỘ VÀ MỘT SỐ BÀI TOÁN LIÊN QUAN LUẬN VĂN THẠC SĨ TOÁN HỌC THÁI NGUYÊN - 2017
  • 2. Tải tài liệu tại sividoc.com Viết đề tài giá sinh viên – ZALO:0973.287.149-TEAMLUANVAN.COM ĐẠI HỌC THÁI NGUYÊN TRƯỜNG ĐẠI HỌC KHOA HỌC --------------  ------------- NGUYỄN THỊ THANH MAI LƯỚI TỌA ĐỘ VÀ MỘT SỐ BÀI TOÁN LIÊN QUAN LUẬN VĂN THẠC SĨ TOÁN HỌC Chuyên ngành: Phương pháp Toán sơ cấp Mã số: 60 46 01 13 NGƯỜI HƯỚNG DẪN KHOA HỌC PGS.TS. Nguyễn Việt Hải THÁI NGUYÊN - 2017
  • 3. Tải tài liệu tại sividoc.com Viết đề tài giá sinh viên – ZALO:0973.287.149-TEAMLUANVAN.COM i Danh möc h…nh 1.1 L÷îi tåa º nguy¶n . . . . . . . . . . . . . . . . . . . . 3 1.2 L÷îi tåa º tr¶n m°t phflng . . . . . . . . . . . . . . . . 4 1.3 l÷îi tåa º nguy¶n vîi hbh cì sð . . . . . . . . . . . . . 4 1.4 H…nh v‡ a. H…nh v‡ b . . . . . . . . . . . . . . . . . . . 6 1.5 Ngô gi¡c v löc gi¡c . . . . . . . . . . . . . . . . . . . . 10 1.6 H…nh vuæng v h…nh b¡t gi¡c . . . . . . . . . . . . . . . 12 1.7 a gi¡c •u-c⁄nh . . . . . . . . . . . . . . . . . . . . . 14 1.8 a gi¡c •u-gâc . . . . . . . . . . . . . . . . . . . . . . 15 1.9 Tam gi¡c •u hƒu nºi ti‚p l÷îi . . . . . . . . . . . . . . 17 1.10 a gi¡c ìn v a gi¡c khæng ìn . . . . . . . . . . . . 19 1.11 Mºt sŁ b i to¡n li¶n quan . . . . . . . . . . . . . . . . . 23 2.1 Cºng v nh¥n c¡c i”m nguy¶n . . . . . . . . . . . . . . 34 . 2.2 . 36 a.C¡c i”m nguy¶n.(2 + 3i); b. C¡c i”m nguy¶n li¶n k‚t 2.3 F(R) v N(R) . . . . . . . . . . . . . . . . . . . . . . . 39 2.4 r(4)=4;r(25)=12 . . . . . . . . . . . . . . . . . . . . . . 41 2.5 n=8, n=6 . . . . . . . . . . . . . . . . . . . . . . . . . 43 2.6 n=12 . . . . . . . . . . . . . . . . . . . . . . . . . . . . 43 3.1 H…nh chœ nh“t 20 12 . . . . . . . . . . . . . . . . . . 55
  • 4. Tải tài liệu tại sividoc.com Viết đề tài giá sinh viên – ZALO:0973.287.149-TEAMLUANVAN.COM ii Möc löc Líi c£m ìn ii Mð ƒu 1 1 L÷îi tåa º v a gi¡c 3 1.1 L÷îi tåa º nguy¶n tr¶n m°t phflng v c¡c t‰nh ch§t . . 3 1.1.1 L÷îi tåa º tr¶n m°t phflng . . . . . . . . . . . . 3 1.1.2 L÷îi tåa º nguy¶n v a gi¡c •u . . . . . . . . 7 1.1.3 a gi¡c nßa •u nºi ti‚p l÷îi nguy¶n . . . . . . . 12 1.2 a gi¡c hƒu nºi ti‚p l÷îi nguy¶n . . . . . . . . . . . . . 16 1.3 Cæng thøc Picard . . . . . . . . . . . . . . . . . . . . . 19 1.3.1 Tam gi¡c ìn nguy¶n thıy . . . . . . . . . . . . 19 1.3.2 Cæng thøc Picard . . . . . . . . . . . . . . . . . 20 1.3.3 Mºt sŁ b i to¡n ¡p döng cæng thøc Picard . . . . 22 1.4 Mºt sŁ øng döng cıa l÷îi tåa º nguy¶n . . . . . . . . . 25 1.4.1 Gi¡ trà væ t cıa h m l÷æng gi¡c . . . . . . . . . 25 1.4.2 C¡c b i to¡n tr¶n æ vuæng . . . . . . . . . . . . 27 2 L÷îi tåa º nguy¶n v ÷íng trÆn 33 2.1 i”m nguy¶n nguy¶n tŁ . . . . . . . . . . . . . . . . . . 33 2.1.1 Sü chia h‚t cıa c¡c i”m nguy¶n . . . . . . . . . 35 2.1.2 ành lþ cì b£n cıa c¡c i”m nguy¶n . . . . . . . 36 2.2 ÷íng trÆn tr¶n l÷îi tåa º nguy¶n . . . . . . . . . . . 38 2.2.1 SŁ c¡c bi”u di„n cıa mºt sŁ tü nhi¶n th nh tŒng hai b…nh ph÷ìng . . . . . . . . . . . . . . . . . . 41
  • 5. Tải tài liệu tại sividoc.com Viết đề tài giá sinh viên – ZALO:0973.287.149-TEAMLUANVAN.COM i 2.2.2 ÷íng trÆn Sinhsel . . . . . . . . . . . . . . . . 42 2.3 Mºt sŁ øng döng v o sŁ håc . . . . . . . . . . . . . . . 46 2.3.1 Bº ba Pythagoras . . . . . . . . . . . . . . . . . 46 2.3.2 C¡c d⁄ng cıa i”m nguy¶n nguy¶n tŁ . . . . . . 47 3 C¡c b i to¡n kh¡c 49 3.1 i”m nguy¶n tr¶n ÷íng cong phflng . . . . . . . . . . . 49 3.2 Mºt sŁ to¡n thi håc sinh giäi v thi Olympic . . . . . . 51 T i li»u tham kh£o 59
  • 6. Tải tài liệu tại sividoc.com Viết đề tài giá sinh viên – ZALO:0973.287.149-TEAMLUANVAN.COM ii Líi c£m ìn ” ho n th nh ÷æc lu“n v«n mºt c¡ch ho n ch¿nh, tæi luæn nh“n ÷æc sü h÷îng d¤n v gióp ï nhi»t t…nh cıa PGS.TS. Nguy„n Vi»t H£i, Gi£ng vi¶n cao c§p Tr÷íng ⁄i håc H£i PhÆng. Tæi xin ch¥n th nh b y tä lÆng bi‚t ìn s¥u s›c ‚n thƒy v xin gßi líi tri ¥n nh§t cıa tæi Łi vîi nhœng i•u thƒy ¢ d nh cho tæi. Tæi xin ch¥n th nh c£m ìn ban l¢nh ⁄o phÆng o t⁄o sau ⁄i håc, quþ thƒy cæ gi£ng d⁄y lîp Cao håc K9B2 (2015 - 2017) Tr÷íng ⁄i håc Khoa håc - ⁄i håc Th¡i Nguy¶n ¢ t“n t…nh truy•n ⁄t nhœng ki‚n thøc quþ b¡u công nh÷ t⁄o i•u ki»n cho tæi ho n th nh khâa håc. Tæi xin gßi líi c£m ìn ch¥n th nh nh§t tîi gia …nh, b⁄n b–, nhœng ng÷íi ¢ luæn ºng vi¶n, hØ træ v t⁄o måi i•u ki»n cho tæi trong suŁt qu¡ tr…nh håc t“p v thüc hi»n lu“n v«n. Xin tr¥n trång c£m ìn! H£i PhÆng, th¡ng 9 n«m 2017 Ng÷íi vi‚t Lu“n v«n Nguy„n Thà Thanh Mai
  • 7. Tải tài liệu tại sividoc.com Viết đề tài giá sinh viên – ZALO:0973.287.149-TEAMLUANVAN.COM 1 Mð ƒu 1. Möc ‰ch cıa • t i lu“n v«n - Nghi¶n cøu c¡c b i to¡n tr¶n l÷îi tåa º nguy¶n: a gi¡c •u tr¶n l÷îi, ÷íng trÆn tr¶n l÷îi, c¡c b i to¡n sŁ håc, h…nh håc tŒ hæp, l÷æng gi¡c... tr¶n l÷îi v c¡c v§n • li¶n quan. - Tr…nh b y cì sð khoa håc v v“n döng c¡c ki‚n thøc cıa sŁ håc, ⁄i sŁ, h…nh håc,...c¡c nguy¶n t›c nh÷ nguy¶n t›c Dirichlet, nguy¶n t›c cüc h⁄n, nguy¶n t›c b§t bi‚n v o vi»c gi£i c¡c b i to¡n vîi l÷îi nguy¶n. - C¡c ph÷ìng ph¡p t÷ duy °c tr÷ng v mºt sŁ k‚t qu£ mîi ÷æc tr…nh b y mºt c¡ch h» thŁng v n¥ng cao qua c¡c b i to¡n hay v khâ trong c¡c ký thi Olympic QuŁc gia v QuŁc t‚. - Ng÷íi nghi¶n cøu câ th¶m ki‚n thøc v n«ng lüc bçi d÷ïng håc sinh giäi v• c¡c v§n • khâ, hay g°p cıa To¡n håc. 2. Nºi dung cıa • t i, nhœng v§n • cƒn gi£i quy‚t Tr…nh b y h» thŁng mºt sŁ b i to¡n tr¶n l÷îi tåa º nguy¶n düa tr¶n c¡c b i b¡o [3], [5] v [2]. Ph¡t bi”u v chøng minh c¡c k‚t qu£ li¶n quan ‚n sŁ håc v h…nh håc. • t i gçm 3 ch÷ìng: Ch÷ìng 1. L÷îi tåa º v a gi¡c B i to¡n °t ra l : Düng ÷æc v khæng düng ÷æc a gi¡c •u n o tr¶n l÷îi tåa º nguy¶n m c¡c ¿nh tròng vîi nót l÷îi? L÷îi tåa º nguy¶n tr¶n m°t phflng v c¡c t‰nh ch§t, a gi¡c hƒu nºi ti‚p l÷îi nguy¶n, Cæng thøc Picard v c¡c øng döng cıa l÷îi nguy¶n v o c¡c b i
  • 8. Tải tài liệu tại sividoc.com Viết đề tài giá sinh viên – ZALO:0973.287.149-TEAMLUANVAN.COM 2 to¡n h…nh håc, sŁ håc, tŒ hæp, l÷æng gi¡c,... l nhœng v§n • ÷æc nghi¶n cøu trong ch÷ìng n y. Ch÷ìng 1 câ c¡c möc sau: 1.1. L÷îi tåa º nguy¶n tr¶n m°t phflng v c¡c t‰nh ch§t 1.2. a gi¡c hƒu nºi ti‚p l÷îi nguy¶n 1.3. Cæng thøc Picard 1.4. Mºt sŁ øng döng cıa l÷îi tåa º nguy¶n Ch÷ìng 2. L÷îi tåa º nguy¶n v ÷íng trÆn L÷îi tåa º nguy¶n ch‰nh l v nh Gauss n¶n nºi dung ch÷ìng n y khai th¡c nhœng v§n • v• sŁ håc c¡c sŁ nguy¶n: i”m nguy¶n nguy¶n tŁ, ÷íng trÆn tr¶n l÷îi tåa º nguy¶n, mºt sŁ øng döng v o sŁ håc v b i to¡n li¶n quan ‚n bº ba Pythagoras,... Ch÷ìng 2 bao gçm c¡c möc sau: 2.1. i”m nguy¶n nguy¶n tŁ 2.2. ÷íng trÆn tr¶n l÷îi tåa º nguy¶n 2.3. Mºt sŁ øng döng v o sŁ håc Ch÷ìng 3. C¡c b i to¡n kh¡c Li¶n quan ‚n l÷îi tåa º nguy¶n cÆn câ c¡c b i to¡n v• i”m nguy¶n tr¶n ÷íng cong phflng, C¡c b i to¡n thi håc sinh giäi v thi Olympic. 3.1. i”m nguy¶n tr¶n ÷íng cong phflng 3.1. C¡c b i to¡n thi håc sinh giäi v thi Olympic T¡c gi£
  • 9. Tải tài liệu tại sividoc.com Viết đề tài giá sinh viên – ZALO:0973.287.149-TEAMLUANVAN.COM 3 Ch÷ìng 1 L÷îi tåa º v a gi¡c 1.1 L÷îi tåa º nguy¶n tr¶n m°t phflng v c¡c t‰nh ch§t 1.1.1 L÷îi tåa º tr¶n m°t phflng Tr¶n m°t phflng ta x†t mºt l÷îi t⁄o bði hai hå c¡c ÷íng thflng song song chia m°t phflng th nh c¡c h…nh b…nh h nh b‹ng nhau. H…nh 1.1: L÷îi tåa º nguy¶n T“p hæp t§t c£ c¡c ¿nh c¡c h…nh b…nh h nh gåi l l÷îi tåa º, b£n th¥n c¡c ¿nh gåi l c¡c nót cıa l÷îi. Måi h…nh b…nh h nh t⁄o bði 2 hå ÷íng thflng song song gåi l h…nh b…nh h nh cì sð cıa ph¥n ho⁄ch hay h…nh b…nh h nh sinh ra l÷îi. Chó þ r‹ng mºt l÷îi câ th” nh“n ÷æc tł c¡c hå ÷íng thflng kh¡c nhau: Tr¶n H…nh 1.1 bi”u di„n mºt l÷îi tåa º nguy¶n, tøc l t“p hæp
  • 10. Tải tài liệu tại sividoc.com Viết đề tài giá sinh viên – ZALO:0973.287.149-TEAMLUANVAN.COM 4 c¡c i”m câ tåa º Descartes l c¡c sŁ nguy¶n. L÷îi tåa º nguy¶n ch‰nh l l÷îi t⁄o bði t§t c£ c¡c ÷íng thflng song song vîi hai tröc tåa º, câ th” coi l tí gi§y k· æ vuæng væ h⁄n (h…nh b…nh h nh cì sð l h…nh vuæng c⁄nh 1). L÷îi tåa º nguy¶n câ th” nh“n ÷æc tł c¡c " ÷íng xi¶n" (khi â h…nh b…nh h nh cì sð l h…nh b…nh h nh ABCD tr¶n H…nh 1.2). H…nh 1.2: L÷îi tåa º tr¶n m°t phflng Nh÷ v“y, kh¡i ni»m h…nh b…nh h nh cì sð khæng ch¿ g›n vîi b£n th¥n l÷îi m cÆn g›n vîi c¡c hå ÷íng thflng sinh ra l÷îi. Ta câ c¡c t‰nh ch§t ìn gi£n cıa l÷îi nh÷ sau: H…nh 1.3: l÷îi tåa º nguy¶n vîi hbh cì sð i. Måi ph†p tành ti‚n song song bi‚n i”m nót n y th nh mºt i”m nót kh¡c s‡ b£o to n l÷îi (bi‚n l÷îi th nh ch‰nh nâ). ii. (BŒ • v• ¿nh thø t÷ cıa h…nh b…nh h nh) N‚u 3 ¿nh cıa h…nh
  • 11. Tải tài liệu tại sividoc.com Viết đề tài giá sinh viên – ZALO:0973.287.149-TEAMLUANVAN.COM 5 b…nh h nh l c¡c i”m nót cıa l÷îi th… ¿nh thø t÷ công l i”m nót cıa l÷îi. iii. N‚u qua 2 i”m Q, R cıa l÷îi k· mºt ÷íng thflng th… ÷íng thflng n y s‡ i qua væ h⁄n c¡c i”m nót cıa l÷îi. Khi â t§t c£ c¡c kho£ng c¡ch giœa c¡c nót gƒn nhau •u b‹ng nhau. iv. N‚u mºt h…nh b…nh h nh vîi ¿nh l c¡c i”m nót khæng chøa trong nâ mºt i”m nót n o cıa l÷îi th… nâ l h…nh b…nh h nh sinh ra l÷îi. v. (BŒ • v• thæ s«n v thä) Gi£ sß tia ‘ i qua i”m nót A cıa l÷îi n o â. Khi â t…m ÷æc mºt nót sao cho kho£ng c¡ch tł â ‚n ‘ nhä hìn sŁ nhä tòy þ cho tr÷îc. Chøng minh. C¡c t‰nh ch§t i:; ii:; iii:; iv: •u hi”n nhi¶n, ta i chøng minh bŒ • v: v• thæ s«n v thä: Kþ hi»u giao i”m cıa tia ‘ v ÷íng thflng nghi¶ng cıa l÷îi l A0 = A; A1; A2; :::; An; :::. Nh§c t§t c£ c¡c h…nh b…nh h nh m câ c¡c i”m n y n‹m tr¶n ÷íng thflng chøa c⁄nh v• h…nh b…nh h nh ABCD, khi â mØi i”m An chuy”n th nh i”m An tr¶n c⁄nh AB. Vîi måi > 0 cho tr÷îc t…m ÷æc c¡c i”m Am v Am+k nh÷ th‚ sao cho kho£ng c¡ch giœa chóng nhä hìn . B¥y gií h¢y chøng minh kho£ng c¡ch tł i”m Ak ‚n mºt trong c¡c nót cıa l÷îi nhä hìn ( °c bi»t n‚u A0 m tròng vîi A0 m+k th… Ak s‡ l nót cıa l÷îi). K‚t qu£ sau l hi”n nhi¶n: Gi£ sß a; b l hai sŁ thüc b§t ký. Chøng minh r‹ng t“p hæp t§t c£ c¡c i”m vîi tåa º (ka; lb) vîi k; l 2 Z l mºt l÷îi. M»nh • 1.1. Vîi måi sŁ væ t > 0 v måi sŁ d÷ìng •u t…m ÷æc c¡c sŁ tü nhi¶n m; n sao cho jm nj < : Chøng minh. L§y h» tåa º Descartes tr¶n m°t phflng v k· ÷íng thflng y = x. V… l sŁ væ t n¶n tr¶n ÷íng thflng n y câ duy nh§t mºt i”m mang tåa º nguy¶n l i”m gŁc tåa º. ÷íng thflng y = x c›t ÷íng x = m t⁄i c¡c i”m (m; m ). Theo t‰nh ch§t v, t…m ÷æc nót (m; n) cıa l÷îi nguy¶n sao cho kho£ng c¡ch (theo chi•u thflng øng) tł â ‚n i”m (m; m ) nhä hìn . â l i•u cƒn chøng minh.
  • 12. Tải tài liệu tại sividoc.com Viết đề tài giá sinh viên – ZALO:0973.287.149-TEAMLUANVAN.COM 6 D„ th§y r‹ng trong t§t c£ c¡c kh£ n«ng th… kho£ng c¡c æi mºt giœa c¡c nót cıa l÷îi b§t ký s‡ l sŁ nhä nh§t. T‰nh ch§t n y còng vîi t‰nh ch§t ii: cıa l÷îi câ th” thay cho ành ngh¾a v• l÷îi. M»nh • 1.2. Gi£ sß t“p hæp M tr¶n m°t phflng câ c¡c t‰nh ch§t sau: (1) Kho£ng c¡ch cıa hai i”m b§t ký khæng nhä hìn sŁ d÷ìng d n o â; (2) N‚u 3 i”m A,B,C cıa M l c¡c ¿nh cıa h…nh b…nh h nh ABCD th… ¿nh thø t÷ D cıa h…nh b…nh h nh n y công thuºc t“p hæp M. Khi â M l mºt l÷îi. Chøng minh. H…nh 1.4: H…nh v‡ a. H…nh v‡ b L§y i”m B tòy þ thuºc M. Gi£ sß A l i”m trong M, gƒn B nh§t (H…nh v‡ a), i”m n y tçn t⁄i v… t§t c£ c¡c kho£ng c¡ch æi mºt giœa c¡c i”m cıa M •u lîn hìn d). Qua A v B k· mºt ÷íng thflng. Trong c¡c i”m cıa M khæng n‹m tr¶n ÷íng thflng n y chån ÷æc i”m gƒn B nh§t, gåi i”m â l C. Ta düng h…nh b…nh h nh ABCD. Theo i•u ki»n (2) i”m D công thuºc t“p hæp M. Ta düng l÷îi sinh bði h…nh b…nh h nh ABCD, ta i chøng minh t“p hæp M tròng vîi l÷îi vła düng.
  • 13. Tải tài liệu tại sividoc.com Viết đề tài giá sinh viên – ZALO:0973.287.149-TEAMLUANVAN.COM 7 Tł i•u ki»n (2) suy ra r‹ng t§t c£ c¡c nót cıa l÷îi n y •u thuºc M. Bði v“y ch¿ cƒn ki”m tra r‹ng ð bi¶n công nh÷ ð trong h…nh b…nh h nh ABCD khæng câ i”m n o cıa M kh¡c vîi ¿nh cıa nâ. i•u n y gƒn nh÷ hi”n nhi¶n: N‚u i”m S n‹m ð trong h…nh b…nh h nh (H…nh v‡ [[[[ b.)th… ‰t nh§t mºt trong c¡c gâc ASB; BSC; CSD; ASD l gâc tò ho°c gâc vuæng v bði v“y kho£ng c¡ch tł S tîi mºt trong c¡c ¿nh cıa nâ nhä hìn c⁄nh n o â cıa nâ(n‚u S n‹m tr¶n bi¶n th… công v“y). Chflng h⁄n gi£ sß â l kho£ng c¡ch SC. Ta düng h…nh b…nh h nh BCSS0 ; (S0 2 M). Khi â BS0 nhä hìn BC ho°c BA nh÷ng i•u â m¥u thu¤n vîi c¡ch chån ho°c cıa i”m C ho°c cıa i”m A. C¡c tr÷íng hæp cÆn l⁄i t÷ìng tü. B i to¡n 1.1. Gi£ sß l sŁ væ t tòy þ. Chøng minh r‹ng: vîi måi > 0 v måi sŁ thüc luæn t…m ÷æc c¡c sŁ m; n sao cho: jm +n j < . B i to¡n 1.2. Chøng minh r‹ng kþ hi»u th“p ph¥n cıa sŁ 2n câ th” b›t ƒu b‹ng chœ sŁ b§t ký cıa tŒ hæp c¡c chœ sŁ cho tr÷îc. 1.1.2 L÷îi tåa º nguy¶n v a gi¡c •u L§y mºt tí gi§y k· æ vuæng. Hi”n nhi¶n h…nh vuæng vîi c¡c ¿nh l c¡c nót cıa l÷îi luæn düng ÷æc v b‹ng væ sŁ c¡ch. C¡ch l m nh÷ th‚ câ thüc hi»n ÷æc Łi vîi tam gi¡c •u hay löc gi¡c •u hay khæng? Trong phƒn n y ta s‡ t…m c¥u tr£ líi th‰ch ¡ng. Gi£ sß ta câ mºt l÷îi nguy¶n hay cÆn gåi l l÷îi Z2 . Tr¶n l÷îi nguy¶n n y h¢y x†t b i to¡n v• sü tçn t⁄i a gi¡c •u nºi ti‚p ÷æc trong l÷îi. ành ngh¾a 1.1. Ta nâi a gi¡c nºi ti‚p ÷æc trong l÷îi n‚u tçn t⁄i mºt a gi¡c çng d⁄ng vîi nâ m c¡c ¿nh l nót cıa l÷îi. B i to¡n nºi ti‚p mºt tam gi¡c •u tr¶n l÷îi nguy¶n l b i to¡n ìn gi£n nh§t trong c¡c b i to¡n nºi ti‚p mºt a gi¡c •u tr¶n l÷îi nguy¶n. V“y m k‚t qu£ "khæng düng ÷æc tam gi¡c •u nºi ti‚p tr¶n l÷îi Z2 " ÷æc ÷a ra v o n«m 1878 do nh to¡n håc E. Lukacy cæng bŁ. — ¥y chóng tæi ÷a ra 5 c¡ch chøng minh k‚t qu£ n y: c¡ch thø nh§t (cıa Lukacy) sß döng lþ thuy‚t chia h‚t c¡c sŁ nguy¶n; c¡ch thø hai düa tr¶n
  • 14. Tải tài liệu tại sividoc.com tài giá sinh viên – ZALO:0973.287.149-TEAMLUANVAN.COM 8 2 t‰nh væ t cıa sŁ tan n , c¡ch thø ba dòng l÷æng gi¡c, c¡ch thø t÷ dòng ph÷ìng ph¡p di»n t‰ch, c¡ch thø n«m dòng ph÷ìng ph¡p cüc h⁄n. B i to¡n n y công ¢ l mºt trong nhœng b i to¡n hay trong ký thi håc sinh giäi quŁc gia v quŁc t‚ trong nhœng n«m gƒn ¥y. M»nh • 1.3. Tam gi¡c •u ABC khæng th” nºi ti‚p ÷æc trong l÷îi nguy¶n. Chøng minh. C¡ch 1. (Ph÷ìng ph¡p chia h‚t, cüc h⁄n) Gi£ sß düng ÷æc tam gi¡c •u tr¶n l÷îi Z2 v ta chån 1 tam gi¡c c⁄nh nhä nh§t, mºt ¿nh l gŁc tåa º, hai ¿nh kia câ tåa º (a; b); (c; d). Khi â bŁn sŁ nguy¶n a; b; c; d khæng câ ÷îc chung kh¡c 1 (tøc chóng nguy¶n tŁ còng nhau). Trong tr÷íng hæp ng÷æc l⁄i s‡ d¤n tîi tam gi¡c vîi tåa º c¡c ¿nh l
  • 15. Tải tài liệu tại sividoc.com tài giá sinh viên – ZALO:0973.287.149-TEAMLUANVAN.COM (0; 0); a ;b ; c ;d vîi k l ÷îc chung cıa bŁn sŁ a; b; c; d. Ta vi‚t k k k k flng thøc c¡c c⁄nh tam gi¡c vuæng ð d⁄ng tåa º: a2 + b2 = c2 + d2 = (a c)2 + (b d)2 : Suy ra: a2 +b2 = c2 +d2 = 2(ac+bd). Do â, a2 +b2 +c2 +d2 = 4(ac+bd). Ngh¾a l tŒng b…nh ph÷ìng bŁn sŁ nguy¶n chia h‚t cho 4. M°t kh¡c b…nh ph÷ìng cıa sŁ nguy¶n khi chia cho 4 ch¿ cho d÷ l 0 ho°c 1. Bði v“y, ho°c t§t c£ 4 sŁ a; b; c; d l chfin ho°c t§t c£ •u l·. Kh£ n«ng thø nh§t khæng x£y ra v… c¡c sŁ n y theo c¡ch chån ð tr¶n l nguy¶n tŁ còng nhau. Kh£ n«ng thø hai công khæng x£y ra v… lóc â khæng th” câ a2 + b2 = (a c)2 + (b d)2 . M¥u thu¤n n y suy ra i•u ph£i chøng minh. C¡ch 2. (Ph÷ìng ph¡p l÷æng gi¡c) Chó þ r‹ng n‚u 2 tia gŁc O k· qua c¡c nót (a; b) v (c; d) cıa l÷îi th… tang cıa gâc giœa hai tia n y l sŁ hœu t ho°c khæng x¡c ành v… d b tantan =ad bc tan = tan( ) = = c a : 1 + tan : tan db ac + bd 1 + ca
  • 16. Tải tài liệu tại sividoc.com Viết đề tài giá sinh viên – ZALO:0973.287.149-TEAMLUANVAN.COM 9 ÷æc tam gi¡c •u tr¶n l÷îi Z2 th… hai tia vîi gŁc l p mºt trong c¡c ¿nh tam gi¡c s‡ t⁄o th nh gâc 60 m tan 60 = 3 l sŁ væ t , tr¡i vîi v‚ ph£i l sŁ hœu t . C¡ch 3. (Ph÷ìng ph¡p di»n t‰ch) Ta t‰nh di»n t‰ch tam gi¡c ABC theo AB2 p AB:AC 3 2 c¡ch: Mºt m°t, S ABC = sin A = sin 60 = AB2 : 2 2 4 Nh÷ng AB2 = sŁ nguy¶n (AB2 = AC2 + CB2 ), ngh¾a l S ABC l sŁ væ t . M°t kh¡c, ngo⁄i ti‚p xung quanh tam gi¡c ABC l h…nh chœ nh“t CB’C’A’ ta nh“n ÷æc: S ABC = SCB0CA0 S AB0 C S AC0 B S A0 BC0 l sŁ hœu t v… t§t c£ c¡c di»n t‰ch ð v‚ ph£i •u l c¡c sŁ hœu t . Ta nh“n ÷æc S ABC vła l sŁ hœu t vła l sŁ væ t . Væ lþ. C¡ch 4. (Ph÷ìng ph¡p tåa º) Gi£ sß A(a1; a2), B(b1; b2), C(c1; c2) l 1 1 1 l mºt sŁ nguy¶n. c¡c i”m nguy¶n. Khi â di»n t‰ch 2S = a1 b1 c1 a2 b2 c2 2p AB 3 M°t kh¡c do ABC l tam gi¡c •u n¶n 2S = l sŁ væ t . M¥u 2 thu¤n â chøng minh b i to¡n. Tł ¥y ta s‡ vi‚t k gi¡c thay cho a gi¡c k c⁄nh, c¡c c¡ch nâi sau ¥y ÷æc sß döng th÷íng xuy¶n: 5-gi¡c hay ngô gi¡c; 8-gi¡c hay b¡t gi¡c; 10-gi¡c hay th“p gi¡c; 12-gi¡c hay th“p nhà gi¡c,. . . M»nh • 1.4. a gi¡c •u duy nh§t nºi ti‚p ÷æc trong l÷îi nguy¶n l h…nh vuæng. Chøng minh. D„ th§y tçn t⁄i h…nh vuæng nºi ti‚p l÷îi, chflng h⁄n â l h…nh vuæng cì sð, câ tåa º c¡c ¿nh (0; 0); (0; 1); (1; 0); (1; 1). Ti‚p theo ta chøng minh b‹ng ph£n chøng r‹ng ngo i h…nh vuæng khæng cÆn a gi¡c •u n o kh¡c nºi ti‚p ÷æc trong l÷îi nguy¶n. Gi£ sß D l a gi¡c nºi ti‚p ÷æc cıa l÷îi nguy¶n (tøc c¡c ¿nh •u l nót). X£y ra c¡c kh£ n«ng sau: a. D l tam gi¡c •u ABC. b. D l ngô gi¡c •u ABCDE:
  • 17. Tải tài liệu tại sividoc.com Viết đề tài giá sinh viên – ZALO:0973.287.149-TEAMLUANVAN.COM 10 c. D l löc gi¡c •u ABCDEF: d. D l n-gi¡c •u vîi n 7: Ta x†t tłng kh£ n«ng: a. Xem M»nh • 1.3 b. Gi£ sß ngô gi¡c •u ABCDE câ c⁄nh b‹ng a. Gåi A0 = AC BE th… A0 l nót cıa l÷îi. Th“t v“y, v… AA0 DE l h…nh b…nh h nh n¶n AA0 =! 0 công l ! ED. Tł ¥y do E, D, A l c¡c i”m nguy¶n n¶n A i”m nguy¶n. Kþ hi»u B0 ; C0 ; D0 ; E0 t÷ìng tü nh÷ A0 th… B0 ; C0 ; D0 ; E0 công l c¡c i”m nguy¶n. V“y ngô gi¡c A0 B0 C0 D0 E0 l ngô gi¡c nºi ti‚p ÷æc. Ta câ A’B’=C’D’=D’E’=E’A’. M°t kh¡c, ABE0 = BA0 C suy 000 000 000 000 000 ra A E D = B A E . T÷ìng tü, ta câ: A E D = B A E = C B A = D CB 0 = C DE . Tł â suy ra A 0 B 0 C 0 D 0 E 0 l ngô gi¡c •u (nºi ti‚p 0 0 0 0 0 ÷æc tr¶n l÷îi) câ c⁄nh b < a, (H…nh 1.5). H…nh 1.5: Ngô gi¡c v löc gi¡c Ti‚p töc qu¡ tr…nh tr¶n ta s‡ thu ÷æc væ sŁ i”m nguy¶n b¶n trong ngô gi¡c ABCDE. i•u n y væ lþ v… trong mºt mi•n giîi h⁄n ch¿ tçn t⁄i mºt sŁ hœu h⁄n i”m nguy¶n. V“y khæng tçn t⁄i ngô gi¡c •u nºi ti‚p ÷æc l÷îi nguy¶n. c. N‚u D l löc gi¡c •u A1B1C1D1E1F1. Trong löc gi¡c •u tçn t⁄i tam gi¡c •u câ ¿nh nguy¶n. Væ lþ d. N‚u D l n-gi¡c •u vîi n 7: Gi£ sß A1A2 : : : An l a gi¡c •u
  • 18. Tải tài liệu tại sividoc.com Viết đề tài giá sinh viên – ZALO:0973.287.149-TEAMLUANVAN.COM 11 nguy¶n ( a gi¡c •u câ ¿nh l c¡c i”m nguy¶n hay c¡c nót l÷îi) câ c⁄nh nhä nh§t l a. Gåi O l ! OB1 ! 1 2 ! 2 = gŁc tåa º. °t = A A ;OB ! A2A3 ! OBn ! n A 1 1 B 2 :::B n l a gi¡c •u theo c¡ch ;:::; = A . Ta suy ra: B düng v l a gi¡c nguy¶n. Khi â B1B2 = B2B3 = : : : = BnB1 = b. b Trong tam gi¡c OB1B2, gåi B1OB2 = th… sin 2 = 2a . M°t kh¡c, = 2 n ; n 7 n¶n < 60 =) sin 2 < 1 2 =) 2 b a < 1 2 =) b < a. M¥u thu¤n vîi gi£ thi‚t A1A2 : : : An l a gi¡c nguy¶n câ c⁄nh nhä nh§t. M»nh • ÷æc chøng minh ho n to n. Nh“n x†t 1.1.1. C¡ch chøng minh nh÷ tr¶n ÷æc gåi l ¡p döng nguy¶n t›c cüc h⁄n. GiŁng nh÷ nguy¶n t›c Dirichle, c£ hai •u l ph÷ìng ph¡p chøng minh ph£n chøng. Nºi dung cıa ph÷ìng ph¡p n y l ¡p döng nguy¶n t›c sau: "Trong mºt t“p hæp hœu h⁄n (kh¡c rØng) c¡c sŁ thüc tçn t⁄i sŁ nhä nh§t v tçn t⁄i sŁ lîn nh§t". Nhí nguy¶n lþ cüc h⁄n n y ta câ th” x†t c¡c phƒn tß m mºt ⁄i l÷æng n o â câ gi¡ trà nhä nh§t ho°c gi¡ trà lîn nh§t. Chflng h⁄n: X†t o⁄n thflng câ º d i lîn nh§t (nhä nh§t) trong mºt sŁ hœu h⁄n c¡c o⁄n thflng. X†t gâc câ sŁ o lîn nh§t (nhä nh§t) trong mºt sŁ hœu h⁄n c¡c gâc. X†t a gi¡c câ di»n t‰ch ho°c chu vi lîn nh§t (nhä nh§t) trong mºt sŁ hœu h⁄n c¡c a gi¡c. X†t kho£ng c¡ch lîn nh§t (nhä nh§t) trong mºt sŁ hœu h⁄n c¡c kho£ng c¡ch. Nh÷ v“y sŁ c¡c a gi¡c •u nºi ti‚p ÷æc tr¶n l÷îi nguy¶n r§t hi‚m hoi. Câ hai c¡ch mð rºng t‰nh nºi ti‚p tr¶n l÷îi nguy¶n cıa a gi¡c nh÷ th‚: Mºt l ta x†t c¡c a gi¡c nßa •u ( •u c⁄nh ho°c •u gâc), hai l ta giœ nguy¶n c¡c a gi¡c •u nh÷ng t…m c¡ch "nºi ti‚p x§p x¿", hay cÆn gåi l "hƒu nºi ti‚p". Phƒn ti‚p theo s‡ i theo hai h÷îng nh÷ v“y.
  • 19. Tải tài liệu tại sividoc.com Viết đề tài giá sinh viên – ZALO:0973.287.149-TEAMLUANVAN.COM 12 1.1.3 a gi¡c nßa •u nºi ti‚p l÷îi nguy¶n Kh¡i ni»m a gi¡c nßa •u ÷æc chia l m hai lo⁄i: •u theo c⁄nh v •u theo gâc (xem [4]). ành ngh¾a 1.2. Mºt a gi¡c gåi l nßa •u n‚u nâ câ c¡c gâc b‹ng nhau ho°c c¡c c⁄nh b‹ng nhau. a gi¡c câ c¡c gâc b‹ng nhau, c¡c c⁄nh câ th” khæng b‹ng nhau, ÷æc gåi l a gi¡c nßa •u-gâc, kþ hi»u t“p hæp â l G; a gi¡c câ c¡c c⁄nh b‹ng nhau, c¡c gâc câ th” khæng b‹ng nhau, ÷æc gåi l a gi¡c nßa •u-c⁄nh, kþ hi»u t“p hæp â l C. D„ th§y giao GC l t“p hæp c¡c a gi¡c •u. Chó þ r‹ng trong ành ngh¾a tr¶n ta khæng nâi g… ‚n a gi¡c lçi hay khæng. C¡c a gi¡c nßa •u-gâc nºi ti‚p tr¶n l÷îi l h…nh vuæng v h…nh b¡t gi¡c. H…nh 1.6: H…nh vuæng v h…nh b¡t gi¡c a gi¡c giîi h⁄n bði ÷íng g§p khóc kh†p k‰n vîi º d i c¡c c⁄nh b‹ng 1 v gâc giœa hai o⁄n b‹ng 1v l tr÷íng hæp °c bi»t cıa a gi¡c nßa •u c⁄nh. Ta x†t mºt sŁ d⁄ng. a. Tam gi¡c •u. B i to¡n nºi ti‚p mºt tam gi¡c •u tr¶n l÷îi nguy¶n l b i to¡n ìn gi£n nh§t trong c¡c b i to¡n nºi ti‚p a gi¡c •u. V“y m k‚t qu£ l khæng düng ÷æc tam gi¡c •u nºi ti‚p tr¶n l÷îi Z2 , ÷æc cæng bŁ ngay tł n«m 1878 bði nh to¡n håc E. Lukacy. b. a gi¡c •u-c⁄nh. M»nh • 1.5. ( ành lþ Boll 1, [4])
  • 20. Tải tài liệu tại sividoc.com Viết đề tài giá sinh viên – ZALO:0973.287.149-TEAMLUANVAN.COM 13 i. Tr¶n l÷îi nguy¶n Z2 khæng th” düng ÷æc a gi¡c •u-c⁄nh vîi sŁ c⁄nh l sŁ l·. ii. Tr¶n l÷îi nguy¶n Z2 câ th” düng ÷æc a gi¡c •u-c⁄nh vîi sŁ c⁄nh l sŁ chfin. Chøng minh. i. Ph£n chøng. Gi£ sß câ düng ÷æc a gi¡c •u-c⁄nh vîi sŁ c⁄nh l·. Ta s‡ coi a gi¡c n y câ º d i c⁄nh lîn nh§t. Gi£ sß ! ~ ~ vk = xki + ykj; (k = 1; 2; : : : ; n) l c¡c v†c tì câ h÷îng dåc theo c¡c ~ câ º d i cıa chóng kh¡c c⁄nh a gi¡c. TŒng c¡c v†c tì n y b‹ng 0 v nhau. Ngh¾a l x1 + x2 + + xn = 0 y1 + y2 + + yn = 0 x2 1 + y1 2 = x2 2 + y2 2 = = x2 n + yn 2 = a2 : trong â, a l º d i c⁄nh a gi¡c ¢ cho. B…nh ph÷ìng hai v‚ 2 flng thøc ƒu, cºng l⁄i ta thu ÷æc n:a2 = X 2 (xixj + yiyj): i6=j Tł ¥y, v… a2 l sŁ tü nhi¶n v n l· n¶n a2 l sŁ chfin. . N‚u a 2 . 4 th… khi â t§t c£ c¡c xi; yi l c¡c sŁ chfin v… tŒng æi mºt . c¡c b…nh ph÷ìng chia h‚t cho 4. i•u â l khæng th” v… trong c¡c v†c 1 tì 2v~i, tåa º nguy¶n, câ th” t⁄o th nh a gi¡c •u-c⁄nh gçm n c⁄nh. a gi¡c n y câ º d i c⁄nh b‹ng mºt nßa º d i c⁄nh a gi¡c ¢ chån. i•u n y væ lþ theo nguy¶n t›c cüc h⁄n. B¥y gií gi£ sß a2 .. . 2 v a2 khæng chia h‚t cho 4. Lóc n y c¡c xi; yi l c¡c sŁ l· v… chóng thäa m¢n ph÷ìng tr…nh xi 2 + yi 2 = a 2 . Nh÷ v“y tŒng P . 2 . (xixj + yiyj) l sŁ chfin v bði v“y a . 4. Væ lþ. i6=l ii. H…nh 1.7 ch¿ ra h…nh vuæng gh†p vîi löc gi¡c, ta nh“n ÷æc mºt th“p gi¡c (sŁ c⁄nh l 10) câ c¡c c⁄nh b‹ng nhau, khæng lçi. Tuy nhi¶n câ th” chøng minh ÷æc r‹ng tr¶n l÷îi nguy¶n düng ÷æc mºt a gi¡c •u-c⁄nh lçi vîi sŁ c⁄nh l sŁ chfin.
  • 21. Tải tài liệu tại sividoc.com Viết đề tài giá sinh viên – ZALO:0973.287.149-TEAMLUANVAN.COM 14 H…nh 1.7: a gi¡c •u-c⁄nh c. a gi¡c •u gâc. ” nh“n ÷æc k‚t qu£ v• a gi¡c •u-gâc ta câ nh“n x†t sau: C¡c sŁ tan 2 n vîi n 2 N l sŁ væ t trł c¡c sŁ øng vîi n = 1; 2; 4; 8. ” câ k‚t qu£ â ta ¡p döng cæng thøc tann = Cn 1 tanCn 3 tan3 + Cn 5 tan5 + + ( 1) n 1 tann 2 n 1 1 Cn 2 tan2 + Cn 4 tan4 + ( 1) tann 2 (1.1) trong â, Ck l h» sŁ trong khai tri”n nhà thøc Newton: n Ph†p chøng minh cæng thøc (1.1) nh“n ÷æc nhí quy n⁄p to¡n håc. Ta s‡ coi n 5 v n = 8 v… c¡c tr÷íng hæp n = 1; 2; 3; 4 l hi”n nhi¶n. Gi£ sß = 2 . Khi â tan n = 0 v theo (1.1) sŁ x = tan 2 l n n nghi»m cıa ph÷ìng tr…nh vîi h» sŁ nguy¶n n Cn 3 x2 + Cn 5 x4 + ( 1) n 1 xn 1 = 0 (1.2) 2 Trong ph÷ìng tr…nh n y h» sŁ cao nh§t l 1, do â, nghi»m hœu t cıa nâ ph£i l sŁ nguy¶n. Bði v“y n‚u x l nghi»m hœu t th… x ph£i l sŁ nguy¶n. X†t 2 tr÷íng hæp: Tr÷íng hæp 1. N‚u n = mp vîi p l sŁ nguy¶n tŁ l·. Câ hai kh£ n«ng: 2 N‚u m = 1 tøc n = p th… tł (1.2) rót ra x2 jp v do â, x = tan n = 1.
  • 22. Tải tài liệu tại sividoc.com Viết đề tài giá sinh viên – ZALO:0973.287.149-TEAMLUANVAN.COM 15 Gi£ sß tan 2 l sŁ hœu t th… nh÷ ¢ th§y trong cæng thøc (1.1), sŁ 2 m n 2 tan = tan công l sŁ hœu t . Væ lþ. n n 2 p k 4. V… tan Tr÷íng hæp 2. B¥y gií, gi£ sß n = 2 ; k = 2 1 6 l sŁ væ t n¶n theo cæng thøc (1.1), t§t c£ c¡c sŁ d⁄ng tan 2 ; k 4 2k công l sŁ væ t . Khflng ành ÷æc chøng minh. Chó þ r‹ng, hai tr÷íng hæp n y l t§t c£ c¡c kh£ n«ng câ th” x£y ra. M»nh • 1.6. ( ành lþ Boll 2, [4]) Trong t§t c£ c¡c a gi¡c •u-gâc ch¿ düng ÷æc h…nh chœ nh“t v h…nh b¡t gi¡c nºi ti‚p tr¶n l÷îi nguy¶n Chøng minh. Ta s‡ coi n 4 v… tr÷íng hæp tam gi¡c ¢ ÷æc chøng minh. H…nh vuæng v b¡t gi¡c ¢ düng ÷æc nh÷ tr¶n h…nh v‡. B¥y gií gi£ sß a gi¡c •u-gâc vîi n > 4 ¿nh l düng ÷æc tr¶n l÷îi nguy¶n Z2 . Khi â, c¡c v†c tì bi”u di„n c¡c c⁄nh cıa nâ câ tåa º nguy¶n v gâc 2 giœa 2 v†c tì cıa hai c⁄nh k• nhau b§t ký •u b‹ng n . Ta s‡ coi O l gŁc tåa º. Khi â, gâc giœa [OP) v [OQ) b‹ng 2 v tan 2 nh÷ ¢ n n th§y ph£i l sŁ hœu t khi n > 4. Nh÷ng i•u n y x£y ra ch¿ khi n = 8, xem h…nh v‡ 1.8. H…nh 1.8: a gi¡c •u-gâc
  • 23. Tải tài liệu tại sividoc.com Viết đề tài giá sinh viên – ZALO:0973.287.149-TEAMLUANVAN.COM 16 C¡c b i to¡n sau ¥y bŒ sung th¶m cho kh¡i ni»m a gi¡c •u, nßa •u tr¶n l÷îi nguy¶n. B i to¡n 1.3. Tr¶n gi§y æ vuæng v‡ mºt ÷íng g§p khóc kh†p k‰n câ c¡c ¿nh l c¡c nót. Chøng minh r‹ng sŁ c¡c o⁄n g§p khóc l chfin. B i to¡n 1.4. Chøng minh r‹ng tr¶n l÷îi Z2 câ th” düng ÷æc a gi¡c lçi •u c⁄nh vîi sŁ c⁄nh l sŁ chfin tòy þ. B i to¡n 1.5. Chøng minh r‹ng a gi¡c •u duy nh§t tr¶n m°t phflng m t§t c£ c¡c ¿nh câ tåa º hœu t l h…nh vuæng. B i to¡n 1.6. Chøng minh flng thøc (1.1) nhí ¡p döng cæng thøc Moivre (Moa-vì-rì): cos n + i: sin n = (cos + i: sin )n : B i to¡n 1.7. a. Chøng minh r‹ng n‚u p v q l c¡c sŁ tü nhi¶n nguy¶n tŁ còng nhau, cos(p =q) l sŁ hœu t th… sŁ cos( =q) công l sŁ hœu t . b.Chøng minh r‹ng n‚u sŁ cos(p =q) vîi p,q nguy¶n tŁ còng nhau v q > 3 l sŁ hœu t khi v ch¿ khi sŁ cos( =q) l sŁ hœu t . c. Gi£ sß p,q nguy¶n tŁ còng nhau, q 3. Chøng minh r‹ng c¡c sŁ sin(p =q) l sŁ væ t vîi q 6= 6 Ph÷ìng ph¡p gi£i: Sß döng k‚t qu£: 2 cos n câ th” vi‚t ÷æc d÷îi d⁄ng fn(2 cos ), trong â, fn(x) l a thøc vîi h» sŁ nguy¶n v h» sŁ cao nh§t b‹ng 1. 1.2 a gi¡c hƒu nºi ti‚p l÷îi nguy¶n Ta x†t b i to¡n thø hai: a gi¡c hƒu nºi ti‚p tr¶n l÷îi nguy¶n. ành ngh¾a 1.3. a gi¡c gåi l nºi ti‚p trong l÷îi M n‚u tçn t⁄i mºt a gi¡c çng d⁄ng vîi nâ (vîi h» sŁ çng d⁄ng l sŁ tü nhi¶n) sao cho kho£ng c¡ch tł mØi ¿nh ‚n nót nhä hìn . Chó þ r‹ng, khi = 0 ta nh“n ÷æc ành ngh¾a a gi¡c nºi ti‚p l÷îi ð tr¶n, cÆn khi qu¡ lîn (lîn hìn kho£ng c¡ch giœ hai nót) th… ành ngh¾a khæng x¡c ành.
  • 24. Tải tài liệu tại sividoc.com Viết đề tài giá sinh viên – ZALO:0973.287.149-TEAMLUANVAN.COM 17 Hìn nœa, ta s‡ gåi a gi¡c hƒu kh›p nºi ti‚p l÷îi M n‚u nâ nºi ti‚p trong l÷îi vîi måi > 0. Rª r ng, a gi¡c s‡ hƒu kh›p nºi ti‚p n‚u nâ nºi ti‚p ÷æc trong l÷îi. i•u ng÷æc l⁄i khæng óng: M»nh• 1.7. Tam gi¡c •u hƒu kh›p nºi ti‚p trong l÷îi nguy¶n. H…nh 1.9: Tam gi¡c •u hƒu nºi ti‚p l÷îi Chøng minh. X†t tam gi¡c •u c⁄nh 2 nh÷ trong H…nh 1.9. Vîi ph†p và tü t¥m O, t sŁ và tü k tü nhi¶n (kh¡c 0), A1B1C1 bi‚n th nh AkBkCk, trong â, Ak; Bk l c¡c nót cıa l÷îi cÆn Ck câ t⁄o º (0; k p 3) (tung p ). V… Ak; Bk l º yk cıa Ck b‹ng yk = ky 1 = k:OC1 = k 3 c¡c nót vîi måi k n¶n b i to¡n quy v• t…m k ” kho£ng c¡ch tł Ck ‚n nót gƒn nh§t nhä hìn . L§y sŁ tü nhi¶n N sao cho N > 1 v chia kho£ng [0,1) th nh N nßa kho£ng câ º d i nhä hìn : N ;:::; ; 1 : 0; N ; N ; N 1 1 2 N 1 B¥y gií chó þ r‹ng tr¶n o⁄n [0; 1] câ c¡c i”m fk p 3g, trong â, fxg p kþ hi»u phƒn ph¥n cıa x; k = 1; 2; : : : ; N + 1. MØi i”m fk 3g rìi v o mºt trong c¡c nßa kho£ng. V… câ N nßa kho£ng m N + 1 i”m n¶n câ hai i”m n o â rìi v o còng mºt nßa kho£ng. Gi£ sß â l i”m p p p fm 3g; fn 3g vîi m > n l c¡c sŁ tü nhi¶n. V… (m n) 3 kh¡c sŁ
  • 25. Tải tài liệu tại sividoc.com Viết đề tài giá sinh viên – ZALO:0973.287.149-TEAMLUANVAN.COM 18 1 nguy¶n nhä hìn N v do â nhä hìn . B‹ng c¡ch °t k b‹ng m n ta nh“n ÷æc i•u cƒn t…m. M»nh • ÷æc chøng minh. Tuy nhi¶n x£y ra mºt sü ki»n tŒng qu¡t: M»nh • 1.8. ( ành lþ cì b£n) Måi a gi¡c •u câ th” hƒu kh›p nºi ti‚p trong mºt l÷îi b§t ký. Chøng minh. Gi£ sß cho l÷îi M, sŁ > 0 v a gi¡c . Ta chøng minh r‹ng qua mºt ph†p và tü n o â vîi t sŁ và tü l sŁ tü nhi¶n N, a gi¡c bi‚n th nh a gi¡c 0 sao cho måi ¿nh cıa a gi¡c 0 •u c¡ch i”m nót mºt kho£ng nhä hìn . X†t ¿nh A cıa a gi¡c. Gi£ sß qua ph†p và tü t¥m O, h» sŁ và tü k i”m A bi‚n th nh i”m v OA = x ~a+y ~ ~ Ak A b, trong â, ~a; b l c¡c v†c ! A ! ~ ~ tì sinh ra l÷îi. Khi â, OAk = k(xA~a + yAb) = (kxA)~a + (kyA)b. i•u ki»n Ak c¡ch i”m nót cıa l÷îi kho£ng c¡ch nhä hìn s‡ ÷æc thüc hi»n n‚u mØi tåa º (kxA) v (kyA) s‡ kh¡c sŁ nguy¶n nhä hìn c , vîi h» sŁ c khæng phö thuºc v o k, câ th” l§y chflng h⁄n c = 1 . ~ ! ~ 2 max(j~aj; jbj) j j j Aj jj 2 Th“t v“y, gi£ sß OF = m~a + nb. Khi â, n‚u m ~a kx ~a < ~ ~ ! OA ~ j j j kx Aj jj < 2 k F j k F j j OF kj j m~a nb v m b b th… A A = = + = ! ! ~ k(xA~a yAb)j < . ~ ~ Nh÷ng c¡c b§t flng thøc jmj~aj kxAj~ajj < 2 v jnjbj kyAjbjj <2 t÷ìng ÷ìng vîi c¡c b§t flng thøc jm kxAj < v jn kyAj < . 2 ~a ~ j j 2 b Bði v“y n‚u kxA v kyA kh¡c sŁ nguy¶n nhä hìn j j th… ~ AkF < vîi i”m nót F n o â v 2 max(j~aj; jbj) ta câ i•u Æi häi. Ta cÆn ph£i khflng ành t§t c£ c¡c ¿nh cıa a gi¡c 0 •u ð trong l¥n c“n cıa i”m nót cıa l÷îi M. Khflng ành â câ n‚u t§t c£ c¡c sŁ N xA; N yA; N xB; N yB : : : kh¡c c¡c sŁ nguy¶n nhä hìn sŁ N(xA; yA; xB; yB; : : :)- h» sŁ trong ph¥n t‰ch c¡c ! ! ! ! b ;. SŁ N ÷æc x¡c ành nhí ành v†c tì OA; OB; : : : theo c¡c v†c tì a ; lþ Kroneker: "Gi£ sß 1; 2; : : : ; n l c¡c sŁ thüc. Khi â vîi måi sŁ
  • 26. Tải tài liệu tại sividoc.com Viết đề tài giá sinh viên – ZALO:0973.287.149-TEAMLUANVAN.COM 19 d÷ìng t…m ÷æc sŁ tü nhi¶n m sao cho mØi sŁ m i, i = 1; 2; : : : n kh¡c sŁ nguy¶n kho£ng , tøc l : fm ig 2 [0; ] [ [1 ; 1]" (Xem [6]). B i to¡n 1.8. Tr¶n gi§y æ vuæng v‡ mºt ÷íng g§p khóc câ ¿nh l c¡c nót sao cho c¡c o⁄n thflng •u b‹ng nhau. Chøng minh r‹ng sŁ c¡c o⁄n thflng l sŁ chfin. B i to¡n 1.9. Chøng minh r‹ng tr¶n l÷îi Z2 câ th” °t a gi¡c lçi •u c⁄nh vîi sŁ c⁄nh chfin b§t ký. 1.3 Cæng thøc Picard 1.3.1 Tam gi¡c ìn nguy¶n thıy Ng÷íi ta ph¥n lo⁄i a gi¡c th nh a gi¡c ìn v a gi¡c khæng ìn ( a gi¡c phøc) nh÷ sau: a gi¡c ìn (Simple polygon): a gi¡c m c¡c c⁄nh ch¿ câ th” c›t nhau t⁄i c¡c ƒu mót ( ¿nh a gi¡c), khæng câ hai c⁄nh khæng k• nhau c›t nhau. a gi¡c ìn câ th” l a gi¡c lçi ho°c a gi¡c lªm. a gi¡c khæng ìn hay a gi¡c phøc hay(Complex polygon) l a gi¡c câ hai c⁄nh khæng k• nhau c›t nhau, i”m c›t nhau â khæng ph£i l ¿nh cıa a gi¡c. Chó þ r‹ng a gi¡c ìn câ th” l a gi¡c lªm ho°c a gi¡c lçi. a gi¡c phøc l a gi¡c lªm. H…nh v‡ sau ¥y ch¿ ra a gi¡c ìn v khæng ìn: H…nh 1.10: a gi¡c ìn v a gi¡c khæng ìn
  • 27. Tải tài liệu tại sividoc.com Viết đề tài giá sinh viên – ZALO:0973.287.149-TEAMLUANVAN.COM 20 1.3.2 Cæng thøc Picard Mºt øng döng trüc ti‚p cıa l÷îi nguy¶n l t‰nh di»n t‰ch h…nh phflng b‹ng c¡ch phı nâ bði l÷îi æ vuæng. Cæng thøc sau ¥y mang t¶n nh to¡n håc ng÷íi Ph¡p Picard, ÷æc t…m ra v o n«m 1899. M»nh • 1.9. (Cæng thøc Picard) Cho a gi¡c ìn D câ c¡c ¿nh l c¡c i”m nguy¶n. SŁ i”m nguy¶n tr¶n c¡c c⁄nh l m, sŁ i”m nguy¶n b¶n trong a gi¡c l n. Khi â di»n t‰ch cıa D b‹ng SD = n + m 1: (1.3) 2 Chøng minh. — ¥y tr…nh b y hai c¡ch chøng minh cæng thøc Picard: C¡ch chøng minh truy•n thŁng v c¡ch chøng minh trüc ti‚p(tham kh£o trong [3]). B i to¡n d¤n ra sau ¥y. C¡ch 1. Cì sð cıa vi»c chøng minh cæng thøc Picard l x¡c ành di»n t‰ch cıa h…nh tam gi¡c ìn nguy¶n thıy (câ c¡c ¿nh nguy¶n v khæng chøa b¶n trong nâ ho°c tr¶n bi¶n mºt i”m nguy¶n n o). C¡c b÷îc chøng minh qua c¡c k‚t qu£ sau: B i to¡n 1.10. Trong mºt a gi¡c ìn vîi sŁ ¿nh lîn hìn ho°c b‹ng 4 luæn tçn t⁄i ÷íng ch†o n‹m ð mi•n trong cıa nâ. B i to¡n 1.11. Câ th” chia n-gi¡c ìn nguy¶n thıy th nh n gi¡c sao cho c¡c tam gi¡c n y khæng câ i”m trong chung v cıa tam gi¡c n‹m trong n-gi¡c â. 2 tam c¡c c⁄nh B i to¡n 1.12. Cho mºt a gi¡c ìn D m måi ¿nh l c¡c i”m nguy¶n. Khi â câ th” nŁi c¡c i”m nguy¶n n‹m trong D v tr¶n c⁄nh cıa D ” ÷æc c¡c tam gi¡c ìn. B i to¡n 1.13. SŁ c¡c tam gi¡c ìn trong mºt tam gi¡c ph¥n cıa a gi¡c ìn D l b‹ng nhau vîi måi ki”u tam gi¡c ph¥n. N‚u b¶n trong D câ k i”m nguy¶n v tr¶n c⁄nh cıa D (k” c£ ¿nh) câ m i”m nguy¶n th… sŁ c¡c tam gi¡c ìn l s = 2k + m 2 Tł c¡c k‚t qu£ tr¶n ta thu ÷æc m»nh • cì b£n sau:
  • 28. Tải tài liệu tại sividoc.com Viết đề tài giá sinh viên – ZALO:0973.287.149-TEAMLUANVAN.COM 21 M»nh • 1.10. Cho ABC l tam gi¡c ìn nguy¶n thıy b§t ký tr¶n l÷îi nguy¶n mØi æ vuæng câ di»n t‰ch 1 ìn và. Khi â SABC = 1 . ” 2 chøng minh SABC = 1 ta s‡ chøng minh hai b§t flng:SABC 1 v 2 2 1 S ABC . 2 Phı tam gi¡c ABC bði h…nh chœ nh“t MNPQ. Khi â, SABC = SMNPQ- l m tŒng di»n t‰ch c¡c tam gi¡c phö vîi ABC = k = vîi k; l; m 2 Z. 2 2 m 1 V“y S ABC = 2 2 : X†t c¡c tam gi¡c ph¥n Łi vîi tam gi¡c phö vîi tam gi¡c ABC trong h…nh chœ nh“t MNPQ. Khi â ta câ tam gi¡c ph¥n cho h…nh chœ nh“t MNPQ vîi sŁ tam gi¡c t⁄o th nh l s 2(m 1)(n 1)+2(m+n) 2 = 2m:n, trong â m; n l hai c⁄nh h…nh chœ nh“t. V… di»n t‰ch h…nh chœ nh“t MNPQ b‹ng m:n n¶n tł flng thøc â suy ra måi tam gi¡c ph¥n 1 •u ph£i câ di»n t‰ch 2. Ta câ i•u ph£i chøng minh. CuŁi còng, theo B i to¡n 1.13 ta câ th” chia D th nh 2k + m 2 tam gi¡c ìn bði c¡c o⁄n thflng nŁi c¡c i”m nguy¶n n‹m trong ho°c tr¶n bi¶n cıa nâ. Theo M»nh • 1.10 mØi mºt tam gi¡c ph¥n câ di»n t‰ch 1 b‹ng . V“y di»n t‰ch cıa a gi¡c D l 2 (2k + m 2): 1 = k + m 1: 2 2 Cæng thøc ÷æc chøng minh C¡ch 2. (Tham kh£o trong [2]) Chøng minh. Vîi mØi a gi¡c D, vîi quy ÷îc m; k nh÷ gi£ thi‚t ta kþ hi»u h m f(D) = k + m 1. Gi£ sß a gi¡c M ÷æc c›t th nh hai a 2 gi¡c M1; M2 vîi c¡c ¿nh l c¡c nót l÷îi. Ta s‡ chøng minh r‹ng cæng thøc Picard n‚u óng Łi vîi 2 trong 3 a gi¡c M; M1; M2 th… công s‡ óng Łi vîi a gi¡c cÆn l⁄i. ” câ i•u â ta i chøng minh flng thøc f(M) = f(M1) + f(M2) (1.4)
  • 29. Tải tài liệu tại sividoc.com Viết đề tài giá sinh viên – ZALO:0973.287.149-TEAMLUANVAN.COM 22 C¡ch l“p lu“n ð ¥y kh¡ °c bi»t nh÷ sau (xem [2]) + Gåi o⁄n thflng c›t a gi¡c M th nh 2 a gi¡c M1; M2 l L. C¡c nót n‹m ngo i L cæng l⁄i b‹ng f(M) v f(M1) + f(M2); c¡c nót khæng ph£i ƒu mót cıa L cºng vîi 1 th nh f(M) v cºng vîi 0,5 th nh f(M1) v f(M2). MØi mºt trong hai i”m nót m l ƒu mót cıa L cºng th¶m 0,5 v o mØi sŁ f(M); f(M1); f(M2) v do â phƒn th¶m cıa c¡c nót t⁄i 2 ƒu mót L v sŁ f(M) ‰t hìn 1 so vîi phƒn th¶m cıa c¡c nót t⁄i 2 ƒu mót L v o f(M1) + f(M2). V… chóng ta ph£i gi£m i 1 tł mØi lƒn th¶m v o f(M) v gi£m i 2 tł mØi lƒn th¶m v o f(M1) + f(M2) n¶n suy ra f(M) = f(M1) + f(M2). + B¥y gií ta i chøng minh cæng thøc Picard Łi vîi tam gi¡c tòy þ. N‚u M l h…nh chœ nh“t vîi c¡c c⁄nh p; q th… f(M) = (p 1)(q 1)+ 2(p + q) 1 = pq = Shcn, tøc l cæng thøc Picard óng Łi vîi h…nh 2 chœ nh“t. C›t h…nh chœ nh“t M th nh hai h…nh tam gi¡c (vuæng) M1; M2 b‹ng ÷íng ch†o cıa nâ v sß döng k‚t qu£ f(M) = f(M1) + f(M2) v f(M1) = f(M2) ta suy ra cæng thøc Picard óng Łi vîi tam gi¡c vuæng câ hai c⁄nh gâc vuæng thuºc ÷íng thflng l÷îi. + B¥y gií tł tam gi¡c b§t ký, phı nâ b‹ng mºt h…nh chœ nh“t ta suy ra i•u ph£i chøng minh. + CuŁi còng chó þ r‹ng måi a gi¡c •u câ th” c›t th nh c¡c tam gi¡c bði c¡c ÷íng ch†o. Ta câ i•u ph£i chøng minh. 1.3.3 Mºt sŁ b i to¡n ¡p döng cæng thøc Picard B i to¡n 1.14. Trong tam gi¡c ABC câ óng mºt i”m nguy¶n. Chøng minh r‹ng i”m â l trång t¥m tam gi¡c. Trong tam gi¡c ABC câ óng mºt i”m nguy¶n D. Khi â tam gi¡c ABC ÷æc ph¥n chia th nh 3 tam gi¡c ABD, ACD, BCD m mØi tam gi¡c khæng chøa i”m nguy¶n n o ngo i ¿nh. Theo m»nh • tr¶n SABC = 1 SACD = SBCD = 2. Suy ra kho£ng c¡ch tł B v C ‚n AD b‹ng nhau, v D thuºc ÷íng trung tuy‚n k· tł A. T÷ìng tü, D n‹m tr¶n trung tuy‚n k· tł B. Ta k‚t lu“n D l trång t¥m tam gi¡c ABC.
  • 30. Tải tài liệu tại sividoc.com Viết đề tài giá sinh viên – ZALO:0973.287.149-TEAMLUANVAN.COM 23 B i to¡n 1.15. a. H¢y t‰nh di»n t‰ch nhä nh§t cıa ngô gi¡c lçi nguy¶n câ th” ⁄t ÷æc. b. N‚u ngô gi¡c tr¶n khæng lçi th… di»n t‰ch nhä nh§t b‹ng bao nhi¶u? Theo V‰ dö 1.4.3 mºt a gi¡c lçi nguy¶n câ ‰t nh§t mºt i”m nguy¶n l i”m trong, ngh¾a k 1; m 5. V“y di»n t‰ch ngô gi¡c b‹ng k+ m 1 2 1+ 5 1 =5 . M°t kh¡c luæn tçn t⁄i a gi¡c lçi nguy¶n câ di»n t‰ch b‹ng 2 2 5 2, chflng h⁄n c¡c ¿nh ngô gi¡c l O(0,0);A(0,1); B(1,2); C(2,1);D(1,0). Ngô gi¡c n y câ 5 i”m nguy¶n tr¶n bi¶n v mºt i”m nguy¶n I(1,1) l 5 i”m trong. V“y theo cæng thøc Picard: SOABCD = . 2 3 Gåi l ngô gi¡c ìn khæng lçi th… sŁ i”m trong l k 0. S . 2 3 3 Ngô gi¡c ìn ABCDI câ di»n t‰ch b‹ng . V“y di»n t‰ch b‹ng . 2 2 H…nh 1.11: Mºt sŁ b i to¡n li¶n quan B i to¡n 1.16. Câ th” °t ÷æc hay khæng mºt tam gi¡c vîi c⁄nh nguy¶n ( º d i c¡c c⁄nh l sŁ nguy¶n) sao cho c¡c ¿nh l c¡c nót l÷îi nh÷ng khæng câ c⁄nh n o thuºc ÷íng thflng l÷îi? Chøng minh. C¥u tr£ líi l câ. D„ ki”m tra ÷æc tam gi¡c vîi c¡c ¿nh câ tåa º (0,0), (12,16), (-12,9) thäa m¢n y¶u cƒu • b i
  • 31. Tải tài liệu tại sividoc.com Viết đề tài giá sinh viên – ZALO:0973.287.149-TEAMLUANVAN.COM 24 B i to¡n 1.17. Câ hay khæng mºt ÷íng g§p khóc kh†p k‰n gçm mºt sŁ l· c¡c o⁄n thflng nguy¶n b‹ng nhau m c¡c ¿nh l c¡c nót cıa l÷îi? Chøng minh. C¥u tr£ líi l khæng. Gi£ sß tçn t⁄i ÷íng g§p khóc A1A2 : : : An nh÷ y¶u cƒu, gçm c¡c o⁄n thflng câ º d i c. X†t v†c tì . Gåi a i v b i l º d i ( ⁄i sŁ) h…nh chi‚u cıa v†c tì ! AiAi+1 ! AiAi+1 t÷ìng øng tr¶n c¡c tröc tåa º Oy, Ox. Ta câ flng thøc c2 = a2 i + b2 i vîi mØi i = 1 : : : n:: D÷ cıa ph†p chia c2 cho 4 l 0,1,2. N‚u c2 chia h‚t cho 4 th… tł flng thøc tr¶n ta suy ra ai v bi công chia h‚t cho 4 ( x†t c¡c d÷ khi chia cho 4 cıa ai v bi). Do â, ph†p và tü t¥m A1, t sŁ và tü 0,5 bi‚n ÷íng g§p khóc A1A2 : : : An th nh ÷íng g§p khóc B1B2 : : : Bn ng›n hìn m c¡c ¿nh v¤n l nót cıa l÷îi. Ti‚p töc l m nh÷ v“y, d¤n tîi mºt ÷íng g§p khóc m c2 cıa nâ khæng chia h‚t cho 4. Chó þ r‹ng vîi måi ÷íng g§p khóc nh÷ th‚ th… a1 + a2 + : : : + an = b1 + b2 + : : : + bn = 0 Tr÷íng hæp 1. N‚u d÷ khi chia c2 cho 4 l 1 th… mºt trong c¡c sŁ ai,bi l l· v sŁ kia l chfin n¶n tŒng a1 + a2 + : : : + an 6= 0 Tr÷íng hæp 2. N‚u d÷ khi chia c2 cho 4 l 2 th… c£ hai sŁ ai,bi •u l· n¶n tŒng a1 + a2 + : : : + an + b1 + b2 + : : : + bn l sŁ l·, kh¡c 0. Væ lþ. B i to¡n 1.18. X†t tam gi¡c nguy¶n sao cho tr¶n c⁄nh cıa nâ khæng chøa c¡c i”m nguy¶n n o kh¡c c¡c ¿nh. Chøng minh r‹ng n‚u mºt tam gi¡c nh÷ v“y chøa trong nâ óng mºt i”m nguy¶n th… i”m n y ch‰nh l trång t¥m cıa tam gi¡c. B i to¡n 1.19. Gi£ sß mºt n-gi¡c nguy¶n lçi sao cho b¶n trong v c⁄nh cıa nâ khæng chøa mºt i”m nguy¶n n o kh¡c c¡c ¿nh, chøng tä n 4. B i to¡n 1.20. Chøng minh r‹ng vîi b§t k… hai i”m nguy¶n A,B sao cho giœa o⁄n nŁi chóng khæng chøa i”m nguy¶n n o kh¡c, t…m ÷æc i”m C sao cho tam gi¡c ABC l nguy¶n thıy. Kho£ng c¡ch tł C ‚n AB b‹ng bao nhi¶u n‚u bi‚t º d i AB = d? B i to¡n 1.21. X†t n i”m nguy¶n(n 3) sao cho b§t k… ba i”m n o giœa chóng •u l“p th nh ÷æc tam gi¡c m c¡c trung tuy‚n cıa nâ khæng c›t mºt i”m nguy¶n n o kh¡c. T…m sŁ n lîn nh§t câ th” ÷æc.
  • 32. Tải tài liệu tại sividoc.com Viết đề tài giá sinh viên – ZALO:0973.287.149-TEAMLUANVAN.COM 25 B i to¡n 1.22. Chøng minh r‹ng mØi h…nh vuæng câ c⁄nh b‹ng n n‹m trong m°t phflng chøa khæng nhi•u hìn (n + 1)2 i”m nguy¶n. 1.4 Mºt sŁ øng döng cıa l÷îi tåa º nguy¶n Vîi nhœng hi”u bi‚t v• l÷îi, ta câ th” câ mºt sŁ øng döng câ ‰ch sau. 1.4.1 Gi¡ trà væ t cıa h m l÷æng gi¡c Ta thß °t c¥u häi: c¡c sŁ sau l sŁ hœu t hay sŁ væ t : sin 1 ; cos ; tan 7 ? 19 55 p Mºt c¡ch tŒng qu¡t: Vîi mØi gâc = (p; q 2 Z), c¡c sŁ cos ; sin ; q tan l sŁ hœu t hay sŁ væ t ? Rª r ng sin hœu t vîi = k ; = k 2 (3k 1); cos hœu t vîi = (6k 1); tan hœu t vîi ; = 3 2 6 + k (k 2 Z). = k ; = 4 p Ta s‡ th§y vîi c¡c gâc cÆn l⁄i = q gi¡ trà cıa c£ 3 h m sŁ •u l sŁ væ t . ” chøng minh i•u â ta nh›c l⁄i hai k‚t qu£ ¢ bi‚t trong l÷æng gi¡c. K‚t qu£ 1.Vîi måi sŁ n 2 N h m sŁ cos nx l mºt a thøc h» sŁ nguy¶n, b“c n Łi vîi cos x, tøc l cos nx = Pn(cos x), trong â, Pn(y) l a thøc h» sŁ nguy¶n b“c n. K‚t qu£ 2. H m sŁ sin nx l t‰ch cıa h m sin x vîi mºt a thøc b“c n 1 cıa cos x; h» sŁ nguy¶n, tøc l : sin nx = sin x:Qn 1(cos x); trong â, Qn 1(y) l a thøc h» sŁ nguy¶n b“c n 1. M»nh • 1.11. Ta câ c¡c h» qu£ sau: a. N‚u cos x l sŁ hœu t th… cos nx công l v sin nx thæng ÷îc (khi sin x =6 0). sŁ hœu t , cÆn c¡c sŁ sin x b. N‚u (p; q) = 1 v cos q l sŁ hœu t th… cos q công l sŁ hœu t .
  • 33. Tải tài liệu tại sividoc.com Viết đề tài giá sinh viên – ZALO:0973.287.149-TEAMLUANVAN.COM 26 Chøng minh. a. Hi”n nhi¶n. b. Nh“n x†t r‹ng n‚u p; q > 1; (p; q) = 1 th… tçn t⁄i sŁ tü nhi¶n k sao cho kp khi chia cho q ÷æc d÷ l 1, tøc l : kp = rq + 1. Bði v“y n‚u cospl sŁ hœu t th… sŁ sau công l sŁ hœu t : q cos k q = cos = ( 1)r cos q q p rq + 1 p Tł flng thøc â công suy ra : N‚u cos l sŁ hœu t th… cos công q q l sŁ hœu t . p Nh÷ v“y, sŁ cos vîi p; q > 1; (p; q) = 1 hœu t khi v ch¿ khi cos q q hœu t . B¥y gií ta chøng minh øng döng M»nh • 1.12. SŁ cos l sŁ væ t vîi måi sŁ tü nhi¶n q > 3. q p m1 2 N). Tr¶n m°t Chøng minh. Gi£ sß cos = l sŁ hœu t (m1; n1 q n 1 phflng l§y h» tåa º Descarte vuæng gâc v tł gŁc tåa º O ta k· c¡c tia t⁄o vîi h÷îng d÷ìng cıa tröc Ox c¡c gâc ; 2 ;:::; (2q 1) . Gåi q q q A0; A1; A2; : : : ; A2q 1 l giao cıa c¡c tia n y vîi ÷íng trÆn ìn và. a gi¡c A0A1A2 : : : A2q 1 l a gi¡c •u, i”m Ak câ tåa º cos k ; sin k . q q V… sŁ cos l sŁ hœu t n¶n ho nh º cıa i”m Ak l sŁ hœu t : q k mk cos = pk(cos ) = q q n k rk Tung º công b‹ng t‰ch c¡c sŁ hœu t Qk(cos ) = vîi sŁ sin ; k = q s k q 1; 2; : : : ; 2q 1; tåa º i”m A0 = (1; 0).
  • 34. Tải tài liệu tại sividoc.com tài giá sinh viên – ZALO:0973.287.149-TEAMLUANVAN.COM 27
  • 35. Tải tài liệu tại sividoc.com tài giá sinh viên – ZALO:0973.287.149-TEAMLUANVAN.COM Nh÷ v“y, Ak = mk ; rk: sin . Kþ hi»u m¤u sŁ chung cıa hai ph¥n n q s k k sŁ mk ; rk l D th… Ak = Mk ;Nk : sin , trong â Mk; Nk l c¡c sŁ n D D q s k k nguy¶n, k = 1; 2; : : : ; 2q 1. D; D: sin q (i; j l c¡c sŁ B¥y gií x†t t§t c £ c¡ c i” m câ t åa º i j nguy¶n), c¡c i”m n y t⁄o th nh mºt l÷îi, hìn nœa c¡c ¿nh cıa 2q gi¡c •u A0A1A2 : : : A2q 1 l c¡c nót cıa l÷îi. Nh÷ng i•u â khæng th” ÷æc (xem phƒn tr÷îc) v… 2q 8. Bði v“y i•u gi£ sß "vîi q > 3 sŁ cos l q sŁ hœu t " l sai, v“y t§t c£ c¡c sŁ cos p(q > 3; (p; q) = 1) l væ t . q B i to¡n 1.23. Cho q 3; (p; q) = 1, chøng minh r‹ng: p a/ c¡c sŁ sin vîi q 6= 6 l c¡c sŁ væ t , q p vîi q 6= 4 l b/ c¡c sŁ tan c¡c sŁ væ t . q TŒng hæp l⁄i câ th” ph¡t bi”u th nh p p p 6= 0; 1 ; 1 công nh÷ M»nh • 1.13. C¡c sŁ arcsin ; arccos vîi q q q 2 p p c¡c sŁ arctan vîi 6= 0; 1; ((p; q) = 1) •u l væ ÷îc vîi sŁ . q q B i to¡n 1.24. (IMO-2016,#3) Gi£ sß P = A1A2 : : : Ak l a gi¡c lçi trong m°t phflng vîi c¡c ¿nh A1; A2; : : : ; Ak câ tåa º nguy¶n v n‹m tr¶n mºt ÷íng trÆn. Gåi S l di»n t‰ch cıa a gi¡c P. Mºt sŁ tü nhi¶n l· n thäa m¢n b…nh ph÷ìng º d i c¡c c⁄nh cıa P •u chia h‚t cho n. Chøng minh r‹ng 2S l sŁ nguy¶n chia h‚t cho n. 1.4.2 C¡c b i to¡n tr¶n æ vuæng B£ng æ vuæng l mºt h…nh chœ nh“t ÷æc chia th nh m:n æ vuæng b‹ng nhau, ta gåi l b£ng æ vuæng k‰ch th÷îc m n. Gi§y æ vuæng l v‰ dö v• b£ng æ vuæng. B£ng æ vuæng l mºt phƒn cıa l÷îi nguy¶n v… th‚ ta câ th” x†t c¡c b i to¡n tr¶n æ vuæng vîi nhœng hi”u bi‚t v• l÷îi tåa º nguy¶n. Tuy nhi¶n do °c i”m cıa c¡c b i to¡n n y l chóng
  • 36. Tải tài liệu tại sividoc.com tài giá sinh viên – ZALO:0973.287.149-TEAMLUANVAN.COM 28 câ t‰nh hœu h⁄n n¶n ta cÆn ¡p döng ÷æc c¡c ph÷ìng ph¡p nh÷ ph÷ìng ph¡p tæ m u, ph÷ìng ph¡p Dirichle, ph÷ìng ph¡p cüc h⁄n,... Nguy¶n lþ Dirichle Thüc ch§t l ph÷ìng ph¡p ph£n chøng: N‚u em nhŁt m thä v o n chi‚c lçng vîi m > n th… ‰t nh§t công câ mºt lçng nhŁt khæng ‰t hìn 2 thä. V‰ dö 1.4.1. Trong mºt b£ng æ vuæng 8 8 ng÷íi ta vi‚t v o mØi æ vuæng mºt sŁ tü nhi¶n tł 1 ‚n 64. Chøng minh r‹ng luæn tçn t⁄i 2 æ vuæng câ chung c⁄nh m hi»u c¡c sŁ ð hai æ â khæng nhä hìn 5. V‰ dö 1.4.2. Tr¶n m°t phflng l§y 5 i”m nguy¶n ph¥n bi»t. Chøng minh r‹ng tçn t⁄i trong chóng hai i”m nguy¶n sao cho o⁄n thflng nŁi hai i”m n y i qua mºt i”m nguy¶n n o â. Chøng minh. °t X1 = 2k + 1; X2 = 2k vîi k 2 Z. Måi i”m M(X1; X2) thuºc mºt trong 4 t“p hæp: d1 = (X1; X1); d2 = (X1; X2); d3 = (X2; X1); d4 = (X2; X2). BŁn t“p hæp n y ríi nhau vîi mØi k 2 Z. Theo nguy¶n t›c Dirichle khi câ 5 i”m nguy¶n th… câ ‰t nh§t 2 i”m thuºc còng mºt trong bŁn t“p hæp d1; d2; d3; d4. Gi£ sß hai i”m A1(x1; y1), A2(x2; y2) thuºc còng trong 4 t“p hæp tr¶n. Khi â trung i”m I cıa A1A2 câ tåa
  • 37. Tải tài liệu tại sividoc.com tài giá sinh viên – ZALO:0973.287.149-TEAMLUANVAN.COM º I x1 + x2 ; y1 + y2 . V… A1; A2 thuºc v o 1 t“p hæp trong 4 t“p hæp 2 2 tr¶n n¶n d„ ki”m tra ÷æc tåa º cıa I l c¡c sŁ nguy¶n. V“y trong 5 i”m luæn tçn t⁄i hai i”m sao cho o⁄n thflng nŁi chóng i qua mºt i”m nguy¶n n o â ( â ch‰nh l trung i”m cıa o⁄n thflng nŁi hai i”m). V‰ dö 1.4.3. Chøng minh r‹ng trong ngô gi¡c lçi nguy¶n luæn tçn t⁄i câ ‰t nh§t mºt i”m nguy¶n l i”m trong. Chøng minh. Theo v‰ dö tr¶n trong 5 i”m A, B, C, D, E tçn t⁄i trung i”m cıa o⁄n thflng nŁi 2 i”m l i”m nguy¶n. Câ hai kh£ n«ng cıa o⁄n thflng: Kh£ n«ng 1: o⁄n thflng n y l ÷íng ch†o ngô gi¡c, hi”n nhi¶n b i to¡n ÷æc gi£i. Kh£ n«ng 2: o⁄n thflng n y l c⁄nh cıa ngô gi¡c, gi£ sß â l c⁄nh DE v E1 l trung i”m cıa DE. Khi â ngô gi¡c ABCDE1 câ 5 i”m
  • 38. Tải tài liệu tại sividoc.com Viết đề tài giá sinh viên – ZALO:0973.287.149-TEAMLUANVAN.COM 29 nguy¶n v công l ngô gi¡c lçi. GiŁng nh÷ ngô gi¡c ABCDE, câ 2 tr÷íng hæp: N‚u rìi v o kh£ n«ng 1 nh÷ tr¶n th… ta dłng l⁄i, n‚u rìi v o kh£ n«ng thø hai ta l°p l⁄i qu¡ tr…nh. Qu¡ tr…nh n y hœu h⁄n bði v“y sau mØi b÷îc ta l⁄i t…m ÷æc mºt i”m nguy¶n n‹m tr¶n c⁄nh ngô gi¡c. Nh÷ v“y t…m ÷æc væ sŁ i”m nguy¶n n‹m tr¶n c⁄nh cıa ngô gi¡c. i•u væ lþ n y chøng minh b i to¡n. Nh“n x†t 1.4.1. Câ th” x†t a gi¡c lçi b§t ký mi„n sao câ 5 ¿nh câ tåa º nguy¶n. p p V‰ dö 1.4.4. a. Tr¶n m°t phflng tåa º l§y i”m A( 2; 3). Chøng minh r‹ng kho£ng c¡ch tł A ‚n c¡c i”m nguy¶n l æi mºt ph¥n bi»t. b. Chøng minh r‹ng vîi måi sŁ nguy¶n d÷ìng n tçn t⁄i mºt h…nh trÆn câ óng n i”m nguy¶n. Chøng minh. a. L§y hai i”m B(x; y); C(u; v) vîi x; y; u; v 2 Z. Ta câ: AB2 = (x p 2)2 + (y p 3)2 ; AC2 = (u p 2)2 + (v p 3)2 . Ta s‡ chøng minh n‚u câ hai i”m nguy¶n còng c¡ch •u i”m A th… hai i”m §y tròng nhau. p p p p N‚u AB = AC =) (x 2)2 +(y 3)2 = (u 2)2 +(y 3)2 =) x 2 + y 2 u 2 v 2 p u) + 2 p sŁ nguy¶n 2 2(x 3(y v). V… v‚ tr¡i l n¶n v‚ ph£i công l sŁ nguy¶n. V“y x = u; y = v. Suy ra B C. p p b. Do A( 2; 3) c¡ch c¡c i”m nguy¶n vîi kho£ng c¡ch æi mºt kh¡c nhau n¶n ta s›p ÷æc c¡c i”m nguy¶n theo thø tü c¡c kho£ng c¡ch t«ng dƒn: d1 < d2 < : : : < dn < : : :. L§y b¡n kinh R thäa m¢n dn < R < dn+1. Khi â h…nh trÆn t¥m A, b¡n k‰nh R chøa óng n i”m nguy¶n. (Ta câ th” chån A l i”m nguy¶n b§t ký). B i to¡n 1.25. Cho h…nh vuæng c⁄nh d i 15 cm. Ng÷íi ta chia h…nh vuæng â th nh 255 h…nh vuæng nhä k‰ch th÷îc 1 cm v ¡nh d§u t¥m cıa mØi h…nh vuæng nhä b‹ng mºt d§u ch§m. Häi r‹ng trong m°t phflng chøa h…nh vuæng ¢ cho vîi 27 ÷íng thflng li»u câ th” chia h…nh vuæng â th nh c¡c phƒn sao cho mØi phƒn câ khæng qu¡ 1 d§u ch§m ÷æc khæng?
  • 39. Tải tài liệu tại sividoc.com Viết đề tài giá sinh viên – ZALO:0973.287.149-TEAMLUANVAN.COM 30 B i to¡n 1.26. Trong h…nh vuæng c⁄nh 1m ng÷íi ta v‡ c¡c ÷íng trÆn b§t ký m tŒng c¡c b¡n k‰nh cıa chóng b‹ng 0,6 m. Chøng minh r‹ng câ th” t…m ÷æc ÷íng thflng song song vîi c⁄nh cıa h…nh vuæng m tr¶n ÷íng thflng â câ nhœng i”m chung cıa ‰t nh§t 2 ÷íng trÆn ¢ cho. V‰ dö 1.4.5. (BŒ • Minkovski) GŁc tåa º l t¥m Łi xøng cıa mºt h…nh lçi H câ di»n t‰ch lîn hìn 4 ( ìn và æ vuæng). Chøng minh r‹ng h…nh H chøa mºt i”m nguy¶n kh¡c gŁc tåa º. Chøng minh. X†t t§t c£ nhœng h…nh lçi nh“n ÷æc tł H qua ph†p tành ti‚n theo c¡c v†c tì m c£ hai tåa º •u chfin. Ta s‡ chøng minh khi â tçn t⁄i ‰t nh§t 2 trong sŁ c¡c h…nh thu ÷æc câ giao kh¡c rØng. Câ th” giîi h⁄n h…nh H bði mºt h…nh trÆn t¥m t⁄i gŁc tåa º, b¡n k‰nh R 2 Z. Trong sŁ c¡c h…nh ang x†t, l§y c¡c h…nh vîi t¥m Łi xøng câ tåa º khæng ¥m, nhä hìn ho°c b‹ng 2n. Khi â ta câ (n + 1)2 h…nh v t§t c£ c¡c h…nh n y •u n‹m trong h…nh vuæng câ c⁄nh 2:(n + R). N‚u c¡c h…nh n y khæng giao nhau vîi måi n th… (n + 1)2 SH 4(n + R)2 .Nh÷ng v… SH > 4 n¶n câ th” chån n sao cho n + R < r SH . Ngh¾a n + 1 4 l tçn t⁄i n sao cho câ ‰t nh§t hai h…nh câ giao kh¡c rØng. Gi£ sß H1 câ t¥m Łi xøng O1; H2 câ t¥m Łi xøng O2 câ i”m chung A. Ta s‡ khflng ành ÷æc trung i”m M cıa O1O2 thuºc c£ hai h…nh. O B O B. V… h…nh câ t¥m Łi xøng Th“t v“y l§y i”m B sao cho ! 1 = ! 2 n¶n B 2 H1. Do A; B 2 H1 n¶n trung i”m cıa AB công thuºc H1 (v… t‰nh ch§t lçi) v trung i”m cıa AB công l trung i”m cıa O1O2. V“y ! M 2 H1 v M khæng tròng vîi O1. Dòng ph†p tành ti‚n theo v†c tì O1O bi‚n M th nh M0 , ta ÷æc M0 l i”m cƒn t…m. V‰ dö 1.4.6. Tr¶n tí gi§y câ v‚t müc di»n t‰ch nhä hìn 1 ( ìn và di»n t‰ch). Chøng minh r‹ng câ th” thi‚t k‚ mºt m⁄ng æ vuæng phı l¶n tí gi§y sao cho khæng câ ¿nh n o cıa m⁄ng l÷îi rìi v o v‚t müc. Chøng minh. Tr÷îc h‚t dòng mºt tí gi§y tr›ng rçi phı tí gi§y b‹ng mºt m⁄ng l÷îi æ vuæng câ c⁄nh song song vîi c⁄nh tí gi§y. C›t c¡c æ vuæng ìn và ra khäi tí gi§y v °t chóng chçng l¶n nhau. Gi£ sß müc câ th” th§m tł æ vuæng n y sang æ vuæng kh¡c. Theo gi£ thi‚t, di»n
  • 40. Tải tài liệu tại sividoc.com Viết đề tài giá sinh viên – ZALO:0973.287.149-TEAMLUANVAN.COM 31 t‰ch v‚t müc nhä hìn 1 n¶n câ 1 i”m cıa h…nh vuæng khæng bà th§m müc. Ti‚p theo ta tr£i c¡c æ vuæng n y v• và tr‰ ban ƒu th… nhœng i”m n y s‡ t⁄o n¶n mºt h» thŁng ¿nh m⁄ng l÷îi æ vuæng ìn và. Hi”n nhi¶n khæng câ i”m n o trong chóng n‹m trong v‚t müc. B i to¡n 1.27. Cho h…nh phflng F câ di»n t‰ch lîn hìn 1. Chøng minh ! AB câ tåa º nguy¶n.r‹ngtçnt⁄ihaii”mA,BthuºcFsaocho Ph÷ìng ph¡p gi£i: p döng nguy¶n t›c Dirichle. B i to¡n 1.28. Trong mºt a gi¡c lçi câ chøa khæng ‰t hìn m2 +1 i”m nguy¶n. Chøng minh r‹ng trong a gi¡c tçn t⁄i m+1 i”m nguy¶n thflng h ng. B i to¡n 1.29. (VMO-2010,#5) Cho b£ng 3 3 v n l sŁ tü nhi¶n cho tr÷îc. T…m sŁ c¡c c¡ch tæ m u khæng nh÷ nhau khi tæ mØi æ bði 1 trong n m u (hai c¡ch tæ m u gåi l nh÷ nhau n‚u c¡ch n y nh“n ÷æc tł c¡ch kia b‹ng c¡ch quay quanh t¥m). Ph÷ìng ph¡p gi£i: Thß vîi n = 1, 2 ” t…m ra h÷îng gi£i. Ta s‡ t…m c¡ch ‚m sŁ c¡c lîp t÷ìng ÷ìng trong t§t c£ n9 c¡ch tæ m u. K‚t qu£: n3 + n5 n3 + n9 n5 =n9 + n5 + 2n3 : 2 4 4 B i to¡n 1.30. Cho ngô gi¡c lçi ABCDE c¡c ¿nh câ tåa º nguy¶n. Chøng minh r‹ng a. Tçn t⁄i ‰t nh§t mºt i”m nguy¶n n‹m trong ho°c tr¶n c⁄nh ngô gi¡c kh¡c c¡c ¿nh. b. Tçn t⁄i ‰t nh§t mºt i”m nguy¶n n‹m trong ngô gi¡c kh¡c c¡c ¿nh. c. Tçn t⁄i ‰t nh§t mºt i”m nguy¶n n‹m trong ho°c tr¶n c⁄nh ngô gi¡c lçi A1B1C1D1E1 t⁄o th nh tł c¡c ÷íng ch†o cıa ngô gi¡c lçi ¢ cho. Ph÷ìng ph¡p gi£i: Dòng nguy¶n t›c Dirichle v nguy¶n t›c cüc h⁄n. Nºi dung cıa Ch÷ìng 1 kh¡ phong phó • c“p ‚n l÷îi nguy¶n vîi a gi¡c •u, hai b i to¡n v• a gi¡c nßa •u v a gi¡c hƒu nºi ti‚p l÷îi nguy¶n còng cæng thøc Picard. C¡c k‚t qu£ ð ¥y ÷æc ¡p döng v o b i
  • 41. Tải tài liệu tại sividoc.com Viết đề tài giá sinh viên – ZALO:0973.287.149-TEAMLUANVAN.COM 32 to¡n tr¶n æ vuæng, t‰nh di»n t‰ch a gi¡c tr¶n l÷îi, t‰nh gi¡ trà væ t cıa h m l÷æng gi¡c,. . . Ch÷ìng ti‚p theo nâi v• quan h» giœa l÷îi nguy¶n v ÷íng trÆn.
  • 42. Tải tài liệu tại sividoc.com Viết đề tài giá sinh viên – ZALO:0973.287.149-TEAMLUANVAN.COM 33 Ch÷ìng 2 L÷îi tåa º nguy¶n v ÷íng trÆn Sau khi x†t c¡c a gi¡c tr¶n l÷îi thu ÷æc mºt sŁ k‚t qu£ tŁt, trong ch÷ìng n y ta công °t ÷íng trÆn tr¶n l÷îi v x†t sŁ c¡c i”m nguy¶n trong v tr¶n ÷íng trÆn. Thüc ch§t b i to¡n li¶n quan ‚n sŁ håc, câ th” gi£i quy‚t ÷æc c¡c v§n • nh÷ bº ba Pythagoras, sŁ c¡c bi”u di„n mºt sŁ tü nhi¶n th nh tŒng hai b…nh ph÷ìng... Düa tr¶n t i li»u tham kh£o [2] v [5]. 2.1 i”m nguy¶n nguy¶n tŁ ¢ câ nhi•u t i li»u giîi thi»u v• " i”m nguy¶n-l÷îi tåa º nguy¶n". — ¥y chóng tæi nh›c l⁄i v bŒ sung th¶m v• c¡c i”m n y tr¶n cì sð k‚t hæp giœa sŁ håc, ⁄i sŁ v h…nh håc, ÷a ra øng döng cıa c¡c kh¡i ni»m v o mºt sŁ v§n • cıa sŁ håc v h…nh håc. C¡c i”m nguy¶n ìn gi£n l c¡c c°p sŁ nguy¶n m ta kþ hi»u l t“p hæp Z2 = f(a; b)ja; b 2 Zg: Câ hai ph†p to¡n cºng v nh¥n tr¶n Z2 x¡c ành nh÷ sau (a; b) + (c; d) = (a + c; b + d); (a; b):(c; d) = (ac bd; ad + bc): (2.1) ” d„ quan s¡t câ th” bi”u di„n t“p hæp Z2 nh÷ l mºt "l÷îi nguy¶n tr¶n m°t phflng", tøc l t“p hæp t§t c£ c¡c i”m tr¶n m°t phflng tåa º vîi ho nh º v tung º •u l c¡c sŁ nguy¶n ho°c t“p hæp c¡c v†ctì vîi gŁc O, c¡c i”m ngån l c¡c i”m n y. Khi â tŒng cıa hai i”m
  • 43. Tải tài liệu tại sividoc.com Viết đề tài giá sinh viên – ZALO:0973.287.149-TEAMLUANVAN.COM 34 nguy¶n bi”u di„n nh÷ tŒng v†ctì thæng th÷íng theo quy t›c h…nh b…nh h nh cÆn ph†p nh¥n l ph†p to¡n n o â m ta bi”u di„n nh÷ tr¶n h…nh v‡ 1. H…nh 2.1: Cºng v nh¥n c¡c i”m nguy¶n C¡ch thø ba bi”u di„n i”m nguy¶n z = (a; b) l d⁄ng tŒng a + bi, trong â i kþ hi»u v†ctì ìn và cıa tröc Oy, tøc l c°p (0,1) = i. C°p (a; 0) l c¡c i”m nguy¶n n‹m tr¶n tröc Ox, c°p (0; b) tøc l c¡c i”m nguy¶n cıa tröc Oy câ d⁄ng bi. Chó þ r‹ng i2 = (0; 1):(0; 1) = ( 1; 0) = 1. Nh÷ v“y, trong G æi khi xu§t hi»n c«n b“c hai cıa sŁ pp ¥m: 1 = i; 9 = 3i; .... MØi i”m nguy¶n z = a + bi 2 G câ th” l“p i”m nguy¶n li¶n hæp vîi nâ: z = a bi 2 G v chu'n l : N(z) = a2 + b2 2 Z 2 . Sß döng c¡c sŁ li¶n hæp, chu'n v cæng thøc ph†p nh¥n d„ nh“n ÷æc quy t›c ” chia i”m nguy¶n z = a + bi cho i”m nguy¶n w = c + di. Cö th”, z zw zw (a + bi)(c di) ac + bd bc ad = = = = + i: (2.2) w N(w) c 2 + d 2 c 2 2 c 2 2 w:w + d + d Công nh÷ c¡c sŁ nguy¶n: 1; 2 = 1 + 1; 3 = 1 + 1 + 1; :::; c¡c i”m nguy¶n câ th” bi”u di„n qua sŁ 1 v i. Chflng h⁄n, 5 3i = 1+1+1+1+ 1 + ( i) + ( i) + ( i). Nh÷ th‚, vîi c¡c i”m nguy¶n câ th” câ ành lþ cì b£n giŁng nh÷ ành lþ cì b£n cıa sŁ håc ” ÷a vi»c nghi¶n cøu c¡c
  • 44. Tải tài liệu tại sividoc.com Viết đề tài giá sinh viên – ZALO:0973.287.149-TEAMLUANVAN.COM 35 i”m nguy¶n v• vi»c nghi¶n cøu c¡c " i”m nguy¶n nguy¶n tŁ" 2.1.1 Sü chia h‚t cıa c¡c i”m nguy¶n N‚u vîi hai i”m nguy¶n a; b 2 Z2 t…m ÷æc c 2 Z2 sao cho a = b:c th… ta nâi a chia h‚t cho b, kþ hi»u l a .. .b hay b chia h‚t a, kþ hi»u l b|a. V‰ dö 2 .. .1 + i v… 2 = (1 + i)(1 i). Tr¶n H…nh 2.2 ch¿ ra c¡c i”m nguy¶n chia h‚t cho 2 + 3i. Ta câ c¡c k‚t qu£ hi”n nhi¶n sau: Quan h» chia h‚t trong Z2 câ c¡c t‰nh ch§t chia h‚t thæng th÷íng: N‚u a .. .b th… ac .. .bc; N‚u ac .. .bc; c 6= 0 th… a .. .b; N‚u a .. .b; b .. .c th… a .. .c. . . Trong lþ thuy‚t c¡c sŁ nguy¶n ta câ tł a; b 2 . . Z, a.b; b.a k†o theo a . = b.. Trong c¡c i”m nguy¶n Z 2 i•u â khæng óng, chflng h⁄n: . . a. a, a. ia. Tuy nhi¶n ta câ . . . . ib: (2.3) a.b; b. a =) a = b ho°c a = Chøng minh. Th“t v“y, gi£ sß a = c khi â c v c 1 = b theo gi£ a b thi‚t, l c¡c i”m nguy¶n. Hi”n nhi¶n 1 = N(1) = N(c:c 1 ). Tł â, N(c) = N(c 1 ) = 1 (v… N(c) v N(c 1 ) l c¡c sŁ nguy¶n d÷ìng thæng th÷íng). Nh÷ng ch¿ tçn t⁄i 4 i”m nguy¶n vîi chu'n b‹ng 1, â l c¡c sŁ 1, -1, i, i m trong phƒn ti‚p theo ta s‡ gåi l c¡c ìn và. Hai i”m nguy¶n ÷æc gåi l li¶n k‚t n‚u th÷ìng cıa chóng l mºt trong c¡c ìn và 1; 1; i; i. i”m nguy¶n p ÷æc gåi l i”m nguy¶n nguy¶n tŁ n‚u p khæng câ ÷îc kh¡c khæng n o kh¡c trł c¡c li¶n k‚t vîi chóng v c¡c ìn và. Chflng h⁄n, c¡c sŁ 3 v 2 + 3i l c¡c i”m nguy¶n nguy¶n tŁ cÆn 5 khæng ph£i l i”m nguy¶n nguy¶n tŁ v… 5 = (2+i)(2 i). ” chøng minh 3 v 2 + 3i l i”m nguy¶n nguy¶n tŁ cƒn ph£i sß döng h» thøc N(a)N(b) = N(ab) v ch⁄y lƒn l÷æt qua c¡c i”m nguy¶n m câ chu'n l ÷îc cıa chu'n cıa sŁ tr÷îc nâ.
  • 45. Tải tài liệu tại sividoc.com Viết đề tài giá sinh viên – ZALO:0973.287.149-TEAMLUANVAN.COM 36 . . H…nh 2.2: a.C¡c i”m nguy¶n.(2 + 3i); b. C¡c i”m nguy¶n li¶n k‚t 2.1.2 ành lþ cì b£n cıa c¡c i”m nguy¶n Nh›c l⁄i ành lþ cì b£n cıa sŁ håc c¡c sŁ nguy¶n: ành lþ 2.1. MØi sŁ nguy¶n a 6= 0 •u bi”u di„n ÷æc d÷îi d⁄ng t‰ch a = :pe 11 pe 22 : : : pe nn trong â, n > 0, l mºt trong c¡c ìn và, pi l c¡c sŁ nguy¶n tŁ ph¥n bi»t (tøc l khæng li¶n k‚t) cÆn e1; e2; : : : en l c¡c sŁ nguy¶n d÷ìng. Ph†p ph¥n t‰ch nh÷ tr¶n l duy nh§t sai kh¡c thø tü c¡c nh¥n tß. T÷ìng tü ta câ k‚t qu£ Łi vîi c¡c i”m nguy¶n nguy¶n tŁ: ành lþ 2.2. MØi i”m nguy¶n g 6= 0 •u bi”u di„n ÷æc d÷îi d⁄ng g = :g1 e1 g2 e2 : : : gn en trong â, l mºt trong c¡c ìn và, g1; g2; : : : ; gn l c¡c i”m nguy¶n nguy¶n tŁ ph¥n bi»t (tøc l khæng li¶n k‚t) cÆn e1; e2; : : : ; en l c¡c sŁ mô d÷ìng. Ph†p ph¥n t‰ch nh÷ tr¶n l duy nh§t theo ngh¾a: måi ph¥n e0 e0 e0 t‰ch kh¡c cıa g th nh c¡c nh¥n tß nguy¶n tŁ câ d⁄ng = 0:h11 h22 : : : hnn vîi bº c¡c sŁ nguy¶n tŁ h1; h2; : : : ; hn kh¡c vîi bº c¡c sŁ nguy¶n tŁ g1; g2; : : : ; gn b‹ng c¡ch thay Œi c¡c phƒn tß v thay Œi phƒn tß th nh phƒn tß li¶n k‚t, cÆn bº c¡c sŁ mô ch¿ kh¡c v• thø tü.
  • 46. Tải tài liệu tại sividoc.com Viết đề tài giá sinh viên – ZALO:0973.287.149-TEAMLUANVAN.COM 37 Chøng minh. ” chøng minh sü tçn t⁄i cıa ph¥n t‰ch tr¶n ta ti‚n h nh quy n⁄p theo sŁ N(g) cıa i”m nguy¶n g: +Ph†p ph¥n t‰ch óng vîi c¡c ìn và vîi chu'n l 1: â l c¡c ìn và 1, i. Gi£ sß N(g) = n v ph†p ph¥n t‰ch óng vîi måi i”m nguy¶n câ chu'n nhä hìn n. N‚u g l i”m nguy¶n nguy¶n tŁ th… ành lþ ÷æc chøng minh, tr¡i l⁄i th… g = b:c vîi N(b) > 1; N(c) > 1. V… N(g) = N(b):N(c) ta nh“n ÷æc N(b) < n; N(c) < n. Theo gi£ thi‚t quy n⁄p, b v c ph¥n t‰ch ÷æc th nh c¡c nh¥n tß nguy¶n tŁ, ngh¾a l i”m nguy¶n g ÷æc ph¥n t‰ch nh÷ tr¶n. + T‰nh duy nh§t ÷æc chøng minh phøc t⁄p hìn v chia l m ba b÷îc B÷îc 1. X†t bŒ • sau (BŒ • v• ph†p chia câ d÷): Vîi mØi c°p i”m nguy¶n a 6= 0 v b •u tçn t⁄i i”m nguy¶n c (th÷ìng khæng ı cıa b cho a) sao cho N(b ca) < N(a). Th“t v“y, gi£ sß b = x + iy l th÷ìng cıa b cho a (nh“n ÷æc tł cæng a thøc ph†p chia). i”m x + iy khæng nh§t thi‚t l i”m nguy¶n (x ho°c y câ th” khæng nguy¶n) nh÷ng nâ câ th” l mºt trong c¡c i”m (x; y) tr¶n m°t phflng Oxy. Ta x†t i”m cƒn t…m c ð l¥n c“n i”m nguy¶n (x; y). Khi â (x; y) n‹m trong h…nh vuæng ìn và t¥m c, v“y kho£ng c¡ch giœa c v b = x + iy nhä hìn 1, v i•u â óng c£ Łi vîi chu'n cıa hi»u a cıa chóng(b…nh ph÷ìng kho£ng c¡ch n y), tøc l : N( b) c) < 1. Bði v“y, a a(a c) = N(a)N a c < N(a) N(b ca) = N b b B÷îc 2. ” thu“n ti»n ta °t a = a0; b = a1 v vi‚t d¢y c¡c ph†p chia câ d÷ sau: a0 = q1a1 + a2 trong â; N(a2) < N(a1) a 1 = q2a1 + a3 trong â; N(a3) < N(a2) a n 2 = q n 1 a n 1 + a n ; trong â; N(an) < N(an 1) a n = q n 1 a n 1
  • 47. Tải tài liệu tại sividoc.com Viết đề tài giá sinh viên – ZALO:0973.287.149-TEAMLUANVAN.COM 38 Qu¡ tr…nh n y k†o d i khi d÷ cıa chu'n kh¡c 0. V… c¡c chu'n n y l c¡c sŁ nguy¶n, ìn i»u gi£m n¶n qua mºt sŁ hœu h⁄n b÷îc ta nh“n ÷æc d÷ b‹ng 0. Ta kþ hi»u an l d÷ cuŁi còng kh¡c 0. N‚u tł d¢y tr¶n, i ng÷æc tł d÷îi l¶n tr¶n ta ÷æc: anjan 1; anjan 2; : : : ; anja0: v d„ chøng minh ÷æc N‚u xnja0; xja1 th… xja2; xja3; : : : ; xjan: Công theo d¢y tr¶n ta th§y c¡c i”m a2; : : : ; a2 bi”u di„n ÷æc d÷îi d⁄ng aj = xja0 + yja1, °c bi»t: an = xna0 + yna1: (2.6) (Nh÷ v“y i”m an l "÷îc chung lîn nh§t" cıa a0 v a1 theo ngh¾a måi i”m nguy¶n chia h‚t cho a0; a1 ho°c li¶n k‚t vîi an ho°c câ chu'n nhä hìn N(an)). Thu“t to¡n t…m ÷îc chung lîn nh§t công ÷æc gåi l thu“t to¡n Euclid. Tł â d„ d ng chøng minh ÷æc m»nh • "N‚u p l i”m nguy¶n nguy¶n tŁ th… ab chia h‚t cho p k†o theo a chia h‚t cho p ho°c b chia h‚t cho p". B÷îc 3. Ta quay l⁄i k‚t thóc ph†p chøng minh ành lþ. N‚u x£y ra flng thøc p1 : : : pn = q1 : : : qn trong â pi; qi l c¡c i”m nguy¶n nguy¶n tŁ, sß döng bŒ • câ th” lƒn l÷æt rót gån c¡c nh¥n tß b¶n tr¡i v b¶n ph£i, suy ra m = n v c¡c nh¥n tß nguy¶n tŁ tròng nhau. Nh“n x†t 2.1.1. Chó þ r‹ng vi»c tr…nh b y ph†p chøng minh tr¶n kh¡c vîi ph†p chøng minh ành lþ cì b£n ð chØ ta ph£i dòng so s¡nh chu'n trong thu“t to¡n Euclid 2.2 ÷íng trÆn tr¶n l÷îi tåa º nguy¶n B i to¡n °t ra l : H¢y ‚m sŁ i”m nguy¶n N(R) n‹m trong ÷íng trÆn K(R) = f(x; y)jx2 + y2 Rg: (2.5) (2.4)
  • 48. Tải tài liệu tại sividoc.com Viết đề tài giá sinh viên – ZALO:0973.287.149-TEAMLUANVAN.COM 39 trong â,R 0 công l sŁ nguy¶n. SŁ N(R) b‹ng di»n t‰ch cıa h…nh F (R) t⁄o th nh tł c¡c h…nh vuæng ìn và cıa l÷îi m ð gâc tr¡i d÷îi n‹m trong K(R). H…nh 2.3: F(R) v N(R) V… kho£ng c¡ch lîn nh§t giœa hai i”m cıa h…nh vuæng ìn và khæng p v÷æt qu¡ 2 n¶n t§t c£ c¡c h…nh vuæng giao vîi ÷íng trÆn x2 +y2 = R2 s›p x‚p trong h…nh v nh kh«n f(x; y)j(R p 2)2 < x2 + y2 < (R + p 2)2 g: (H…nh v‡ bi”u di„n vîi R = 4)Di»n t‰ch cıa h…nh v nh kh«n b‹ng (R + p p 2)2 = 4 p R: 2)2 (R 2 v bði v“y, jSF (R) R2 j < 4 p 2R, trong â, SF (R) kþ hi»u di»n t‰ch N(R) p N(R) R cıa h…nh F. Nh÷ v“y, 2 4 2 . Ngh¾a l , vîi måi R ı lîn ta R câ flng thøc gƒn óng . Ch‰nh x¡c hìn câ th” ph¡t bi”u th nh R2 k‚t qu£ sau: M»nh • 2.1. (K.Gauss) Vîi c¡c kþ hi»u nh÷ tr¶n ta câ: lim N(R) = (2.7) R2 ! R !1
  • 49. Tải tài liệu tại sividoc.com Viết đề tài giá sinh viên – ZALO:0973.287.149-TEAMLUANVAN.COM 40 Nh to¡n håc thi¶n t i K. Gauss ¢ x§p x¿ cæng thøc 2.7 b‹ng c¡ch l“p b£ng t÷ìng øng vîi c¡c gi¡ trà gƒn óng cıa sŁ R 10 20 30 100 200 300 N(R) 317 1257 2831 31517 125629 282697 3,17 3,1425 3,134 3,1417 3,140725 3,14107 Ph†p chøng minh flng thøc (2.7) g›n li•n vîi mºt trong c¡c t‰nh ch§t cì b£n cıa l÷îi Z2 : Di»n t‰ch måi h…nh b…nh h nh sinh ra l÷îi Z2 , •u b‹ng 1. H…nh b…nh h nh nh÷ v“y công ÷æc gåi l h…nh b…nh h nh cì sð cıa l÷îi Z2 . Nh›c l⁄i r‹ng l h…nh b…nh h nh cì sð cıa l÷îi Z2 n‚u nâ chia m°t phflng th nh c¡c h…nh b…nh h nh b‹ng nhau, t“p hæp c¡c ¿nh cıa c¡c h…nh b…nh h nh tròng vîi t“p hæp c¡c nót cıa l÷îi Z2 Gi¡ trà tuy»t Łi cıa hi»u giœa di»n t‰ch h…nh trÆn K(R) v di¶n t‰ch h…nh F t⁄o tł t“p hæp t§t c£ c¡c h…nh b…nh h nh øng vîi c¡c nót trong K(R), nhä hìn di»n t‰ch h…nh v nh kh«n f(x; y)j(R a)2 x2 + y2 (R + a)2 g: trong â, a pl ÷íng ch†o lîn nh§t cıa h…nh b…nh h nh (tr¶n h…nh v‡, R = 4; a = 13). N‚u S = th… SF = N(R) v do â, j :N(R) R2 j < (R + a)2 (R a)2 = 4a R N(R) Ngh¾a l < 4a . Cho R ti‚n ra væ t“n, theo chøng minh tr¶n R2 R: ta nh“n ÷æc: N(R) = lim = ; tøc l = 1 R2 n!1 Ta cÆn nh“n ÷æc mºt h» qu£ cıa (2.7): ⁄i l÷æng N(R) bi”u di„n sŁ t§t c£ c¡c c°p sŁ nguy¶n s›p thø tü (x; y) 2 Z2 thäa m¢n x2 + y2 R2 . Vîi mØi nót (x; y) 2 Z2 sŁ x2 +y2 l sŁ nguy¶n, bði v“y n‚u kþ hi»u r(k) l sŁ c¡c c¡ch bi”u di„n ph¥n bi»t cıa sŁ tü nhi¶n k th nh tŒng cıa hai b…nh ph÷ìng c¡c sŁ nguy¶n (c¡c bi”u di„n k = a2 + b2 = ( a)2 + b2 = a2 + ( b)2 = ( a)2 + ( b)2 coi l æi mºt ph¥n bi»t) th… N(R) = r(0) + r(1) + : : : + r(n); vîi n = R2 :
  • 50. Tải tài liệu tại sividoc.com Viết đề tài giá sinh viên – ZALO:0973.287.149-TEAMLUANVAN.COM 41 M»nh • 2.2. (K.Gauss) Ta câ flng thøc sau lim r(0) + r(1) + : : : + r(n) = ! n n!1 Chó þ r‹ng b£n th¥n h m r(n) khæng ch‰nh quy. Chflng h⁄n r(0) = 1; r(1) = 4; r(2) = 4; r(3) = 0; r(4) = 4; r(5) = 8; r(6) = 0; r(7) = 0; r(8) = 4; : : : ; r(21) = 0; r(22) = 0; r(23) = 0; r(24) = 0; r(25) = 12. 2.2.1 SŁ c¡c bi”u di„n cıa mºt sŁ tü nhi¶n th nh tŒng hai b…nh ph÷ìng ” t…m sŁ c¡c ph¥n t‰ch n = a2 + b2 vîi n 2 N tòy þ ta câ k‚t qu£ sau M»nh • 2.3. Vîi måi n 2 N, sŁ c¡c i”m nguy¶n trong ÷íng trÆn p b¡n k‰nh n b‹ng sŁ c¡c c¡ch ph¥n t‰ch sŁ nguy¶n n th nh tŒng hai b…nh ph÷ìng, óng b‹ng sŁ r(n) x¡c ành nh÷ sau: N‚u n khæng chia h‚t cho 2 th… r(n) = 4[d1(n) d3(n)], vîi d1(n); d3(n) l c¡c ÷îc cıa n thø tü câ d⁄ng 4n + 1; 4n + 3; N‚u n = 2 ps 11 ps 22 : : : ps kk :q1 t1 : : : te ll l ph¥n t‰ch ch‰nh t›c cıa n th nh c¡c nh¥n tß nguy¶n tŁ vîi pi câ d⁄ng 4k + 1; qj câ d⁄ng 4k + 3 th… r(n) = ( 4(s + 1) : : : (s + 1) khi t ; : : : ; t •u chfin 0 1 k trong1 tr÷íngl hæp ng÷æc l⁄i. Tr¶n h…nh v‡ 2.4: H…nh 2.4: r(4)=4;r(25)=12
  • 51. Tải tài liệu tại sividoc.com Viết đề tài giá sinh viên – ZALO:0973.287.149-TEAMLUANVAN.COM 42 Vîi n = 4, ta câ 4 c¡ch ph¥n t‰ch 22 =02 +22 =22 +02 =02 +( 2)2 = ( 2)2 + 02 : Vîi n = 25 ta câ 12 c¡ch ph¥n t‰ch 52 = ( 5)2 +02 = ( 4)2 + ( 3)2 = ( 3)2 +( 4)2 =(0)2 +( 5)2 =32 +( 4)2 =42 +( 3)2 =02 +52 = 42 +32 = 32 +42 =02 +52 =( 3)2 +42 =( 4)2 +(3)2 . Tł ành lþ 2.3 ta rót ra: Ph÷ìng tr…nh x2 + y2 = 5k ; (k 0) câ 4(k + 1) k nghi»m nguy¶n, nâi c¡ch kh¡c, ÷íng trÆn vîi b¡n k‰nh 52 , t¥m ð gŁc tåa º s‡ i qua óng 4(k + 1) nót cıa l÷îi Z2 . 2.2.2 ÷íng trÆn Sinhsel Tr…nh b y phƒn n y düa theo [2]. ƒu ti¶n ta câ nh“n x†t:Vîi måi p 1 sŁ tü nhi¶n n tçn t⁄i h…nh trÆn vîi t¥m t⁄i i”m ( 2; ) chøa trong nâ 3 óng n i”m nguy¶n cıa l÷îi. ” chøng minh nh“n x†t n y ta h¢y chøng tä: n‚u (x1; y1); (x2; y2) l hai nót ph¥n bi»t cıa l÷îi th… kho£ng c¡ch tł chóng ‚n i”m ( p ;1 ) 2 p 2 1 2 p 2 3 khæng b‹ng nhau. Th“t v“y, n‚u (x1 2) + (y1 ) = (x2 2) + 1 2 p 3 2 2 2 2 2 1 (y2 ) th… x1 x2 + y1 y2 y 2 2(x1 + x2) = 0. Tł â rót ra: 3 3 2 2 x1 = x2 v y2 2 y1 2 y1 y2 = 0. flng thøc thø hai t÷ìng ÷ìng 3 3 y1 1 2 = y2 1 2 () 3y1 1 = (3y2 1): 3 3 Tøc l ho°c y1 = y2 ho°c 3(y1 + y2) = 2, nh÷ng flng thøc sau khæng x£y ra. Nh÷ v“y, x1 = x2; y1 = y2. Ta l§y d¢y c¡c b¡n k‰nh R1 < R2 < : : : < Rn < : : :, n‚u Rn < R < Rn+1 th… ÷íng trÆn t¥m p 2; 3 , b¡n 1 k‰nh R chøa trong nâóng n i”m nguy¶n. C¥u häi cƒn quan t¥m v khâ hìn l : vîi sŁ n cho tr÷îc düng ÷æc hay khæng ÷íng trÆn i qua n i”m nót l÷îi?. D„ t…m ÷æc ÷íng trÆn i qua 1,2,3 v 4 i”m. Công d„ ÷a ra c¡c v‰ dö vîi n = 8; n = 12 (H…nh v‡ 2.5, 2.6). t tƒm th÷íng hìn l tr÷íng
  • 52. Tải tài liệu tại sividoc.com Viết đề tài giá sinh viên – ZALO:0973.287.149-TEAMLUANVAN.COM 43 hæp n = 6 (H…nh v‡ 2.5). Tuy nhi¶n vîi n = 5; 7; 17; : : : th… vi»c düng ÷íng trÆn khæng h• ìn gi£n. M»nh • 2.4. (A. Singsel) Vîi måi sæ tü nhi¶n n •u tçn t⁄i ÷íng trÆn i qua óng n nót l÷îi Chøng minh. K‚t qu£ thø hai trong (2.3) cho ph†p k‚t lu“n r‹ng: vîi måi n •u düng ÷æc ÷íng trÆn m tr¶n â câ óng 4n nót l÷îi. ” l m i•u â ch¿ cƒn l§y t¥m ÷íng trÆn l gŁc tåa º cÆn b¡n k‰nh chån l R = 5(k 1)=2 (Xem chó þ sau ành lþ 2.3). H…nh 2.5: n=8, n=6 H…nh 2.6: n=12
  • 53. Tải tài liệu tại sividoc.com Viết đề tài giá sinh viên – ZALO:0973.287.149-TEAMLUANVAN.COM 44 H…nh v‡ 2.5 vîi 6 i”m n‹m tr¶n ÷íng trÆn gæi þ cho ta suy ngh¾ x†t ÷íng trÆn t¥m l i”m 1 ; 0 . N‚u l§y b¡n k‰nh l sŁ R = 5(k 1)=2 2 2 th… ph÷ìng tr…nh ÷íng trÆn câ d⁄ng 1 2 5k 1 x + y2 = (2.8) 2 4 hay (2x 1)2 + (2y)2 = 5k 1 (2.9) Công nh÷ chó þ ð tr¶n, ph÷ìng tr…nh a2 + b2 = 5k 1 câ 4 nghi»m. Rª r ng trong ph÷ìng tr…nh (2.10) mºt trong c¡c sŁ a; b ph£i chfin, sŁ kia l·. Trong ph÷ìng tr…nh (2.9), t‰nh ch§t cıa mØi sŁ h⁄ng l cŁ ành v… v“y mØi nghi»m trong hai nghi»m (a; b); (b; a) cıa (2.10) nh“n ÷æc óng mºt nghi»m cıa (2.9). Nh÷ v“y ph÷ìng tr…nh (2.9) câ óng 2k nghi»m ( b‹ng mºt nßa sŁ nghi»m cıa ph÷ìng tr…nh (2.10)). Nh÷ v“y ta câ th” düng ÷æc ÷íng trÆn i qua n = 2k i”m. Chflng h⁄n vîi 10 i”m tr¶n l÷îi, ch¿ vi»c düng ÷íng trÆn (2.9) øng vîi k = 5. T§t nhi¶n khæng th” l§y i”m 1 ; 0 l m t¥m ÷íng trÆn khi sŁ n 2 l·, n = 2k + 1. Tr÷íng hæp n y ta s‡ düng ÷íng trÆn t¥m 1 ; 0 , b¡n 3 k‰nh R =5k : (x 1 )2 + y2 = 52k hay 3 3 9 (3x 1)2 + (3y)2 = 52k : (2.11) Nh÷ng k‚t lu“n cıa m»nh • (2.3), ph÷ìng tr…nh a2 + b2 = 52k (2.12) câ 4(2k+1) nghi»m. B‹ng c¡ch x†t d÷ cıa ph†p chia cho 3 (b…nh ph÷ìng cıa sŁ nguy¶n khi chia cho 3, câ d÷ l 0 v 1), ta rót ra mºt trong c¡c sŁ a; b chia h‚t cho 3, sŁ cÆn l⁄i khæng chia h‚t cho 3. Gi£ sß a 0(mod3); b 1(mod3). Khi â, trong 4 c°p (a; b); (a; b); (b; a); ( b; a) (2.10)
  • 54. Tải tài liệu tại sividoc.com Viết đề tài giá sinh viên – ZALO:0973.287.149-TEAMLUANVAN.COM 45 ch¿ câ óng mºt c°p l nghi»m cıa ph÷ìng tr…nh (2.11). Do â sŁ nghi»m cıa nâ ‰t hìn 4 lƒn sŁ nghi»m cıa ph÷ìng tr…nh (2.12), tøc l (2.11) câ 2k + 1 nghi»m. Chflng h⁄n vîi k = 2 tøc n = 5, sau khi düng ÷íng trÆn (3x 1)2 + (3y)2 = 54 , ta ÷æc 5 i”m nguy¶n tr¶n nâ: (7,5),(7,-5),(-2,8),(-8,0),(-2,-8). C¡c ph÷ìng tr…nh (2.8), (2.11) ÷æc gåi l c¡c ÷íng trÆn Singsel. Chó þ r‹ng vîi sŁ n cho tr÷îc c¡c ph÷ìng tr…nh n y ch÷a ch›c ¢ cho ÷íng trÆn nhä nh§t i qua n i”m cıa l÷îi. Chflng h⁄n vîi n = 4 câ p 2 th” l§y b¡n k‰nh ÷íng trÆn l , vîi n = 9, ÷íng trÆn Sinhsel câ 2 b¡n k‰nh 625 nh÷ng ÷íng trÆn vîi t¥m ( 1; 0), b¡n k‰nh 65 công i 3 3 3 qua 9 nót cıa l÷îi. C¡c b i to¡n kh¡c: B i to¡n 2.1. Chøng minh r‹ng vîi mØi sŁ tü nhi¶n n tçn t⁄i qu£ cƒu p p 1 ) chøa trong nâ óng n i”m cıa l÷îi Z3 .t¥m(2;3; 3 B i to¡n 2.2. Gi£ sß ÷íng trÆn (x x0)2 + y2 = R 2 l ÷íng trÆn Singsel i qua óng n i”m nguy¶n cıa Z2. Chøng minh r‹ng tr¶n m°t p 0 B i to¡n 2.3. Chøng minh r‹ng vîi måi sŁ tü nhi¶n n •u tçn t⁄i ÷íng trÆn i qua óng n i”m cıa m°t phflng thuºc l÷îi tam gi¡c •u; l÷îi löc gi¡c •u. B i to¡n 2.4. Chøng minh r‹ng n‚u ‰t nh§t mºt tåa º cıa t¥m ÷íng trÆn l sŁ væ t th… tr¶n ch‰nh ÷íng trÆn t…m ÷æc khæng qu¡ hai i”m câ tåa º hœu t . B i to¡n 2.5. Chøng minh r‹ng a. Tçn t⁄i ÷íng trÆn câ t¥m t⁄i nót l÷îi nguy¶n m tr¶n nâ khæng câ i”m hœu t n o. b. Tçn t⁄i ÷íng trÆn m tr¶n nâ chøa óng mºt i”m hœu t . c. Tçn t⁄i ÷íng trÆn m tr¶n nâ chøa óng hai i”m hœu t .
  • 55. Tải tài liệu tại sividoc.com Viết đề tài giá sinh viên – ZALO:0973.287.149-TEAMLUANVAN.COM 46 d. N‚u ÷íng trÆn t¥m ð gŁc tåa º (0,0) i qua ‰t nh§t mºt i”m hœu t th… tr¶n ÷íng trÆn â s‡ chøa væ sŁ c¡c i”m hœu t cıa m°t phflng. B i to¡n 2.6. Tr¶n gi§y k· æ, k‰ch th÷îc mØi æ l 1 1 v‡ ÷íng trÆn b¡n k‰nh R, t¥m l i”m nót. Chøng minh r‹ng n‚u tr¶n ÷íng trÆn câ p 1988 i”m nót cıa l÷îi th… ho°c R ho°c R 2 l sŁ nguy¶n. B i to¡n 2.7. Chøng minh r‹ng vîi måi n chfin, tçn t⁄i ÷íng trÆn t¥m ( 1 3; 0) i qua óng n nót cıa l÷îi Z2 . 2.3 Mºt sŁ øng döng v o sŁ håc 2.3.1 Bº ba Pythagoras Bº ba sŁ tü nhi¶n (a; b; c) mØi sŁ l º d i c⁄nh tam gi¡c vuæng, tøc l thäa m¢n i•u ki»n cıa ành lþ Pythagore a2 + b2 = c2 . Ta s‡ sß döng ành lþ cì b£n v• i”m nguy¶n ” t…m t§t c£ c¡c bº ba â. Ta câ c2 = (a + bi)(a bi). Ph¥n t‰ch c v a + bi th nh c¡c nh¥n tß nguy¶n tŁ: c = pe1 pe2 : : : pe n ; a + bi = ql1 ql2 : : : ql m sk1 sk2 : : : skl ; qi = qi; si 6= si 1 2 n 1 2 m 1 2 l = ( q1 l1 q2 l2 : : : qm l m ):(sk 11 (sk 22 (sk ll ):( :q1 l 1 q2 l 2 : : : qm l m ):(q1 l 1 s2 k 2 : : : sl k l ) = = :q1 2l1 q2 2l2 : : : qm 2l m :sk 11 sk 22 : : : sk ll :s1 k 1 s2 k 2 : : : sl k l . Tł ¥y ki = 2ri. Bði v“y, c = d(u+iv)(u iv) vîi d = q1 2l1 q2 2l2 : : : qm 2l m ; (u+iv) = sr 11 r2 k2 : : : sr ll . Khi â, c2 = d2 (u + iv)2 (u + iv)2 = [d(u + iv)]2 :[d(u + iv)]2 . ‚n ¥y sß döng sü ph¥n t‰ch duy nh§t th nh c¡c nh¥n tß nguy¶n tŁ( ành lþ cì b£n) ta ÷æc: a + bi = d(u + iv)2 = d(u2 v2 ) + (2duv)i Tł â suy ra: M»nh • 2.5. MØi bº ba Pythagore bi”u di„n ÷æc d⁄ng a = d(u2 v2 ); b = 2duv; c = d(u2 + v2 ); u; v 2 N:
  • 56. Tải tài liệu tại sividoc.com Viết đề tài giá sinh viên – ZALO:0973.287.149-TEAMLUANVAN.COM 47 2.3.2 C¡c d⁄ng cıa i”m nguy¶n nguy¶n tŁ Mºt øng döng thø hai quan trång l ta câ th” mæ t£ t§t c£ c¡c i”m nguy¶n nguy¶n tŁ. M»nh • 2.6. C¡c i”m nguy¶n nguy¶n tŁ câ mºt trong ba d⁄ng sau: (A) 1+i v c¡c i”m li¶n k‚t vîi nâ, (B) C¡c sŁ nguy¶n tŁ thæng th÷íng d⁄ng p=4k+3 v c¡c li¶n k‚t, (C) Nh¥n tß a+bi cıa sŁ nguy¶n tŁ p thæng th÷íng d⁄ng 4k + 1; k 2 N Chøng minh. X†t i”m nguy¶n nguy¶n tŁ q. BŒ •: i”m nguy¶n nguy¶n tŁ q l ÷îc cıa óng mºt sŁ nguy¶n tŁ p. Th“t v“y, ta luæn câ qjN(q) n¶n q chia h‚t mºt nh¥n tß nguy¶n tŁ n o â cıa N(q). M°t kh¡c q khæng th” chia h‚t 2 sŁ nguy¶n tŁ ph¥n bi»t l v r v… ta luæn t…m ÷æc c¡c sŁ nguy¶n m v n sao cho ml + rn = 1. BŒ • ÷æc chøng minh. Nh÷ v“y, i”m nguy¶n tŁ q cho tr÷îc luæn luæn chia h‚t mºt sŁ nguy¶n tŁ p(thæng th÷íng). Tøc l q = a + bijp hay q:t = p, vîi t 2 Z2 . Ta suy ra:N(q)N(t) = N(p) = p2 . Bði v“y câ hai kh£ n«ng " N(q) = a2 + b2 = p; t l t = q = a bi (b) N(t) = 1; tøc l ìn và v q li¶n k‚t vîip (a) B¥y gií ta x†t 3 tr÷íng hæp Łi vîi sŁ nguy¶n tŁ p: N‚u p = 2 = 12 + 12 = (1 + i)(1 i) = i(1 i2 ). Khi â, c¡c i”m nguy¶n 1 + i = (1; 1); 1 i = (1; 1); 1 + i = ( 1; 1); 1 i = ( 1; 1) l c¡c i”m li¶n k‚t vîi nhau. Ta câ d⁄ng (A). N‚u p = 4k + 3, flng thøc trong (b) khæng x£y ra v… b…nh ph÷ìng cıa mºt sŁ nguy¶n b§t ký khi chia cho 4 câ d÷ l 0 ho°c 1. Bði v“y sŁ nguy¶n tŁ d⁄ng 4k + 3 l i”m nguy¶n tŁ. Ta câ d⁄ng (B). N‚u p = 4k + 1. Sß döng bŒ • Eluler: "N‚u p = 4k + 1; k 2 Z l sŁ nguy¶n tŁ th… tçn t⁄i x 2 N sao cho pjx2 + 1, tøc pj(x + i):(x i)" th… trong tr÷íng hæp cıa ta p khæng th” l i”m nguy¶n nguy¶n tŁ. Ta suy
  • 57. Tải tài liệu tại sividoc.com Viết đề tài giá sinh viên – ZALO:0973.287.149-TEAMLUANVAN.COM 48 ra p ph£i chia h‚t cho mºt i”m nguy¶n n o â g = (a + bi). Tł ¥y . . . . (c + di) khæng li¶n theo ành lþ cì b£n p.(a bi). N‚u câ p.(c + di) m . . k‚t vîi a + bi th… p.(c di). Suy ra: . 2 2 2 2 . di) = (a )(c ). Tr¡i vîi gi£ thi‚t p.(a + bi)(a bi)(c + bi)(c + b + d p l sŁ nguy¶n tŁ. Ngh¾a l p = (a + bi):(a bi) = a2 + b2 = N(g): (2.13) M»nh • ÷æc chøng minh ho n to n. Nºi dung ch÷ìng 2 l " ‚m" sŁ i”m nguy¶n tr¶n ÷íng trÆn v trong ÷íng trÆn. Phƒn øng döng v o sŁ håc cho ta mºt sŁ k‚t qu£ nh÷ bº ba Pythagoras, c¡c d⁄ng cıa i”m nguy¶n nguy¶n tŁ,: : : cho th§y sü g›n k‚t, h i hÆa giœa sŁ håc v h…nh håc.
  • 58. Tải tài liệu tại sividoc.com Viết đề tài giá sinh viên – ZALO:0973.287.149-TEAMLUANVAN.COM 49 Ch÷ìng 3 C¡c b i to¡n kh¡c 3.1 i”m nguy¶n tr¶n ÷íng cong phflng Phı mºt l÷îi tåa º nguy¶n l¶n m°t phflng ta x†t mºt ÷íng cong tr¶n l÷îi v °t b i to¡n t…m c¡c i”m nguy¶n m ÷íng cong i qua. C¡c i”m n y câ th” rØng, hœu h⁄n ho°c væ h⁄n. Thüc ch§t b i to¡n l gi£i ph÷ìng tr…nh nghi»m nguy¶n, tuy nhi¶n x†t tr¶n l÷îi b i to¡n s‡ câ trüc quan h…nh håc hìn. Ph÷ìng ph¡p gi£i c¡c b i to¡n n y sß döng c¡c ki‚n thøc v• sŁ håc c¡c sŁ nguy¶n. Sau ¥y l c¡c v‰ dö. V‰ dö 3.1.1. Chøng minh r‹ng câ væ sŁ c¡c i”m nguy¶n n‹m tr¶n Hypecbol (H): x2 2y2 = 1. p p Líi gi£i.Ta câ x2 2y2 = 1 () (x + y 2)(x y 2) = 1. Ta câ ngay i”m (x1; y1) = (3; 2) l i”m n‹m tr¶n Hypecbol. Vîi n 1, ta °t xn+1 = 3xn + 4yn; yn+1 = 2xn + 3yn. B‹ng c¡ch quy n⁄p chøng minh ÷æc vîi måi n (x1 + y1 p n p n p 2) = (3 + 2 2) = xn + yn 2; p p p : (x1 y1 2)n = (3 2 2)n = xn yn 2 p p p n p n , t÷ìng ÷ìng Suy ra (xn + yn 2)(xn yn 2) = (3 + 2 2) (3 2 2) vîi xn 2 2yn 2 = (9 8)2 = 1. Nh÷ v“y ngo i i”m (3,2) ta t…m ÷æc væ sŁ i”m nguy¶n Xn(xn; yn) n‹m tr¶n Hypecbol x2 2y2 = 1. V‰ dö 3.1.2. (IMO-2006,#4) T…m t§t c£ c¡c c°p sŁ nguy¶n (x,y) sao cho 1 + 2x + 22x+1 = y2 .
  • 59. Tải tài liệu tại sividoc.com Viết đề tài giá sinh viên – ZALO:0973.287.149-TEAMLUANVAN.COM 50 Chøng minh. B i to¡n câ th” ph¡t tri”n nh÷ sau: T…m t§t c£ c¡c i”m nguy¶n x; y) n‹m tr¶n ÷íng cong 1 + 2x + 22x+1 = y2 . N‚u (x; y) l nghi»m th… hi”n nhi¶n x 0 v (x; y) công l nghi»m. Vîi x = 0 ta câ hai nghi»m (0; 2) v (0; 2). Gi£ sß (x; y) l nghi»m vîi x > 0. Khæng m§t t‰nh ch§t tŒng qu¡t ta coi y > 0. Vi‚t l⁄i ph÷ìng tr…nh nh÷ sau: 2x (1 + 2x+1 = (y 1)(y + 1 chøng tä y 1 v y + 1 l chfin, ch‰nh x¡c l mºt trong chóng chia h‚t cho 4. Do â x 3 v mºt trong c¡c nh¥n tß n y chia h‚t cho 2x 1 nh÷ng khæng chia h‚t cho 2x . V“y y = 2x 1 m + ; m l·; = 1: (3.1) Thay v o ph÷ìng tr…nh ¢ cho thu ÷æc: 2x (1 + 2x+1 ) = (2x 1 m + )2 1 = 22x 2 + 2x m (3.2) Vîi = 1 k†o theo m2 8 0, tøc m = 1, khæng thäa m¢n (3.4). Vîi = 1, ph÷ìng tr…nh (3.4) cho ta 1 + m = 2x 2 (m2 8) 2(m2 8): K†o theo 2m2 m 17 0. Tł â, m 3. M°t kh¡c theo (3.4), m 6= 1. V… m l· n¶n m = 3, cho ta x = 4. Tł (3.3) ta câ y = 23. Ki”m tra l⁄i ta câ c¡c gi¡ trà n y thäa m¢n ph÷ìng tr…nh. Nhî r‹ng y = 23 công thäa m¢n. Nh÷ v“y, ta câ c¡c i”m nguy¶n (0; 2); (0; 2); (4; 23); (4; 23) tr¶n ÷íng cong ¢ cho. V‰ dö 3.1.3. (IMO-1997,#5) T…m t§t c£ c¡c c°p sŁ nguy¶n (a,b) vîi a; b 1 thäa m¢n ph÷ìng tr…nh ab2 = ba . Chøng minh. B i to¡n câ ph¡t tri”n nh÷ sau: T…m t§t c£ c¡c i”m nguy¶n (x; y) vîi x; y 1 n‹m tr¶n ÷íng cong xy2 = yx Tr÷îc h‚t chó þ r‹ng: n‚u ta câ am = bn th… luæn tçn t⁄i c n o â sao cho a = ce ; b = cf vîi m = f d; n = ed v d l ÷îc chung lîn nh§t cıa m v n (Chøng minh b‹ng c¡ch ph¥n t‰ch a; b ra thła sŁ nguy¶n tŁ). Trong tr÷íng hæp b i to¡n, gi£ sß d l ÷îc chung lîn nh§t cıa a v b2 v ta °t a = de; b2 = df. Khi â theo chó þ tr¶n, a = ce ; b = cf vîi
  • 60. Tải tài liệu tại sividoc.com Viết đề tài giá sinh viên – ZALO:0973.287.149-TEAMLUANVAN.COM 51 c n o â. Do â, f ce = ec2f . Khæng th” x£y ra e = 2f v… khi â, gi£n ÷îc hai v‚ cho e = f. Væ lþ. Gi£ sß 2f > e th… f = ec2f e . Suy ra e = 1 v f = c2f 1 . N‚u c = 1 th… f = 1 v ta câ nghi»m a = b = 1. N‚u c 2 th… c2f 1 2f > f, væ nghi»m. CuŁi còng, gi£ sß 2f < e th… e = f:ce 2f . Tł â, f = 1 v e = ce 2 :ce 2 2e 2 e vîi e 5 n¶n ta ph£i câ e = 3 ho°c e = 4(e > 2f = 2). Gi¡ trà e = 3 cho nghi»m a = 27; b = 3 cÆn gi¡ trà e = 4 cho nghi»m a = 16; b = 2. Ta thu ÷æc 3 i”m nguy¶n (1; 1); (16; 2); (27; 3). 3.2 Mºt sŁ to¡n thi håc sinh giäi v thi Olympic Ta s‡ x†t øng döng cıa l÷îi tåa º v o mºt sŁ b i to¡n thi håc sinh giäi v thi Olympic quŁc t‚, mð ƒu l b i to¡n thi Olympic To¡n quŁc t‚ n«m 2016. B i to¡n • c“p ‚n mºt a gi¡c nguy¶n, c¡c ¿nh n‹m tr¶n ÷íng trÆn v câ t‰nh ch§t °c bi»t. ” gi£i nâ, ki‚n thøc dòng ‚n kh¡ quen thuºc: i”m nguy¶n, chia h‚t c¡c sŁ nguy¶n, cæng thøc Picard,: : : vîi sü t÷ duy r§t linh ho⁄t. V‰ dö 3.2.1. (IMO.2016,#3) Cho P = A1A2 : : : Ak l mºt a gi¡c lçi trong m°t phflng. C¡c ¿nh câ tåa º nguy¶n v n‹m tr¶n mºt ÷íng trÆn. Mºt sŁ tü nhi¶n n l· thäa m¢n b…nh ph÷ìng º d i mØi c⁄nh cıa P •u chia h‚t cho n. Gåi S l di»n t‰ch cıa P, chøng minh 2S l sŁ tü nhi¶n chia h‚t cho n. Chøng minh. Khæng m§t t‰nh ch§t tŒng qu¡t, gi£ sß a gi¡c P l a gi¡c khæng câ ÷íng ch†o n o m b…nh ph÷ìng º d i chia h‚t cho n v… n‚u tr¡i l⁄i ta s‡ chia a gi¡c th nh hai a gi¡c câ sŁ c⁄nh nhä hìn. + Ta x¡c ành vp(x) = maxk 2 Zjpk jx vîi måi x 2 Q(pk jx (= 9q 2 Z; x = qpk , v‰ dö v3(81) = 4; v5( 1 ) = 3). Nh›c l⁄i r‹ng måi tam 125 gi¡c ABC vîi c⁄nh p a; p b; p , di»n t‰ch S = S(ABC) v b¡n k‰nh c ÷íng trÆn ngo⁄i ti‚p R, ta câ hai cæng thøc: abc = 4(2S)2 R2 : (3.3)
  • 61. Tải tài liệu tại sividoc.com Viết đề tài giá sinh viên – ZALO:0973.287.149-TEAMLUANVAN.COM 52 4(2S)2 = 2ab + 2bc + 2ca a2 b2 c2 : (3.4) Vîi måi a gi¡c nguy¶n P, ành lþ Picard cho ta S(F ) = n + m 1 2 vîi n v m l sŁ c¡c i”m nguy¶n ð trong v tr¶n c⁄nh a gi¡c P. Do â, 2S(P) = 2n + m 2 2 Z+ . Ta câ Ai l c¡c i”m nguy¶n n¶n AiAj 2 l sŁ nguy¶n. Ta ch¿ cƒn chøng minh b i to¡n Łi vîi tr÷íng hæp n l lôy thła cıa mºt sŁ nguy¶n tŁ l· p. °t n = pm v do â gi£ thi‚t l pm jAiAj 2 vîi måi i; j 2 1; 2; : : : ; n v cƒn chøng minh pm j2S(P). a. Tr÷íng hæp k = 3. Gi£ sß p p = A2A3; p = A3A1 = A1A2; b a c sao cho pm ja; b; c. Theo cæng thøc (3.4), 4(2S )2 = 2ab + 2bc + 2ca a2 b2 c2 : M Måi sŁ h⁄ng b¶n ph£i chia h‚t cho p2m , do â, p2m j4(2SM)2 , k†o theo pm j2SM. b. Tr÷íng hæp k 4. Ta quy n⁄p theo k: Gi£ sß b i to¡n ÷æc chøng minh Łi vîi (k 1 gi¡c, ta i chøng minh nâ óng vîi måi k gi¡c. p p p m ja; b. Vîi SM l di»n x = A3A1 vîi p °t a = A1A2; b = A2A3; t‰ch tam gi¡c A1A2A3, O l t¥m ÷íng trÆn i qua c¡c ¿nh a gi¡c P. N‚u pm jx th… M A1A2A3 v a gi¡c Q = A1A3A4 : : : Ak l tam gi¡c v (k 1) gi¡c thäa m¢n gi£ thi‚t quy n⁄p, nh÷ ¢ ch¿ ra ð tr¶n pm j2SM; pm j2SQ. Do â: pm j2SM + 2SQ = 2S nh÷ y¶u cƒu. B¥y gií, gi£ sß pm khæng chia h‚t x. Khi â, c = vp(c) < m. Theo cæng thøc (3.4) ta câ 4(S)2 = 2ab + 2ax + 2bx a2 b2 x2 Lôy thła cao nh§t cıa p khi chia cho x2 nhä hìn (nghi¶m ng°t) lôy thła cao nh§t cıa p khi chia cho sŁ h⁄ng b§t ký trong 5 sŁ h⁄ng cıa v‚ ph£i flng thøc tr¶n n¶n suy ra: vp((2S)2 ) = vp( x2 ) = 2c: B¥y gií cæng thøc (3.3) cho ta: abx = 4(2S)2 R2 , do â, R2 l sŁ hœu t v câ th” l§y gi¡ trà øng vîi lôy thła cao nh§t cıa p chia cho R2 nh÷
  • 62. Tải tài liệu tại sividoc.com Viết đề tài giá sinh viên – ZALO:0973.287.149-TEAMLUANVAN.COM 53 thæng th÷íng. Theo t‰nh ch§t cıa vp(x) ta câ: vp(a) + vp(b) + c = vp((2S)2 ) + vp(R2 ) = 2c + vp(R2 ): m i•u â k†o thoe vp(R2 ) = vp(a) + vp(b) c 2m c > m. Do â, pm jR2 (theo ành ngh¾a cıa vp(R2 )). X†t M AiOAi+1 vîi i = 1; 2; : : : ; k. °t AiAi+1 = p y thäa m¢n pm jy. L⁄i theo cæng thøc (3.4) ta câ: 4(2SMAiOAi+1 )2 = 2R2 y + 2R2 R2 R4 R4 y2 = y(4R2 y) V… pm jy; pm jR2 n¶n pm j2SMAiOAi+1 vîi måi i. TŒng c¡c sŁ h⁄ng 2SMAiOAi+1 ( nh÷ c¡c di»n t‰ch tam gi¡c ành h÷îng) vîi i = 1; 2; : : : ; k quy v• 2S. Suy ra pm j2S. B…nh lu“n Ta câ th” x†t th¶m tr÷íng hæp P l tø gi¡c b‹ng c¡ch dòng ành lþ Ptolemy coi nh÷ mºt b i to¡n nhä. Theo nh÷ nhi•u ºi tuy”n Olympic to¡n QuŁc t‚ nh“n x†t: ¥y l mºt b i to¡n khâ nh§t trong • thi n«m 2016, do Nga • nghà. Thüc ch§t l b i to¡n sŁ håc nh÷ng ng÷íi gi£i cƒn hi”u bi‚t s¥u s›c v• c¡c cæng thøc, ành lþ h…nh håc ¡p döng tr¶n l÷îi tåa º nguy¶n. Sau ¥y l mºt v‰ dö nœa li¶n quan d‚n l÷îi æ vuæng: V‰ dö 3.2.2. (IMO-1996,#1) Cho sŁ d÷ìng r v mºt b£ng h…nh chœ nh“t ABCD vîi AB=20,BC=12. H…nh chœ nh“t ÷æc chia th nh 20 12 ævuæng. X†t ph†p dàch chuy”n sau: Dàch chuy”n tł æ vuæng n y sang ævuæng kia ch¿ ÷æc thüc hi»n n‚u kho£ng c¡ch giœa hai t¥m æ vuæng p b‹ng r. Nhi»m vö °t ra l t…m mºt d¢y c¡c ph†p dàch chuy”n i tł æ vuæng câ A l ¿nh ‚n æ vuæng câ B l ¿nh. a. Chøng minh r‹ng nhi»m vö l b§t kh£ thi n‚u r chia h‚t cho 2 ho°c chia h‚t cho 3. b. Chøng minh r‹ng nhi»m vö l kh£ thi n‚u r = 73. c. Nhi»m vö câ thüc hi»n ÷æc khæng n‚u r = 97?
  • 63. Tải tài liệu tại sividoc.com Viết đề tài giá sinh viên – ZALO:0973.287.149-TEAMLUANVAN.COM 54 Chøng minh. (a) Gi£ sß dàch chuy”n a ìn và theo mºt h÷îng v b ìn và theo h÷îng vuæng gâc, khi â a2 + b2 = r. N‚u r chia h‚t cho 2 th… c£ a v b •u chfin ho°c •u l·. Nh÷ng i•u â ngh¾a l ta ch¿ câ th” câ c¡c æ en ho°c æ tr›ng (gi£ sß c¡c æ ÷æc tæ m u nh÷ tr¶n b n cí vua). Hai gâc h…nh chœ nh“t còng mºt m u n¶n nhi»m vö khæng thüc hi»n ÷æc. X†t ph†p chia c¡c sŁ a; b; r cho 3: n‚u r chia h‚t cho 3 th… a v b c£ hai •u chia h‚t cho 3. i•u â ngh¾a l n‚u b›t ƒu tł æ câ tåa º (0,0) th… ta ch¿ câ th” dàch chuy”n tîi æ câ tåa º d⁄ng (3m; 3n). i”m Æi häi câ tåa º (19,0) khæng ð d⁄ng â n¶n nhi»m vö khæng thüc hi»n ÷æc. (b) N‚u r = 73 th… ta câ a = 8; b = 3 (ho°c ng÷æc l⁄i). Câ 4 ki”u dàch chuy”n: A : (x; y) 7! (x + 8; y + 3) B : (x; y) 7! (x + 3; y + 8) C : (x; y) 7! (x + 8; y 3) D : (x; y) 7! (x + 3; y 8) Ta l§y (x 8; y 3) 7! (x; y) trong ph†p dàch chuy”n ki”u A,v.v: : : th… khi câ a ph†p dàch chuy”n ki”u A, b ph†p dàch chuy”n ki”u B v v.v: : : s‡ câ c¡c ph÷ìng tr…nh sau: 8(a + c) + 3(b + d) = 19; 3(ac) + 8(b d) = 0 Mºt nghi»m d„ th§y l a = 5; b = 1; c = 3; d = 2. B›t ƒu vîi nghi»m n y, b‹ng mºt sŁ t‰nh to¡n ta t…m ÷æc c¡c b÷îc thüc hi»n nhi»m vö (0; 0) 7! (8; 3) 7! (16; 6) 7! (8; 9) 7! (11; 1) 7! (19; 4) 7! (11; 7) 7! (19; 10) 7! (16; 2) 7! (8; 5) 7! (16; 8) 7! (19; 0). (c) N‚u r = 97 ta cƒn câ a = 9; b = 4. GiŁng nh÷ tr¶n, gi£ sß ta b›t ƒu ð (0,0). B‹ng c¡ch lþ lu“n "chfin, l·" ta rót ra nhi»m vö l b§t kh£ thi. V‰ dö 3.2.3. (IMO-1997,#1) Trong m°t phflng c¡c i”m vîi tåa º nguy¶n l c¡c ¿nh cıa c¡c h…nh vuæng ìn và. C¡c h…nh vuæng ÷æc tæ m u en, tr›ng ( nh÷ tr¶n b n cí vua). Vîi mØi c°p sŁ nguy¶n d÷ìng (m,n) x†t mºt tam gi¡c vuæng m c¡c ¿nh câ tåa º nguy¶n, c¡c c⁄nh
  • 64. Tải tài liệu tại sividoc.com Viết đề tài giá sinh viên – ZALO:0973.287.149-TEAMLUANVAN.COM 55 b¶n chi•u d i m v n, n‹m dåc theo c⁄nh h…nh vuæng. Kþ hi»u S1 l tŒng di»n t‰ch phƒn m u en cıa tam gi¡c v S2 l tŒng di»n t‰ch phƒn m u tr›ng cıa tam gi¡c. °t f(m; n) = jS1 (a) T‰nh f(m,n) vîi måi m v n nguy¶n d÷ìng còng chfin ho°c còng l·. (b) Chøng minh r‹ng f(m; n) 1 maxfm; ng vîi måi m v n. 2 H…nh 3.1: H…nh chœ nh“t 20 12 Chøng minh. (a) N‚u m v n •u chfin th… f(m; n) = 0. Th“t v“y, gåi M l trung i”m cıa c⁄nh huy•n th… M l mºt i”m nót cıa l÷îi. N‚u ta quay tam gi¡c mºt gâc 1800 th… tam gi¡c tròng vîi nßa kia cıa h…nh chœ nh“t v ta th§y sŁ m u en tr›ng tr¶n c¡c æ l nh÷ nhau. Do â, S1 v S2 Łi vîi tam gi¡c l mØi nßa gi¡ trà cıa chóng Łi vîi h…nh chœ nh“t. Nh÷ng sŁ c¡c æ en, tr›ng tr¶n h…nh chœ nh“t th… b‹ng nhau n¶n f(m; n) = jS1 S2j = 0. N‚u m v n •u l· th… trung i”m c⁄nh huy•n l t¥m cıa mºt h…nh vuæng v ta v¤n câ sŁ m u en, tr›ng cıa mØi nßa h…nh chœ nh“t b‹ng nhau. Lóc n y S1 v S2 hìn 1 so vîi h…nh chœ nh“t, bði v“y f(m; n) = 1 . 2 (b) K‚t qu£ l“p tøc suy ra tł (a) n‚u m v n còng t‰nh chfin l·. L“p lu“n trong (a) s‡ sai khi m v n câ d§u tr¡i nhau v… hai nßa cıa h…nh S2j.
  • 65. Tải tài liệu tại sividoc.com Viết đề tài giá sinh viên – ZALO:0973.287.149-TEAMLUANVAN.COM 56 chœ nh“t câ m u ng÷æc nhau. Gi£ sß m l c⁄nh l·. Khi â n‚u ta k†o d i c⁄nh º d i m bði 1 ( ìn và) th… s‡ t⁄o ra mºt tam gi¡c mîi chøa tam gi¡c ban ƒu nh÷ng nâ câ c£ hai c⁄nh •u chfin v do â S1 = S2. Tr÷íng hæp x§u nh§t l t§t c£ c¡c di»n t‰ch n y còng m u. Trong tr÷íng hæp â ta câ f(m; n) = n 2. Nh÷ng n max(m; n) n¶n ta câ i•u ph£i chøng minh. B i to¡n 3.1. (Thi væ àch Moscow, 1961) Cho mºt b£ng æ vuæng câ k‰ch th÷îc 4 4. Chøng minh r‹ng câ th” °t 7 d§u v o c¡c æ cıa b£ng sao cho n‚u xâa 2 h ng ho°c 2 cºt b§t ký cıa b£ng th… v¤n câ ‰t nh§t 1 d§u ch÷a bà xâa. Chøng minh r‹ng n‚u sŁ d§u nhä hìn 7 th… luæn câ th” xâa 2 h ng v 2 cºt ” måi d§u •u bà xâa. B i to¡n 3.2. (Thi v o lîp chuy¶n To¡n-Tin H Nºi, Amsterdam, 1998) Cho h…nh vuæng c⁄nh n(n l sŁ nguy¶n lîn hìn 1) ÷æc chia th nh n n æ vuæng nhä. Trong mØi æ vuæng nhä n y ch¿ ghi mºt trong 3 sŁ 1,0,-1. H…nh vuæng nh÷ th‚ ÷æc gåi l "b£ng sŁ æ vuæng c⁄nh n". a. H¢y l“p mºt b£ng sŁ vuæng c⁄nh 6 sao cho tŒng c¡c sŁ ghi trong b£ng theo måi h ng, måi cºt •u kh¡c nhau. b. Câ hay khæng mºt b£ng sŁ c⁄nh n n o â m tŒng c¡c sŁ ghi trong b£ng theo måi h ng, måi cºt v theo hai ÷íng ch†o •u kh¡c nhau. B i to¡n 3.3. (Thi v o lîp chuy¶n To¡n-Tin H Nºi, ⁄i håc tŒng hæp th nh phŁ Hç Ch‰ Minh,1994) Cho b£ng k‰ch th÷îc 2n 2n æ vuæng. Ng÷íi ta ¡nh d§u v o 3n æ b§t ký cıa b£ng. Chøng minh r‹ng câ th” chån ra n h ng v n cºt cıa b£ng sao cho c¡c æ ÷æc ¡nh d§u •u n‹m tr¶n n h ng ho°c n cºt n y. B i to¡n 3.4. (IMO-1987,#5) Cho sŁ tü nhi¶n n 3. Chøng minh r‹ng câ mºt t“p hæp gçm n i”m trong m°t phflng sao cho kho£ng c¡ch giœa hai i”m b§t ký l mºt sŁ væ t v mØi bº 3 i”m x¡c ành mºt tam gi¡c khæng suy bi‚n câ di»n t‰ch hœu t . Ph÷ìng Ph¡p gi£i: Gåi xn l i”m câ tåa º (n; n2 ) vîi n = 1; 2; 3; : : :. Ta i chøng minh kho£ng c¡ch giœa hai i”m b§t ký l sŁ hœu t v tam gi¡c x¡c ành bði 3 i”m b§t ký câ di»n t‰ch hœu t kh¡c khæng.